100022 - exstatic.zbgedu.com

152

Upload: others

Post on 08-Jan-2022

1 views

Category:

Documents


0 download

TRANSCRIPT

Hot line: 400-699-1716

Q & A E-mail: [email protected]

Complaint E-mail: [email protected]

ACCA connect: http://www.accaglobal.com/contancts/connect/

Address: 北京市朝阳区广渠路金泰国际大厦 C 座(100022)

ACCA 官方电话:+44 (0)141 582 2000

ACCA- FR 2021-2022 Content

CONTENT

FR Introduction ........................................................................................................................................... 3

1. Aim of paper ............................................................................................................................................ 3

2. Main capabilities ...................................................................................................................................... 3

3. The syllabus .............................................................................................................................................. 4

4. Exam format ............................................................................................................................................. 4

5. Linking to other papers ............................................................................................................................ 5

6. Examinable documents Sep 2021 to Jun 2022 ........................................................................................ 5

Part A FRAMEWORK ....................................................................................................................................... 7

Chapter 1 IASB Framework .................................................................................................................................. 7

1. The IASB conceptual framework for financial reporting .......................................................................... 7

2. Regulatory framework ........................................................................................................................... 11

3. Not-for-profit and public sector entities ................................................................................................ 13

Part B IFRS STANDARDS (ACCOUNTING STANDARDS) ..................................................................................... 15

Chapter 2 IAS1 Presentation of Financial Statements ....................................................................................... 15

1. A complete set of financial statements comprises ................................................................................ 15

2. Single entity financial statements .......................................................................................................... 15

3. Illustration of group financial statements .............................................................................................. 18

Chapter 3 Non-Current Assets ........................................................................................................................... 21

1. IAS 16 Property, plant and equipment ................................................................................................... 22

2. IAS 40 Investment property ................................................................................................................... 27

3. IAS 38 Intangible assets ......................................................................................................................... 28

4. IAS 36 Impairment of assets .................................................................................................................. 32

5. IAS 23 Borrowing costs .......................................................................................................................... 35

6. IAS 20 Government grants ..................................................................................................................... 36

7. IFRS 5 Non-current assets held for sale and discontinued operations .................................................. 38

Chapter 4 Revenue............................................................................................................................................. 43

1. IFRS15 Revenue from contracts with customers ................................................................................... 43

2. IFRS15 Part construction contracts ........................................................................................................ 47

Chapter 5 Other Standards ................................................................................................................................ 52

1. IAS 8 Accounting policies, changes in accounting estimates and errors ................................................ 52

2. IAS 2 Inventories .................................................................................................................................... 54

3. IAS 41 Agriculture .................................................................................................................................. 56

4. IFRS 13 Fair value measurement ............................................................................................................ 58

Chapter 6 Financial Instruments ........................................................................................................................ 59

1. Need for IFRS Standards ........................................................................................................................ 59

2. IAS 32 Financial instruments: Presentation ........................................................................................... 59

3. IFRS 9 Financial instruments .................................................................................................................. 60

Chapter 7 Leases ................................................................................................................................................ 67

1. Objective of IFRS 16 leases .................................................................................................................... 67

2. Accounting by lessee.............................................................................................................................. 67

3. Accounting treatment ............................................................................................................................ 69

1

ACCA- FR 2021-2022 Content

Chapter 8 Taxation ............................................................................................................................................. 72

1. IAS 12 Income tax................................................................................................................................... 72

2. Current tax ............................................................................................................................................. 72

3. Deferred tax ........................................................................................................................................... 72

Chapter 9 Foreign Currency Transactions and Entities ...................................................................................... 78

1. Why adjustments for foreign currency transactions are necessary ....................................................... 78

2. Functional and presentation currency ................................................................................................... 78

3. Accounting for individual transactions designated in a foreign currency .............................................. 78

Chapter 10 Events after the Reporting Period, Provision and Contingencies .................................................... 81

1. IAS 10 Events after the reporting period ............................................................................................... 81

2. IAS 37 Provision, contingent assets and contingent liabilities ............................................................... 83

Chapter 11 Earning per Share ............................................................................................................................ 90

1. Introduction ........................................................................................................................................... 90

2. Basic EPS ................................................................................................................................................ 90

3. Diluted earnings per share ..................................................................................................................... 93

4. The importance of EPS & DEPS .............................................................................................................. 95

5. Limitations of EPS as a performance indicator ...................................................................................... 95

Chapter 12 Statement of Cash Flows ................................................................................................................. 96

1. The need for a statement of cash flows ................................................................................................. 96

2. Definitions .............................................................................................................................................. 96

3. Preparation of statement of cash flows ................................................................................................. 97

4. Interpretation of cash flow .................................................................................................................... 99

5. Limitations of the statement of cash flow ........................................................................................... 101

Part C CONSOLIDATION ACCOUNTS .............................................................................................................. 105

Chapter 13 The Consolidation Statement of Financial Position ...................................................................... 105

1. The nature of business combination ................................................................................................... 105

2. Accounting issues ................................................................................................................................. 106

3. The single entity concept ..................................................................................................................... 106

4. Basic principle of consolidation financial statement ........................................................................... 107

5. Basic for consolidation statement of financial position ....................................................................... 107

6. Explanations of group account--IFRS 3 business combination ............................................................ 109

Chapter 14 The Consolidated Statement of Profit or Loss and Other Comprehensive Income ...................... 122

1. Accounting for subsidiary companies .................................................................................................. 122

2. Accounting for disposal of a subsidiary ............................................................................................... 125

Chapter 15 Accounting for Associates ............................................................................................................. 127

1. Defining an associated undertaking ..................................................................................................... 127

Part D ANALYSING AND INTERPRETING FINANCIAL STATEMENTS OF SINGLE ENTITIES AND GROUPS .............. 131

Chapter 16 Interpretation of Financial Statements ......................................................................................... 131

1. Interpreting financial information ....................................................................................................... 131

2. Ratio analysis ....................................................................................................................................... 132

2

ACCA- FR 2021-2022 Introduction

FR Introduction 1. Aim of paper

To develop knowledge and skills in understanding and applying IFRS Standards(Accounting standards) and the

theoretical framework in the preparation of financial statements of entities, including groups and how to analyze

and interpret those financial statements.

2. Main capabilities

On successful completion of this exam, candidates should be able to:

A Discuss and apply a conceptual and regulatory frameworks for financial reporting.

B Account for transactions in accordance with IFRS Standards.

C Analyse and interpret financial statements.

D Prepare and present financial statements for single entities and business combinations in accordance with IFRS

Standards.

E Demonstrate employability and technology skills

Computer-based exams

The syllabus is assessed by a three hour computer based examination.

All questions are compulsory. The exam will contain both computational and discursive elements. Some

questions will adopt a scenario/case study approach.

Section A of the computer-based exam comprises 15 objective test questions of 2 marks each plus additional

content as per below.

Section B of the computer-based exam comprises three questions each containing five objective test questions

plus additional content as per below.

Section C of the exam comprises two 20 mark constructed response questions.

The 20 mark questions will examine the interpretation and preparation of financial statements for either a

single entity or a group. The section A questions and the other questions in section B can cover any areas of

the syllabus.

The conceptual and regulatory framework for

financial reporting ( A )

Preparation of financial statements ( D )

Accounting for transactions in financial

statements ( B )

Analysing

and

interpreting

financial

statements( C )

Employability

and

technology

skills ( E )

3

ACCA- FR 2021-2022 Introduction

3. The syllabus

3.1 The conceptual and regulatory framework for financial reporting

3.1.1 The need for a conceptual framework the characteristics of useful information

3.1.2 Recognition and measurement

3.1.3 Regulatory framework

3.1.4 The concepts and principles of groups and consolidated financial statements

3.2 Accounting for transactions in financial statements

3.2.1. Tangible non-current assets

3.2.2. Intangible assets

3.2.3. Impairment of assets

3.2.4. Inventory and biological assets

3.2.5. Financial instruments

3.2.6. Leasing

3.2.7. Provisions and events after the reporting period

3.2.8. Taxation

3.2.9. Reporting financial performance

3.2.10. Revenue

3.2.11. Government grants

3.2.12. Foreign currency transactions 3.3 Analysing and interpreting financial statements of single entities and groups

3.3.1. Limitations of financial statements

3.3.2. Calculation and interpretation of accounting ratios and trends to address users’ and stakeholders’ needs

3.3.3. Limitations of interpretation techniques

3.3.4. Specialised, not-for-profit, and public sector entities 3.4 Preparation of financial statements

3.4.1. Preparation of single entity financial statements

3.4.2. Preparation of consolidated financial statements including an associate 3.5 Employability and technology skills

Use computer technology to efficiently access and manipulate relevant information

Work on relevant response options, using available functions and technology, as would be required in the

workplace.

Navigate windows and computer screens to create and amend responses to exam requirements, using the

appropriate tools.

Present data and information effectively, using the appropriate tools.

4. Exam format

Format of the paper

Marks

Section A – 15 MCQs 30

Section B – three case questions (10 marks each) 30

Section C – two constructed response questions (20 marks each) 40

100

4

ACCA- FR 2021-2022 Introduction

5. Linking to other papers

6. Examinable documents Sep 2021 to Jun 2022

Title FR SBR INT

and UK

International Accounting Standards (IAS standards)/International

Financial Reporting Standards (IFRS standards)

IAS 1 Presentation of Financial Statements ✓ ✓

IAS 2 Inventories ✓ ✓

IAS 7 Statement of Cash Flows ✓ ✓

IAS 8 Accounting Policies, Changes in Accounting Estimates and Errors ✓ ✓

IAS 10 Events after the Reporting Period ✓ ✓

IAS 12 Income Taxes ✓ ✓

IAS 16 Property, Plant and Equipment ✓ ✓

IAS 19 Employee Benefits ✓

IAS 20 Accounting for Government Grants and Disclosure of Government

Assistance ✓ ✓

IAS 21 The Effects of Changes in Foreign Exchange Rates ✓ ✓

IAS 23 Borrowing Costs ✓ ✓

IAS 24 Related Party Disclosures ✓

IAS 27 Separate Financial Statements ✓ ✓

IAS 28 Investments in Associates and Joint Ventures ✓ ✓

IAS 32 Financial Instruments: Presentation ✓ ✓

IAS 33 Earnings per Share ✓ ✓

IAS 34 Interim Financial Reporting ✓

IAS 36 Impairment of Assets ✓ ✓

IAS 37 Provisions, Contingent Liabilities and Contingent Assets ✓ ✓

IAS 38 Intangible Assets ✓ ✓

IAS 40 Investment Property ✓ ✓

IAS 41 Agriculture ✓ ✓

IFRS 1 First-time Adoption of International Financial Reporting Standards ✓

IFRS 2 Share-based Payment ✓

IFRS 3 Business Combinations ✓ ✓

Strategic Business Reporting

( SBR )

Strategic Business Leader

(SBL )

Financial Accounting ( FA )

Corporate and Business

Law ( LW )

Audit and Assurance

( AA ) Financial Reporting ( FR )

5

ACCA- FR 2021-2022 Introduction

IFRS 5 Non-Current Assets Held for Sale and Discontinued Operations ✓ ✓

IFRS 7 Financial Instruments: Disclosures ✓ ✓

IFRS 8 Operating Segments ✓

IFRS 9 Financial Instruments ✓ ✓

IFRS 10 Consolidated Financial Statements ✓ ✓

IFRS 11 Joint Arrangements ✓

IFRS 12 Disclosure of interests in Other Entities ✓

IFRS 13 Fair Value Measurement ✓ ✓

IFRS 15 Revenue from Contracts with Customers ✓ ✓

IFRS 16 Leases ✓ ✓

IFRS for SMEs IFRS for Small and Medium Sized Entities ✓

Other Statements

Conceptual Framework for Financial Reporting (Conceptual Framework)

(March, 2018) ✓ ✓

The International <IR> Framework ✓

IFRS Practice

Statement Management Commentary ✓

IFRS Practice

Statement Making Materiality Judgements ✓

EDs, Discussion Papers and Other Documents

ED/ 2019/7** General Presentation and Disclosures ✓

** The content of the IASB Snapshot: General Presentation and Disclosures (issued December 2019) is sufficient for

the SBR examination

6

ACCA- FR 2021-2022 Part A-Chapter 1

Part A FRAMEWORK

Chapter 1 IASB Framework

Content

1. The IASB conceptual framework for financial reporting

2. Regulatory framework

3. Not-for-profit and public sector entities

1. The IASB conceptual framework for financial reporting

1.1 The meaning of a conceptual framework

A conceptual framework is a coherent system of interrelated objectives and fundamental principles. These

theoretical principles provide the basis for the development of new IFRS Standards and the evaluation of those

already in existence. The financial reporting process is concerned with providing information that is useful in

the business and economic decision-making process. Therefore a conceptual framework will form the

theoretical basis for determining which events should be accounted for, how they should be measured and

how they should be communicated to the user.

⚫ There are two main approaches to accounting:

A principles-based or conceptual framework approach such as that used by the IASB.

A rules-based approach such as that used in the US.

⚫ Why has a conceptual framework?

There are a variety of arguments for having a conceptual framework.

It enables IFRS Standards and generally accepted accounting practice(GAAP) to be developed in

accordance with agreed principles.

It avoids ‘fire-fighting’, whereby IFRS Standards are developed in a piecemeal way in response to specific

problems or abuses. ‘Fire-fighting’ can lead to inconsistencies between different IFRS Standards, and

between IFRS Standards and legislation.

Lack of a conceptual framework may mean that certain critical issues are not addressed, e.g. until recently

there was no definition of basic terms such as ‘asset’ or ‘liability’ in any accounting standard.

As transactions become more complex and businesses become more sophisticated it helps preparers and

auditors of accounts to deal with transactions which are not the subject of an accounting standard. 1.2 Objectives of Framework

⚫ Defines the objective of financial reporting; identifies the qualitative characteristics that make

information in financial statements useful; and

⚫ Defines the basic elements of financial statements and the concepts for recognizing and measuring them

in financial statements. 1.3 Objective of Financial Statements

⚫ The objective of financial statements is to provide information about the financial position, financial

performance and cash flows of an entity that is useful to a wide range of users in making economic decisions.

Financial statements also show the results of the management’s stewardship of the resources entrusted to

it.

7

ACCA- FR 2021-2022 Part A-Chapter 1

⚫ The Conceptual Framework makes it clear that this information should be prepared on an accruals basis.

Accruals basis. The effects of transactions and other events are recognised when they occur (and not as

cash or its equivalent is received or paid) and they are recorded in the accounting records and reported

in the financial statements of the periods to which they relate.

1.4 Underlying Assumptions

The Framework sets out the going concern assumptions of financial statements:

⚫ Going Concern assumes that the entity has neither the need nor the intention to liquidate or curtail

materially the scale of its operations. If this is not the case then the financial statements would be prepared

on a different basis.

1.5 Qualitative Characteristics of Financial Statements

If financial information is to be useful, it must be relevant and faithfully represent what it purports to represent.

The usefulness of financial information is enhanced if it is comparable, verifiable, timely and understandable.

⚫ The fundamental qualitative characteristics are relevance and faithful representation.

⚫ Comparability, verifiability, timeliness and understandability are qualitative characteristics that enhance

the usefulness of information that is relevant and faithfully represented.

1.5.1 Fundamental qualitative characteristics

⚫ Relevance

Relevant information is capable of making a difference in the decisions made by users. It is capable of

making a difference in decisions if it has predictive value, confirmatory value or both. The relevance of

information is affected by its nature and its materiality.

Materiality. Information is material if omitting it or misstating it could influence decisions that users

make on the basis of financial information about a specific reporting entity.

⚫ Faithful representation

To be a perfectly faithful representation, a depiction would have three characteristics. It would be

complete, neutral and free from error.

A complete depiction includes all information necessary for a user to understand the phenomenon

being depicted, including all necessary descriptions and explanations.

A neutral depiction is without bias in the selection or presentation of financial information. This

means that information must not be manipulated in any way in order to influence the decisions of

users.

Free from error means there are no errors or omissions in the description of the phenomenon and

no errors made in the process by which the financial information was produced. It does not mean

that no inaccuracies can arise, particularly where estimates have to be made.

Substance over form is not a separate qualitative characteristic under the conceptual Framework.

The IASB says that to do so would be redundant because it is implied in faithful representation.

Faithful representation of a transaction is only possible if it is accounted for according to its

substance and economic reality.

8

ACCA- FR 2021-2022 Part A-Chapter 1

1.5.2 Enhancing qualitative characteristics

⚫ Comparability

Comparability is the qualitative characteristic that enables users to identify and understand similarities

in, and differences among, items. Unlike the other qualitative characteristics, comparability does not

relate to a single item. A comparison requires at least two items.

⚫ Verifiability

Verifiability helps assure users that information faithfully represents the economic phenomena it

purports to represent. It means that different knowledgeable and independent observers could reach

consensus that a particular depiction is a faithful representation.

⚫ Timeliness

Timeliness means having information available to decision-makers in time to be capable of influencing

their decisions. Generally, the older the information is the less useful it is. However, some information

may continue to be timely long after the end of a reporting period because, for example, some users

may need to identify and assess trends.

⚫ Understandability

Classifying, characterising and presenting information clearly and concisely makes it understandable.

Some phenomena are inherently complex and cannot be made easy to understand. Excluding

information about those phenomena from financial reports might make the information in those

financial reports easier to understand. However, those reports would be incomplete and therefore

potentially misleading.

Financial reports are prepared for users who have a reasonable knowledge of business and

economic activities and who review and analyse the information diligently. At times, even well-

informed and diligent users may need to seek the aid of an adviser to understand information about

complex economic phenomena.

1.6 The Elements of Financial Statements

Financial statements portray the financial effects of transactions and other events by grouping them into broad

classes according to their economic characteristics. These broad classes are termed the elements of financial

statements.

⚫ The elements directly related to financial position (statement of financial position) are:

Assets

Liabilities

Equity

⚫ The elements directly related to performance (statement profit or loss and other comprehensive income)

are:

Income

Expenses

⚫ The statement of cash flows reflects both income statement elements and changes in statement of financial

position elements.

9

ACCA- FR 2021-2022 Part A-Chapter 1

1.6.1 Definitions of the elements

⚫ Financial position

Asset. An asset is a present economic resource controlled by the entity as a result of past events.

Liability. A liability is a present obligation of the entity to transfer an economic resource as a result

of past events.

Equity. Equity is the residual interest in the assets of the enterprise after deducting all its liabilities.

⚫ Performance

Income. Income is increases in assets, or decreases in liabilities, that result in increases in equity,

other than those relating to contributions from holders of equity claims.

Expense. Expenses are decreases in assets, or increases in liabilities, that result in decreases in

equity, other than those relating to distributions to holders of equity claims.

1.6.2 Recognition of the elements of financial statements

⚫ An asset or liability is recognised only if recognition of that asset or liability and of any resulting income,

expenses or changes in equity provides users of financial statements with information that is useful, ie

with:

Relevant information about the asset or liability and about any resulting income, expenses or

changes in equity and

A faithful representation of the asset or liability and of any resulting income, expenses or changes

in equity.

⚫ Recognition is subject to cost constraints: the benefits of the information provided by recognising an

element should justify the cost of recognising that element.

1.6.3 Measurement of the elements of financial statements

⚫ Measurement involves assigning monetary amounts at which the elements of the financial statements

are to be recognised and reported.

⚫ The framework acknowledges that a variety of measurement bases are used today to different degrees

and in varying combinations in financial statements, including:

Historical cost

Fair value

Value in use

Current cost

⚫ Historical cost

An asset when it is acquired or created is the value of the costs incurred in acquiring or creating the

asset, comprising the consideration paid to acquire or create the asset plus transaction costs.

The historical cost of a liability when it is incurred or taken on is the value of the consideration

received to incur or take on the liability minus transaction costs.

Recent standards use the concept of fair value, which is defined by IFRS 13 as ‘the price that would be

received to sell an asset or paid to transfer a liability in an ordinary transaction between market

participants at the measurement date’.

10

ACCA- FR 2021-2022 Part A-Chapter 1

⚫ Current Value

Fair value

· Fair value is the price that would be received to sell an asset, or paid to transfer a liability, in an

orderly transaction between market participants at the measurement date.

Value in use

· Value in use is the present value of the cash flows, or other economic benefits, that an entity

expects to derive from the use of an asset and from its ultimate disposal.

· Fulfilment value is the present value of the cash, or other economic resources, that an entity

expects to be obliged to transfer as it fulfils a liability.

Current cost

· An asset is the cost of an equivalent asset at the measurement date, comprising the

consideration that would be paid at the measurement date plus the transaction costs that

would be incurred at that date.

· A liability is the consideration that would be received for an equivalent liability at the

measurement date minus the transaction costs that would be incurred at that date.

2. Regulatory framework

2.1 The regulatory framework

⚫ The regulatory framework of accounting in each country which uses IFRS is affected by a number of legislative

and quasi-legislative influences as well as IFRS:

National company law

EU directives

Security exchange rules

⚫ Why a regulatory framework is necessary?

Regulation of accounting information is aimed at ensuring that users of financial statements receive a

minimum amount of information that will enable them to make meaningful decisions regarding their

interest in a reporting entity. A regulatory framework is required to ensure that relevant and reliable

financial reporting is achieved to meet the needs of shareholders and other users.

IFRS Standards on their own would not be a complete regulatory framework. In order to fully regulate

the preparation of financial statements and the obligations of companies and directors, legal and market

regulations are also required.

2.2 Principles-based and rules-based framework

2.2.1 Principles-based framework:

⚫ Based upon a conceptual framework such as the IASB’s Framework.

⚫ IFRS Standards are set on the basis of the conceptual framework.

2.2.2 Rules-based framework:

⚫ ‘Cookbook’ approach.

⚫ IFRS Standards are a set of rules which companies must follow.

In the UK there is a principles-based framework in terms of the Statement of Principles and IFRS Standards and

a rules-based framework in terms of the Companies Act, EU directives and stock exchange rulings.

11

ACCA- FR 2021-2022 Part A-Chapter 1

2.3 The structure is set out below: (Optional)

2.3.1 International Financial Reporting Standards (IFRS) Foundation

The IFRS Foundation is the supervisory body for the IASB and responsible for governance issues and

ensuring each body is properly funded.

The objective of the IFRS Foundation include:

⚫ Develop a set of global IFRS Standards high quality which are understandable and enforceable.

⚫ Promote using and applying these standards.

⚫ Bring about the convergence of national and IFRS Standards.

2.3.2 International Accounting Standards Board (IASB)

The IASB is solely responsible for issuing International Financial Reporting Standards (IFRSs).

2.3.3 IFRS Interpretations Committee (IFRS IC)

IFRS IC issues rapid guidance on accounting matters where divergent interpretations of IFRSs have arisen

and these must be approved by the IASB. The interpretations cover both:

⚫ Newly identified financial reporting issues not specifically dealt with in IFRSs; or

⚫ Issues where unsatisfactory or conflicting interpretations have developed, or seem likely to develop in

the absence of authoritative guidance, with a view to reaching a consensus on the appropriate

treatment.

2.3.4 IFRS Advisory Council (IFRS AC)

The IFRS AC provides a forum for the IASB to consult a wide range of interested

parties affected by the IASB’s work, with the objective of :

⚫ Advising the Board on agenda decisions and priorities in the Board’s work,

⚫ Informing the Board of the views of the organizations and individuals on the council on major

standard-setting projects, and

⚫ Giving other advice to the Board or to the Trustees.

IFRS Foundation

IFRS IC

IFRS AC IASB

12

ACCA- FR 2021-2022 Part A-Chapter 1

2.4 IASB’s Standard setting process

The procedure for the development of an IFRS is as follows:

⚫ Setp1: During the early stages of a project, the IASB may establish an Advisory Committee to give advice on

issues arising in the project.

⚫ Step 2: IASB may develop and publish Discussion Papers for public comment.

⚫ Step 3: Following the receipt and review of comments, the IASB could issue a final International Exposure

Draft for public comment.

⚫ Step 4: Following the receipt and review of comments, the IASB could issue a final International Financial

Reporting Standard.

3. Not-for-profit and public sector entities

3.1 The main aims of not-for-profit and public sector entities are very different to those of profit-orientated entities:

Profit-orientated sector Not-for-profit / public sector

⚫ Financial aim is to make profit and increase

shareholder wealth.

⚫ Directors are accountable to shareholders.

⚫ External finance freely available in the form of

loans and share capital.

⚫ Financial aim is to achieve value for money /

provide service.

⚫ Managers are accountable to trustees /

government / public.

⚫ Finance limited to donations / government

subsidies.

3.2 IFRS Standards and not-for-profit and public sector entities

3.2.1 IFRS Standards are designed to:

⚫ Measure financial performance accurately and consistently.

⚫ Report the financial position accurately and consistently.

⚫ Account for the director’s stewardship of the resources and assets.

3.2.2 Not-for-profit and public sector organizations:

⚫ Do not aim to achieve a profit but will have to account for their income and costs.

⚫ Will have to account for their effectiveness, economy and efficiency.

⚫ Do not have to produce financial statements for the public (but in many cases may do so).

⚫ Some IFRS Standards will be relevant such as those relating to inventory, non-current assets, leasing,

etc. Others relating purely to reporting such as earnings per share (EPS) will not be so relevant.

13

ACCA- FR 2021-2022 Part A-Chapter 1

Example 1 Conceptual framework (Referred to Dec 2014 Q3)

Although most items in financial statements are shown at their historical cost, increasingly the IASB is requiring or

allowing current cost to be used in many areas of financial reporting.

Drexler acquired an item of plant on 1 October 20X2 at a cost of $500,000. It has an expected life of five years

(straight-line depreciation) and an estimated residual value of 10% of its historical cost or current cost as appropriate.

As at 30 September 20X4, the manufacturer of the plant still makes the same item of plant and its current price is

$600,000.

What is the correct carrying amount to be shown in the statement of financial position of Drexler as at 30

September 20X4 under historical cost and current cost?

Historical cost Current cost

$ $

A 320,000 600,000

B 320,000 384,000

C 300,000 600,000

D 300,000 384,000

Example 2 Regulatory framework

Which one of the following would not be an advantage of adopting IFRS?

A It would be easier for investors to compare the financial statements of companies with those of foreign

competitors.

B Cross-border listing would be facilitated.

C Accountants and auditors would have more defenses in case of litigation.

D Multinational companies could more easily transfer accounting staff across national borders.

14

ACCA- FR 2021-2022 Part B-Chapter 2

Part B IFRS STANDARDS (ACCOUNTING STANDARDS)

Chapter 2 IAS1 Presentation of Financial Statements

Content

1. A complete set of financial statements comprises

2. Single entity financial statements

3. Illustration of group financial statements

1. A complete set of financial statements comprises

⚫ Statement of financial position (SOFP)

⚫ Statement profit or loss and other comprehensive income (SPL&OCI; SOCI)

⚫ Statement of changes in equity (SOCIE)

⚫ Statement of cash flows (SOCF)

⚫ Accounting policies and explanatory notes

The first three statements are shown below; the cash flow statement will be discussed in much greater detail

later.

These formats are in the Implementation Guidance that accompanies IAS 1. They are not obligatory, but are

obviously of vital examination importance.

2. Single entity financial statements

2.1 Statement of financial position

XYZ Company

Statement of Financial Position as at 31 December 20X9

ASSETS $ $

Non-current assets

Property, plant and equipment X

Other intangible assets X

Investments in associates X

Investments X

X

Current assets

Inventories X

Trade receivables X

Cash and cash equivalents X

X

Total assets X

15

ACCA- FR 2021-2022 Part B-Chapter 2

EQUITY AND LIABILITIES

Equity and reserves

Share capital X

Share premium X

Other reserves X

Retained earnings X

Total equity X

Non-current liabilities

Long-term borrowings X

Deferred tax X

X

X

Current liabilities

Trade payables X

Short-term borrowings X

Proposed dividend X

Current tax payable X

X

Total equity and liabilities X

2.1.1 The current/non-current assets distinction

An asset should be classified as a current asset when it:

⚫ Is expected to be realised in, or is held for sale or consumption in, the normal course of the entity's

operating cycle*; or

⚫ Is held primarily for trading purposes or for the short-term and expected to be realised within 12

months of the end of the reporting period; or

⚫ Is cash or a cash equivalent asset which is not restricted in its use.

*The operating cycle of an entity is the time between the acquisition of assets for processing and their

realisation in cash or cash equivalents.

All other assets should be classified as non-current assets. Non-current assets include tangible, intangible,

operating and financial assets of a long-term nature. Other terms with the same meaning can be used (eg

'fixed', 'long-term').

2.1.2 The current/non-current liabilities distinction

A liability should be classified as a current liability when it:

⚫ Is expected to be settled in the normal course of the entity's operating cycle; or

⚫ Is held primarily for the purpose of trading; or

⚫ Is due to be settled within 12 months after the end of the reporting period; or when

⚫ The entity does not have an unconditional right to defer settlement of the liability for at least 12

months after the end of the reporting period.

All other liabilities should be classified as non-current liabilities.

16

ACCA- FR 2021-2022 Part B-Chapter 2

2.2 Statement profit or loss and other comprehensive income

⚫ The revised IAS1 requires to present ‘other comprehensive income’ items (such as revaluation gains and

losses, and actuarial gains and losses), as well as the usual income statement items, on the face of the primary

financial statements.

⚫ Total comprehensive income: is the realized profit or loss for the period, plus other comprehensive income.

⚫ Other comprehensive income is the income and expenses that are not recognized in the profit or loss (i.e.

they are recorded in reserve rather than as an element of the realized profit for the period). For the purposes

of FR, other comprehensive income includes any change in the revaluation surplus.

XYZ Company

Statement of comprehensive income for the year ended 31 December 20X9

$ $

Revenue x

Cost of sales (x)

Gross profit x

Distribution cost (x)

Administrative expense (x)

Operating profit x

Finance cost (x)

Profit before tax (PBT) x

Tax (x)

Profit for the period x

Other comprehensive income

Revaluation surplus on property, plant and equipment x

Reversal of revaluation surplus on property, plant and equipment (x)

Revaluation gain or loss on investment in equity instrument

( FVTOCI financial asset) x

x

Total comprehensive income x 2.3 Statement of Changes in Equity

XYZ Company

Statement of Changes in Equity for the year ended 31 December 20X9

Share

capital

Share

premium

Revaluation

reserve

Retained

earnings

Total

$000 $000 $000 $000 $000

Opening balance X X X X X

Changes in accounting policies X X

Total comprehensive income X/(X) X X

Revaluation transfer to RE (X) X 0

Dividends paid (X) (X)

Issue of share capital X X X

Closing balance X X X X X

17

ACCA- FR 2021-2022 Part B-Chapter 2

3. Illustration of group financial statements

3.1 XYZ Group – Consolidated statement of financial position as at 31 December 20X9

20X9 20X8

$000 $000

ASSETS

Non – current assets Property, plant and equipment 350,700 360,020

Goodwill 80,800 91,200

Other intangible assets 227,470 227,470

Investment in associates 100,150 110,770

Investment in equity ( FVTOCI financial asset) 142,500 156,000

901,620 945,460

Current assets

Inventories 135,230 132,500

Trade receivables 91,600 110,800

Other current assets 25,650 12,540

Cash and cash equivalents 312,400 322,900

564,880 578,740

Total assets 1,466,500 1,524,200

EQUITY AND LIABILITIES

Equity attributable to owners of the parent

Share capital 650,000 600,000

Retained earnings 243,500 161,700

Other components of equity 10,200 21,200

903,700 782,900

Non-controlling interest 70,050 48,600

Total equity 973,750 831,500

Non-current liabilities

Long-term borrowings 120,000 160,000

Deferred tax 28,800 26,040

Long-term provisions 28,850 52,240

Total non-current liabilities 177,650 238,280

Current liabilities

Trade and other payables 115,100 187,620

Short-term borrowings 150,000 200,000

Current portion of long-term borrowings 10,000 20,000

Current tax payable 35,000 42,000

Short-term provisions 5,000 4,800

Total current liabilities 315,100 454,420

Total liabilities 492,750 692,700

Total equity and liabilities 1,466,500 1,524,200

18

ACCA- FR 2021-2022 Part B-Chapter 2

3.2 Consolidated statement of comprehensive income for the year ended 31 December 20X9

20X9 20X8

$000 $000

Revenue 390,000 355,000

Cost of sales (245,000) (230,000)

Gross profit 145,000 125,000

Other income 20,667 11,300

Distribution costs (9,000) (8,700)

Administrative expenses (20,000) (21,000)

Other expense (2,100) (1,200)

Financial costs (8,000) (7,500)

Share of profit of associates(a) 35,100 30,100

Profit before tax 161,667 128,000

Income tax expense (40,417) (32,000)

Profit for the year from continuing operations 121,250 65,500

Loss for the year from discontinued operations - (30,500)

PROFIT FOR THE YEAR 121,250 35,000

Other comprehensive income:

Gains on property revaluation 933 3,367

Investments in equity instruments (24,000) 26,667

Income tax 5,834 (7,667)

TOTAL COMPREHENSIVE INCOME FOR THE YEAR 104,017 57,367

Profit attributable to:

Owners of the parent 97,000 25,900

Non-controlling interest 24,250 9,100

121,250 35,000

Total comprehensive income attributable to:

Owners of the parent 83,567 43,667

Non-controlling interest 20,450 13,700

104,017 57,367

Earnings per share (in currency units):

Basic and diluted 0.46 0.30

⚫ This means the share of associates profit attributable to owner of the associates, i.e. it is after tax and non-

controlling interests in the associates.

⚫ This illustrates the aggregated presentation, with disclosure of the current year gain or loss and

reclassification adjustment presented in the notes. Alternatively, a gross presentation can be used.

⚫ This means the share of associates other comprehensive income attributable to owners of the associates, i.e.

it is after tax and non-controlling interests in the associates.

⚫ The income tax relating to each component of other comprehensive income is disclosed in the notes.

19

ACCA- FR 2021-2022 Part B-Chapter 2

Example 1

Where equity dividends paid is presented in the financial statements?

A As a deduction from retained earnings in the statement of changes in equity.

B As a liability in the statement of financial position.

C As an expense in profit or loss.

D As a loss in ‘other comprehensive income’.

20

ACCA- FR 2021-2022 Part B-Chapter 3

Chapter 3 Non-Current Assets

Content

1.IAS16 Property, Plant and Equipment

2.IAS40 Investment property

3.IAS38 Intangible assets

4.IAS36 Impairment of assets

5.IAS23 Borrowing costs

6.IAS20 Government grant

7.IFRS5 Non-current asset held for sale and discontinued operations

Session Content

Tangible

Non-current Initial

IAS 40 assets costs

Investment

properties

Subsequent

expenditure

Finance

costs Depreciation

Revaluation

Indications

Impairment

CGU NRV &

Value in use

Treatment

Research & Recognition

Development

Problems

Criteria Treatment

Special : Non-current assets held for sale

Tangibles

Intangibles

21

ACCA- FR 2021-2022 Part B-Chapter 3

1. IAS 16 Property, plant and equipment

1.1 Definition

⚫ Property, plant and equipment are tangible assets held by an enterprise for more than one accounting period

for use in the production or supply of goods or services, for rental to others or for administrative purposes.

⚫ Owner occupied assets (used for ordinary business activities).

1.2 Recognition

⚫ Item of property, plant and equipment should be recognised as assets when:

It is probable that the future economic benefits associated with the asset will flow to the entity, and

The cost of the asset can be measured reliably. 1.3 Initial measurement

An item of property, plant and equipment shall be measured at its cost.

⚫ Purchase price, including import duties and non-refundable purchase taxes, after deducting trade discounts

and rebates.

⚫ Include all costs involved in bringing the asset into working condition

Site preparation cost

Initial delivery and handling costs

Installation costs

Testing costs whether the asset is functioning properly

Professional fees

Borrowing costs (Refer to IAS23)

⚫ Initial estimate of unavoidable costs of dismantling and removing the asset and restoring the site on which

it is located. Present value of these costs should be capitalized (Refer to IAS37 Provision, contingent assets

and liabilities).

⚫ Excluding:

Administration and general overheads

Set-up and similar pre- production costs

Initial operating losses before the asset reaches planned performance 1.4 Subsequent expenditure

Subsequent expenditure should be recognised in the carrying amount of an item of property, plant and

equipment if :

It enhances the economic benefits provided by the asset (this could be extending the asset’s life, and

expansion or increase the productivity of the asset).

It related to an overhaul or required major inspection of the asset- the costs associated with this should

be recognised in the carrying amount and depreciated over the time until the next overhaul or safety

inspection.

It is replacing a component of a complex asset. The replaced component will be derecognized. A complex

asset is an asset made up of a number of components, which each depreciate at different rates, Eg: an

aircraft would comprise body, engines and interior.

22

ACCA- FR 2021-2022 Part B-Chapter 3

⚫ All other subsequent expenditure should be recognized in the statement of profit or loss (expense), because

it merely maintains the economic benefits originally expected. Eg: the cost of general repairs should be

written off immediately.

1.5 Depreciation

⚫ Depreciation is the systematic allocation of the depreciable amount of an asset over its useful life.

⚫ It is in essence a cost allocation process in compliance with the accruals / matching principle. Each

accounting period must bear a charge for depreciation to reflect the usage of the asset and benefit it helps

to generate.

⚫ Depreciable amount is the cost of an asset less its residual value.

⚫ All assets with a finite useful life must be depreciated. Land has no finite life, therefore excluded from

depreciation.

⚫ Depreciation must be charged from the date the asset is available for use. This may be earlier than the date

it is actually brought into use, for example when staff needs to be trained to use it.

⚫ Depreciation is continued even if the asset is idle.

1.5.1 Depreciation methods

The depreciation method used should reflect as fairly as possible the pattern in which the asset’s economic

benefits are consumed by the entity.

⚫ Straight line

Annual Depreciation =Cost-residual value

Useful life or (cost-residual value)*depreciation rate %

⚫ Reducing balance

Annual Depreciation = Net book value opening balance* depreciation rate %

1.5.2 Changes in useful life or method

⚫ The useful lives of assets or depreciation method should be reviewed at least at each financial year-

end and, when necessary, revised. When a material change becomes necessary, the depreciation

charge for the current and future periods should be adjusted.

⚫ Change of useful life or method is a change in accounting estimate not a change in accounting policy.

⚫ Apply new rate/method to existing NBV, previous year accounts unchanged.

Straight line

Cost $20,000 Residual value $2000 Useful economic life 4 years

Annual depreciation: (20000-2000)/4=4500

Reducing balance

Cost $20,000 Reducing balance% 25%

Annual depreciation:

Y1: 20000*25%=5000

Y2: (20000-5000)*25%=3750

23

ACCA- FR 2021-2022 Part B-Chapter 3

1.5.3 Separate components

Some items of property, plant and equipment comprise separate components with different useful lives.

For example, an aircraft might itself have a life of 30 years while the seats and fabrics in the interior only

have a life of five years. In such situations the separate components should be capitalized as separate

assets and each depreciated over their useful lives.

1.6 Subsequent measurement

⚫ IAS 16 allows a choice of accounting treatment for PPE:

The cost model

The revaluation model

⚫ Accounting treatment

Cost model Revaluation model

Initial

recognition

Initially measured at cost

Subsequent

measurement

Cost

–accumulative depreciation

–impairment losses

Revalued amount – accumulative depreciation

– impairment losses

Dr: Cost

Dr: Acc depreciation

Cr: Other

comprehensive

income(OCI)

Dr: Acc

depreciation

Cr: OCI

Cr: Cost

Dr: Acc

depreciation

Cr: OCI

Depreciation Based on historical cost

Dr: Depreciation

Cr: Acc depreciation

Based on revalued amount

Dr: Depreciation

Cr: Acc depreciation

Any difference between actual depreciation and

historical depreciation may be transferred from RR to

RE:

Dr: Revaluation reserve

Cr: Retained earnings

Disposal Profit or loss on disposal =

Net proceeds–CV of the asset

Recognized as income or

expense in I/S

Profit or loss on disposal =

net proceeds – CV of the asset

Recognized as income or expense in I/S

Remaining balance may be transferred from RR to RE or

left in equity under the heading revaluation surplus

Dr: Revaluation reserve

Cr: Retained earnings

1.6.1 Revaluations

⚫ If the revaluation alternative is adopted, two conditions must be complied with.

Revaluations must subsequently be made with sufficient regularity to ensure that the carrying

amount does not differ materially from the fair value at each balance sheet date.

When an item of property, plant and equipment is revalued, the entire class of assets to which the

item belongs must be revalued.

24

ACCA- FR 2021-2022 Part B-Chapter 3

⚫ Revaluation gains are credited to other comprehensive income and equity (revaluation reserve), the

increase shall be recognized in profit and loss to the extent that it reveres a revaluation decrease

previously recognized in profit and loss.

⚫ Revaluation losses should be recognized as an expense to profit and loss. However the decrease shall

be recognized in other comprehensive income to the extent of any credit balance existing in the

revaluation surplus.

1.6.2 Depreciation of revalued asset

⚫ Must charge, based on valuation less residual value, over remaining useful economic life (UEL).

⚫ Whole charge must go to income statement for year.

⚫ Make annual reserve transfer (Revaluation reserve →Retained Earnings) for extra depreciation on

revalued amount compared to cost.

⚫ Journals:

$ $

Dr P/L – depreciation charge X

Cr Non-current asset (NBV) X

Dr Revaluation reserve X

Cr Retained earnings X

1.7 Disclosure

⚫ Illustration

Property, plant and equipment Land and building Plant Total

$ million $ million $million

Cost or valuation:

At I October 20X3 280 150 430

Additions 50 50

Revaluation (5 - 20) (15) nil (15)

At 30 September 20X4 265 200 465

Accumulated depreciation:

At I October 20X3 40 105 145

Charge for year 9 35 44

Revaluation (40) Nil (40)

At 30 September 20X4 9 140 149

Carrying value 30 September 20X4 256 60 316

The land and buildings were revalued by an appropriately qualified valuer on an existing use basis on 1 October

20X3. They are being depreciated on a straight-line basis over a 25-year life. Plant is depreciated at 20% per

annum on cost.

25

ACCA- FR 2021-2022 Part B-Chapter 3

Example 1 IAS 16 (Initial cost recognition)

Broadleaf company purchased a new machine during the year. The related costs were as follows:

$’000

List price 100

Installation 20

Pre-production testing 10

Insurance 2

Warranty 2

Maintenance 3 The company received a 10% trade discount on the list price. The installation should have cost $18,000 but Broadleaf

wasted $2,000 on installing the wrong machine supports at first. The maintenance occurred after the start of

production and was required by the warranty. Both the warranty and the insurance were for one year only. What is initial cost of new machine recognized by Broadleaf Co.

$

Example 2 IAS 16 (Revaluation)

On 1 Jan 20X8 Cheetah Ltd buys a non-current asset for $120,000. The asset has an estimated useful life of 20 years

with no residual value and is depreciated on a straight-line basis with a full year’s charge in the year of acquisition

and none in the year of disposal.

Cheetah Ltd’s year end is 31 Dec. On 31 Dec 20Y0, the asset will be included in the statement of financial position

at a cost of $120,000 with accumulated depreciation of $18,000.

On 1 Jan 20Y1, when the balance on the company’s accumulated profits was $500,000, the asset is revalued to

$136,000. The total useful life remains unchanged. Required:

1. Show the journal to record the revaluation.

2. Calculate the revised depreciation charge and show how it would be accounted for if reserve transfers as the

usage of the asset.

3. If the asset is revalued to $85,000, $102,000 on 1 Jan 20Y1, 1 Jan 20Y2 respectively, what are the double entries.

4. On 31 Dec 20Y0, the asset is sold for $127,000. Show how the disposal would be recorded.

Example 3 IAS 16 Past Exam Jun 2009 Q5

Flightline is an airline which treats its aircraft as complex non-current assets. The cost and other details of one of its

aircraft are:

$’000 estimated life

Exterior structure – purchase date 1 April 1995 120,000 20 years

Interior cabin fittings – replaced 1 April 2005 25,000 5 years

Engines (2 at $9 million each) – replaced 1 April 2005 18,000 36,000 flying hours

No residual values are attributed to any of the component parts.

At 1 April 2008 the aircraft log showed it had flown 10,800 hours since 1 April 2005. In the year ended 31 March

2009, the aircraft flew for 1,200 hours for the six months to 30 September 2008 and a further 1,000 hours in the six

months to 31 March 2009.

26

ACCA- FR 2021-2022 Part B-Chapter 3

On 1 October 2008 the aircraft suffered a ‘bird strike’ accident which damaged one of the engines beyond repair.

This was replaced by a new engine with a life of 36,000 hours at cost of $10·8 million. The other engine was also

damaged, but was repaired at a cost of $3 million; however, its remaining estimated life was shortened to 15,000

hours. The accident also caused cosmetic damage to the exterior of the aircraft which required repainting at a cost

of $2 million. As the aircraft was out of service for some weeks due to the accident, Flightline took the opportunity

to upgrade its cabin facilities at a cost of $4·5 million. This did not increase the estimated remaining life of the cabin

fittings, but the improved facilities enabled Flightline to substantially increase the air fares on this aircraft are: Required:

Calculate the charges to the income statement in respect of the aircraft for the year ended 31 March 2009 and its

carrying amount in the statement of financial position as at that date.

Note: the post-accident changes are deemed effective from 1 October 2008. (10 marks)

2. IAS 40 Investment property

2.1 Definition:

⚫ Investment property is defined by IAS 40 as land or a building held to earn rentals or for capital appreciation

or both, rather than for use in the enterprise or for sale in the ordinary course of business.

⚫ The following items are not investment properties

Accounted for under

Owner occupied property IAS 16

Property held for sale in the normal course of business IAS 2

Property being constructed for third parties IFRS 15 PART

2.2 Accounting treatment

⚫ Investment properties should initially be measured at cost.

⚫ IAS 40 then gives a choice between following a cost model or a fair value model.

2.2.1 Cost model--the asset should be accounted for in line with the benchmark treatment laid out in IAS 16.

2.2.2 Fair value model

⚫ The asset is revalued to fair value at the end of each year

⚫ The gain or loss is shown directly in the statement pf profit or loss (not OCI)

⚫ No depreciation is charged on the asset

Fair value is normally established by reference to current prices on an active market for properties in the

same location and condition.

If it is impossible to measure fair value reliably, then the cost-based model should be adopted.

It should not change from one model to the other unless the change will result in a more appropriate

presentation. IAS 40 states that it is highly unlikely that a change from the fair value model to the cost

model will result in a more appropriate presentation.

27

ACCA- FR 2021-2022 Part B-Chapter 3

2.2.3 Transfers

Transfer to or from investment property should only be made when there is a change in use.

⚫ Transfer from PPE to investment property and the fair value model for investment property

The asset must first be revalued per IAS 16 PPE and then transferred into investment property at FV

⚫ Transfer from investment to PPE, and the fair value model for investment property used.

Revalued the property first per IAS 40 and then transfer to PPE at FV

2.2.4 Disposal

Profit or loss on disposal = Net proceeds – Carrying amount of the asset, recognized as income or expense

in I/S.

Examples 1

Which of the following are examples of investment property?

(i) Land held for long-term capital appreciation rather than for short-term sale in the ordinary course of business;

(ii) A building that is vacant but is held to be leased out under one or more operating leases;

(iii) Land held for a currently undetermined future use;

(iv) Property occupied by employees;

A (i), (ii) and (iv)

B (i), (ii) and (iii)

C (i) and (ii)

D (i) and (iii)

Examples 2

Kingdom owns a building which it has been used as head office. At June 30 20X5, Kingdom let it out to a third party.

The building has an original cost on 1 January 20X0 at $ 500,000 and was being depreciated over 50 years. At 30

June 20X5, the fair value of this building is $480,000. At the year-end date of 31 December 20X5, the fair value of

this building is $510,000. Kingdom uses the fair value of investment property .

What amount will be shown in revaluation surplus at 31.December.20X5 in respect of this building?

A 35,000

B 45,000

C 55,000

D 65,000

3. IAS 38 Intangible assets

3.1 Introduction

⚫ When we considered IAS 16 Property, plant and equipment, we saw how businesses account for investment

in tangible assets. Many businesses invest significant amounts with the intention of obtaining future value

on areas such as:

Scientific/technical knowledge

Design of new processes and systems

Licenses and quotas

28

ACCA- FR 2021-2022 Part B-Chapter 3

Intellectual property e.g. Patents and copyrights

Market knowledge e.g. Customer lists, relationships and loyalty

Trademarks

The objective of IAS 38 is to prescribe the specific criteria that must be met an intangible asset can be recognized

in the accounts.

The standard covers all intangible assets other than those covered by another standard, which for our purposes

is Goodwill which is covered by IFRS 3.

3.2 Definition

⚫ An intangible asset is defined as, an identifiable non-monetary asset without physical substance. Therefore,

to meet the definition the asset must be:

Identifiable (is separable or arises from contractual /other legal rights);

Controlled by the entity as a result of past events;

A resource from which future economic benefits are expected to flow.

3.3 Recognition and measurement

To be recognized as an intangible asset must be meet

⚫ The definition of an intangible asset

⚫ The recognition criteria

It is probable that the future economic benefits attributable to the asset will flow to the entity

The cost of the asset can be measured reliably.

An intangible asset shall be measured initially at cost.

3.3.1 Acquired separately

⚫ If an intangible asset is purchased separately (such as a license, patent, brand name), it should be

recognized and initially measured at cost.

3.3.2 Acquisition as part of business combination

⚫ If an intangible is asset acquired as part of business combination, it should be recognized and initially

measured at fair value (and therefore the cost) as there is a rebuttable presumption that the fair value

of an intangible asset acquired in a business combination can be measured reliably.

⚫ However, an expenditure (included in the cost of acquisition) on an intangible item that does not meet

both the definition of and recognition criteria for an intangible asset should form part of the amount

attributed to the Goodwill recognized at the acquisition date.

3.3.3 Goodwill —IFRS 3 Treatment

⚫ Recognition and measurement of Goodwill. Goodwill is recognized by the acquirer as an asset from

the acquisition date;

⚫ No amortization of Goodwill. IFRS 3 prohibits the amortization of Goodwill. Instead Goodwill must be

tested for impairment at least annually in accordance with IAS 36 Impairment of Assets.

29

ACCA- FR 2021-2022 Part B-Chapter 3

⚫ Negative Goodwill. If the acquirer's interest in the net fair value of the acquired identifiable net assets

exceeds the cost of the business combination, that excess (sometimes referred to as Negative Goodwill)

must be recognized immediately in the income statement as a gain.

3.3.4 Internally Generated Goodwill

⚫ In some cases, expenditure is incurred to generate future economic benefits, but it does not meet the

recognition criteria of intangible asset. Such expenditure is often described as contributing to internally

generated goodwill. E.g.: advertising expenses.

⚫ Internally Generated Goodwill is not recognized as an asset because it is not an identifiable resource

(i.e. it is not separable nor does it arise from contractual or other legal rights) controlled by the entity

that can be measured reliably at cost.

3.3.5 Internally generated intangible assets

⚫ Generally, internally-generated intangible assets cannot be capitalised, as the cost associated with

these cannot be identified separately from the costs associated with running business.

The following internally generated items may never be recognized as assets:

Brands

Mastheads

Publishing titles

Customer lists

⚫ The production of internally generated assets is split into two phases: research and development.

Research costs: Original and planned investigation undertaken with the prospect of gaining new

scientific knowledge and understanding, which is charged to I/S as incurred.

Development costs: The application of research findings or other knowledge to a plan or design for

the production of' new or substantially improved materials, devices, products, processes, systems

or services before the start of commercial production or use.

Development costs are capitalized if they meet the criteria as follows:

• Technically feasible;

• Intention to complete;

• The availability of adequate technical, financial and other resources to complete;

• Ability to use or sell;

• Generation of future economic benefits: there must either be a market or an internal use for it;

• Ability to reliably measure the attributable expenditure on the project.

It is only expenditure incurred after the recognition criteria have been net which should be

recognized as an asset. Development expenditure recognized as an expense in P/L cannot

subsequently be reinstated as an asset.

30

ACCA- FR 2021-2022 Part B-Chapter 3

3.3.6 Acquisition in a Business Combination - In-process Research and Development

A research and development project acquired in a business combination is recognized as an asset at cost,

even if a component is research. Subsequent expenditure on that project is accounted for as any other

research and development cost (expensed except to the extent that the expenditure satisfies the criteria

in IAS 38 for recognizing such expenditure as an intangible asset).

3.3.7 Measurement after initial recognition

Two choices:

⚫ Cost model (more commonly used in practice): The accounting treatment is that intangible assets

should be carried at cost less amortization and any impairment losses.

⚫ Revaluation model: Intangible assets may be carried at a revalued amount (based on fair value) less

any subsequent amortization and impairment losses only if fair value can be determined by reference

to an active market. Such active markets are expected to be uncommon for intangible assets. Examples

where they might exist:

Milk quotas

Stock exchange seats

Taxi medallions

3.3.8 Amortisation

An entity must assess whether the useful life of an intangible asset is finite or indefinite.

With a finite useful life • Should be amortized on the pattern in which the asset's future

economic benefits are consumed.

• If such a pattern cannot be predicted reliably, the straight-line method

• Residual value assumed to be zero

With an indefinite useful

life

• Not amortized.

• Instead an impairment review must be carried out at least annually.

• The useful life of an intangible asset that is not amortized must be

reviewed each period to ensure that the life is still indefinite.

3.3.9 Disclosure

The same as IAS 16

Example 1

Which of the items will be recognized as intangible assets in the statement of financial position at the reporting

date?

(i) Licenses that are acquired from government;

(ii) Customer relationship that brings significant benefit to the business;

(iii) Self-owned trademarks;

(iv) Software that is acquired from outsiders and limited for 2-year use;

A (ii) and (iii)

B all four

C (i) and (ii)

D (i) and (iv)

31

ACCA- FR 2021-2022 Part B-Chapter 3

Example 2 (Referred to Jun 2015 Q2)

Which of the following statements relating to intangible assets is true?

(此题为综合考题,主体考核 IAS 38,穿插 IAS 36 内容(选项 D), 建议 IAS 36 学习后再次练习)

A All intangible assets must be carried at amortised cost or at an impaired amount; they cannot be revalued

upwards.

B The development of a new process which is not expected to increase sales revenues may still be recognised as

an intangible asset.

C Expenditure on the prototype of a new engine cannot be classified as an intangible asset because the prototype

has been assembled and has physical substance.

D Impairment losses for a cash generating unit are first applied to goodwill and then to other intangible assets

before being applied to tangible assets.

Example 3

Skyline is a company which commenced the development stage of a project to produce a new software on 1 October

20X4. The total expenditure of $60,000 incurred until the project was finished on 31 March 20X5 when the software

went into immediate production. The software became feasible on all aspects on 1 January 20X5. The software’s

estimated life span is five years. Skyline’s financial year end is 30 June. Time apportion is applicable when necessary.

What amount will be shown on Skyline’s statement of profit or loss for Development costs, including any

amortization for the year ended 30 June 20X5.

A $60,000

B $30,000

C $31,500

D $1,500

4. IAS 36 Impairment of assets

The objective of IAS 36 Impairment of assets is to set rules to ensure that the assets of an enterprise are carried

at no more than their recoverable amount (i.e. value to the business).

4.1 Definitions

⚫ Impairment: An asset has been impaired if its carrying value in the balance sheet exceeds its recoverable

amount.

⚫ Recoverable amount: It is the greater of fair value less cost to sell and value in use (future cash flows

generated by the asset discounted at a pre-tax market determined rate which allows for the risks specific to

the asset).

4.1.1 Measurement of FV less cost to sell / net selling price

⚫ Fair value is measured by following sequence:

Price in a binding sales agreement

Active market price

To estimate FV using best estimate (no active market)

⚫ Costs of disposal are the direct added costs only (not existing costs or overhead).

32

ACCA- FR 2021-2022 Part B-Chapter 3

4.1.2 Measurement of value in use

⚫ The discounted present value of estimated future cash flows expected to arise from the continuing use

of an asset, and from its disposal at the end of its useful life.

⚫ The calculation should reflect the following elements:

An estimate of the future cash flows;

Discount rate should be the pre-tax rate that reflects time value of money and the risks specific to

the asset. 4.2 Indications of impairment

The first step in applying IAS 36 is that at each balance sheet date, an enterprise should assess whether there

are any indications that the value of an asset may be impaired. You should think about any external or internal

factors that would affect either the selling price of the asset or the value that we can get out of it through using

it.

4.2.1 External sources of information

⚫ The asset's market value has declined more than expected.

⚫ Changes in the technological, market, economic or legal environment have had an adverse effect on

the enterprise.

⚫ Interest rates have changed, thus increasing the discount rate used in calculating the asset's value in

use.

⚫ The carrying amount of the net assets of the entity is more than its market capitalisation.

4.2.2 Internal sources of information

⚫ There is evidence of obsolescence or damage of the asset.

⚫ Changes in the way the asset is used have occurred or are imminent.

⚫ Evidence is available from internal reporting that indicates that the economic performance of an asset

is, or will be, worse than expected.

Even if there are no indications of impairment, the following asset must always be tested annually for

impairment.

⚫ Goodwill acquired in a business combination (see IFRS 3)

⚫ An intangible asset with an indefinite useful life (see IAS 38)

⚫ An intangible asset not yet available for use (see IAS 38)

4.3 Recognition and measurement of an impairment

⚫ If there is no indication of impairment then no further action need be taken. If there is an indication of

impairment, the recoverable amount should be calculated and any impairment loss should be immediately

recognized in the income statement.

⚫ The only exception to this is if the impairment reverses a previous gain taken to the revaluation reserve in

which case the impairment will be taken first to the revaluation reserve until the revaluation gain is reversed

and then to the income statement.

33

ACCA- FR 2021-2022 Part B-Chapter 3

4.4 Cash-generating units (CGUs)

⚫ In practice, cash flows belong to groups of assets rather than to individual assets. For example, a factory

production line will be made up of many individual machines and robots, but the revenues will be earned by

the production line as a whole.

⚫ Cash generating unit is the smallest identifiable group of assets that generates cash inflows from continuing

use and that are largely independent of the cash inflows from other assets.

⚫ Impairment should be calculated for the unit as a whole.

4.4.1 Impairment loss for a CGU

Goodwill is by definition not a separable asset and therefore should be allocated on acquisition to the

CGUs expected to benefit.

If a CGU is impaired the assets must be written down in a strict order:

⚫ Specific assets that are known to be impaired.

⚫ Goodwill allocated to the CGU.

⚫ Other assets (pro-rata according to carrying value).

Note: No individual asset should be written down below recoverable amount.

4.5 Reversal of previous impairment losses

⚫ Where events turn out to be better than earlier predicted and the recoverable amount increases as a result

of improved economic conditions then the previous write-down should be reversed. However the reversal

must not take the value of the asset above the original depreciable amount.

⚫ There are additional restrictions on goodwill and other intangibles. A reversal is only permissible when the

specific external event that caused the impairment has reversed.

Example 1

Certain items of machinery for Parker seem to have suffered an impairment. The stocks produced by the machines

are being sold below cost, leading to a reduction in the value of the machinery. The CV of the machinery based on

historical cost is $300,000. The FV less cost to sell is estimated at $120,000 and the expected cash flows from the

machines for the next 3 years is now $100,000 per annum. A market discount rate is 10% per annum. Required:

Describe how Parker should treat the above impairment in its accounts.

Example 2 (Referred to Dec 2014 Q18)

Which of the following is NOT an indicator of impairment?

A Advances in the technological environment in which an asset is employed have an adverse impact on its future

use.

B An increase in interest rates which increases the discount rate an entity uses.

C The carrying amount of an entity’s net assets is higher than the entity’s number of shares in issue multiplied by

its share price.

D The estimated net realisable value of inventory has been reduced due to fire damage although this value is

greater. than its carrying amount.

34

ACCA- FR 2021-2022 Part B-Chapter 3

Example 3

A cash generating unit (CGU) of Hubby has the following assets:

Items $

Manufacture machines 240,000

Allocated goodwill 60,000

Website 30,000

Working capital (at the realizable value) 20,000

Total amount 350,000

Due to the negative market movement, the recoverable amount of the CGU is measured at only $200,000.

What would be the value of the Manufacture machines after the allocation of the impairment loss?

A $200,000

B $180,000

C $160,000

D $140,000

5. IAS 23 Borrowing costs

5.1 Definition

⚫ IAS 23 Borrowing costs regulate the extent to which enterprises are allowed to capitalize borrowing costs

incurred on money borrowed to finance the acquisition of certain assets.

⚫ Borrowing costs must be capitalized as part of the cost of an asset, if that asset is a qualifying asset (one

which necessarily takes a substantial time to get ready for its intended use or sale).

5.2 The rate of interest to be taken

5.2.1 Borrowings specifically made to acquire a qualifying asset

⚫ If the borrowings are specially made, the borrowing costs which may be capitalized are those actually

incurred, less any investment income on the temporary investment of the borrowings.

5.2.2 Funds for the project taken from borrowings generally

⚫ If the funds come from general borrowings, the weighted average cost of general borrowings is taken.

This excludes borrowings with specific functions.

5.2.3 Commencement of capitalization

Capitalization of borrowing costs should commence when:

⚫ Expenditure for the asset is being incurred;

⚫ Borrowing costs are being incurred;

⚫ Activities that are necessary to prepare the asset for its intended use or sale are in progress.

5.2.4 Suspension of capitalization

An entity shall suspend capitalisation of borrowing costs during extended periods in which it suspends

active development of a qualifying asset.

35

ACCA- FR 2021-2022 Part B-Chapter 3

5.2.5 Cessation of capitalization

Capitalization of borrowing costs should cease when:

⚫ Once substantially all the activities necessary to prepare the qualifying asset for its intended use or

sale are complete, then capitalisation of borrowing costs should cease. This will normally be when

physical construction of the asset is completed, although minor modifications may still be outstanding.

⚫ The asset may be completed in parts or stages, where each part can be used while construction is still

taking place on the other parts. Capitalisation of borrowing costs should cease for each part as it is

completed. The example given by the standard is a business park consisting of several buildings.

5.2.6 Disclosures

⚫ The accounting policy adopted for borrowing costs;

⚫ The amount of borrowing costs capitalized during the period;

⚫ The capitalization rate used.

Example 1

A company constructed a supermarket whose useful life is 10 years at a cost of $50 million over six months from 1

January to 30 June. To assist the financing of the project the company raised a $40 million 10% loan on 1 January.

The loan was repaid on 30 September. The store did not open until the following year.

Required:

Prepare the statement of financial position and income statement as at 31 December.

Example 2

Which of the following statement is incorrect?

A When the loan is specifically issued to finance the building of a qualifying asset, the interest should be capitalized

immediately when the entity issues the loan.

B A supermarket which takes one year to build is an item of a qualifying asset.

C An entity shall suspend capitalisation of borrowing costs during extended periods in which it suspends active

development of a qualifying asset.

D An entity shall cease capitalising borrowing costs when substantially all the activities necessary to prepare the

qualifying asset for its intended use or sale are complete.

6. IAS 20 Government grants

6.1 Key definitions

⚫ Government grants are assistance in form of cash or assets in return for past of future compliance with

conditions relating to enterprise’s operating activity.

⚫ Grants related to assets. Government grants whose primary condition is that an entity qualifying for them

should purchase, construct or otherwise acquire non-current assets-know as capital grants.

⚫ Grants related to income. Government grants other than those related to assets-known as revenue grants.

36

ACCA- FR 2021-2022 Part B-Chapter 3

6.2 Accounting treatment

⚫ Recognize in income statement to match with expenditure towards which grants are intended to contribute

if:

All of the conditions for receipt have been complied with;

There is reasonable assurance that the grant will be received.

Accounting treatment Repayment

Grants

related

to assets

Treated as deferred income and released

to I/S over the life of asset

(so offsetting the higher depreciation

charge on the original cost)

1st: Debit the repayment to any provision for

deferred income

2nd: Any excess charged to I/S immediately

Deducted from the cost of the asset and

depreciate the reduced cost (released to

I/S over the life of asset by means of

reduced depreciation charge)

1st: Debit the cost of the asset with the

repayment

2nd: Cumulative additional depreciation

recognized and charged to I/S

Grants

related

to income

Matched to the related costs

Recognized as income

1st: Debit the repayment to any provision for

deferred income

2nd: Any excess charged to I/S immediately Matched to the related costs

Deducted from the related expenditure

Illustration

Europe Limited buys a building for a total cost of $200,000. They receive a European Community grant of $60,000

towards the cost and pay the net of $140,000 by cheque.

The useful economic life of the building is estimated at 50 years.

Required:

(a) As a deduction from NCA.

(b) As a deferred grant income.

Solution:

(a) Extract of SOFP

Building Year 1 Year2 Year 3 Year4 Year 50

Original Cost 140,000 140,000 140,000 140,000 140,000

Accumulated Depreciation (2,800) (5,600) (8,400) (11,200) (140,000)

Net Book Value 137,200 134,400 131,600 128,800 0

Extract of SOCI

Year 1 Year2 Year 3 Year4 Year 50

Depreciation Expense (2,800) (2,800) (2,800) (2,800) (2,800)

37

ACCA- FR 2021-2022 Part B-Chapter 3

(b) Extract of SOFP

Asset -Building Year 1 Year2 Year 3 Year4 Year 50

Original Cost 200,000 200,000 200,000 200,000 200,000

Accumulated Depreciation (4,000) (8,000) (12,000) (16,000) (200,000)

Net Book Value 196,000 192,000 188,000 184,000 0

Liability- deferred income 58,800 57,600 56,400 55,200 0

Extract of SOCI

Year 1 Year2 Year 3 Year4 Year 50

Other income

government grant

1,200 1,200 1,200 1,200 1,200

Depreciation Expense (4,000) (4,000) (4,000) (4,000) (4,000)

Examples 1 IAS 20 Government Grant

ABC’s year end is 30 September 20X5. ABC received a $500,000 government grant at 1 April 20X5 as the entity

purchased environmentally-friendly equipment, which was put into use at the same time. The list price of the

equipment is $2,000,000 and its useful life is five years with zero residual value.

It is ABC’s company policy to deduct the grant in calculating the carrying amount of the asset. The depreciation

method adopted by ABC is straight-line method.

What amount will ABC charge to profit or loss for the year ended 30 September 20X5?

A $150,000

B $200,000

C $300,000

D $400,000

7. IFRS 5 Non-current assets held for sale and discontinued operations

7.1 Objective

The objective of IFRS 5 is two-fold:

⚫ To set out requirements for the classification, measurement and presentation of non- current assets held for

sale, in particular requiring that such assets should be presented separately on the face of statement of

financial position.

⚫ To set out updated rules for the presentation of discontinued operational in particular requiring that the

results of discontinued operations should be presented separately in the income statement.

7.2 Classification as held for sale

7.2.1 A non-current asset should be classified as 'held for sale' if its carrying amount will be recovered principally

through a sale transaction rather than through continuing use.

7.2.2 For this to be the case, the following conditions must apply:

⚫ The asset must be available for immediate sale in its present condition

⚫ The sale must be highly probable, meaning that:

Management are committed to a plan to sell the asset;

38

ACCA- FR 2021-2022 Part B-Chapter 3

There is an active programme to locate a buyer; and

The asset is being actively marketed.

⚫ The sale is expected to be completed within 12 months of its classification as held for sale.

⚫ It is unlikely that the plan will be significantly changed or will be withdrawn.

7.2.3 A non-current asset acquired exclusively with a view to its subsequent disposal should be classified on

acquisition 'held for sale' as long as the above conditions apply. Note that, in order to qualify as held for

sale, assets must be expected to be disposed of through sale. Operations that are to be abandoned or

wound down gradually cannot be classified as held for sale, although they might meet the definition of

discontinued operations once they have been abandoned (see below).

7.3 Measurement of non-current assets held for sale

Non-current assets that qualify as held for sale should be measured at the lower of their

⚫ Carrying amount

⚫ Fair value less costs to sell

Any impairment loss recognised, to write down an asset to its fair value less costs to sell, Should normally be

charged to the income statement.

Held for sale non-current asset should be:

⚫ Presented separately on the face of the statement of financial position under current assets

⚫ Cease to be depreciated (or amortized).

Depreciation is a measure of consumption, which is no longer relevant as the asset will be sold rather than

consumed.

7.4 Discontinued operations

7.4.1 A discontinued operation is a component of an enterprise that either has been disposed of, or is classified

as held for sale, and:

⚫ Represents, a separate major line of business or geographical area of operations;

⚫ Is part of a single co-ordinated plan to dispose of a separate major line of business or geographical

area of operations; or

⚫ Is a subsidiary acquired exclusively with a view to resale.

7.4.2 Presentation in the income statement

⚫ An enterprise must disclose a single amount on the face of the income statement, comprising the total

of:

The post-tax profit or loss of discontinued operations; and

The post-tax gain or loss recognised on the measurement to fair value less costs to sell, or on the

disposal, of the assets constituting the discontinued operation.

⚫ An analysis of this single amount must be presented, either in the notes or on the face of the income

statement. The analysis must disclose:

The revenue, expenses and pre-tax profit or loss of discontinued operations.

The related income tax expense.

39

ACCA- FR 2021-2022 Part B-Chapter 3

The gain or loss recognized on the measurement to fair value less costs to sell, or on the disposal,

of the assets constituting the discontinued operation.

The related income tax expense.

Example illustrations of statement of profit or loss and other comprehensive income presentation

20X8

Continuing operations

Revenue X

Cost of sales (X)

Gross profit X

Distribution costs (X)

Administrative expenses (X)

Operating profit X

Finance costs (X)

Profit before tax X

Income tax expense (X)

Profit for the period from continuing operations X

Discontinued operations

Profit for the period from discontinued operations* X

Total profit for the period X

The analysis of this single amount would be given in the notes. Alternatively the analysis could be given

on the face of the income statement, with separate columns for continuing operations, discontinued

operations, and total.

7.4.3 Illustration:

St. Valentine produced cards and sold roses. However, half way through the year ended 31 March 20x6,

the rose business was closed and the assets sold off, incurring losses on the disposal of non-current assets

of $ 76,000 and redundancy costs of $ 37,000. the directors reorganized the continuing business at a cost

of $ 98,000.

Trading results may be summarized as follows:

Cards Roses

$ 000 $ 000

Turnover 650 320

Cost of Sales 320 150

Administration 120 110

Distribution 60 90

Other trading information (to be allocated to continuing operation) is as follows:

Total

$ 000

Interest Payable 17

Tax 31

Draft the Income Statement for the year ended 31 March 20x6.

40

ACCA- FR 2021-2022 Part B-Chapter 3

Solution:

Income statement for St Valentine for the year ended 31 March 20x6:

Continuing operations: $ 000

Revenue: 650

Cost of sales (320)

Gross profit 330

Administration costs (120)

Distribution costs (60)

Operating profit 150

Reorganization costs (98)

52

Finance cost (17)

Profit before tax 35

Income tax (31)

Profit for period from continuing operation 4

Discontinued operations:

Loss for the period from discontinued operation: (143)

Loss for period from total operations: (139)

In the notes to the accounts disclose analysis of the discontinued operation figure:

$ 000

Revenue: 320

Cost of sales (150)

Gross Profit 170

Administration costs (110)

Distribution costs (90)

Operating Loss (30)

Loss on disposal (76)

Redundancy costs (37)

Overall loss (143)

Example 1

On 1 January 20X1 Squawk purchased a machine for $100,000. It was expected that it would have a useful economic

life of eight years and a residual value of $20,000. However, during December 20X2, after only two years of use the

directors decide to sell the machine.

The company removes the machine from the farm in readiness for a quick disposal and prepares the machine for

viewing by potential purchasers; the Fair Value less costs to sell can be estimated reliably at $45,000. They appoint

an agent to assist with the marketing and advertising. The agents advise that the disposal may take two to six months.

Required:

Show the movement on the machine valuation during the year ended 31 December 20X2 and describe how the

asset would be disclosed on the statement of financial position at the year end.

41

ACCA- FR 2021-2022 Part B-Chapter 3

Example 2

Total Co has the following two lines of business that have been disposed of in the year:

Sector X operated in Country A. Total Co has no other operations in Country A, and Country A made up 0.5% of the

total revenue of Total Co.

Sector Y operated in the same country as the Total Co head office. It produced a different item from the other parts

of total Co, and this item contributed 10% of the total revenue of Total Co.

Which of these Sectors, if any, should be disclosed as a discontinued operation in the current year?

A Sector X is discontinued, Sector Y is not.

B Sector Y is discontinued, Sector X is not.

C Both Sector X and Sector Y are discontinued.

D Neither Sector X nor Sector Y are discontinued.

42

ACCA- FR 2021-2022 Part B-Chapter 4

Chapter 4 Revenue

Content

1.IFRS15 Revenue from contracts with customers

2.IFRS15 Part construction contracts

1. IFRS15 Revenue from contracts with customers

1.1 Definition

⚫ Revenue: Income arising in the course of an entity’s ordinary activities.

⚫ Contract: An agreement between two or more parties that creates enforceable rights and obligations.

⚫ Performance obligation: A promise in a contract with a customer to transfer to the customer either:

A good or service (or a bundle of goods or services) that is distinct; or

A series of distinct goods or services that are substantially the same and that have the same pattern of

transfer to the customer.

1.2 Recognition of revenue in accordance with core principle by applying the five steps

⚫ Step 1: Identify the contract(s) with a customer—a contract is an agreement between two or more parties

that creates enforceable rights and obligations. And the contract has commercial substance.

⚫ Step 2: Identify the performance obligations in the contract—a contract includes promises to transfer goods

or services to a customer. If those goods or services are distinct, the promises are performance obligations

and are accounted for separately.

A good or service is distinct if either of the following criteria is met:

The entity regularly sells the good or service separately’ or

The customer can benefit from the good or service either on its own or together with resources that are

readily available to the customer.

⚫ Step 3: Determine the transaction price—the transaction price is the amount of consideration in a contract

to which an entity expects to be entitled in exchange for transferring promised goods or services to a

customer. The transaction price can be a fixed amount of customer consideration, but it may sometimes

include variable consideration or consideration in a form other than cash such as discounts, rebates, refunds,

credits, incentives, bonuses, penalties. Under the proposal, these variable amounts would be estimated and

included in the transaction price using either the ‘expected value’ or the ‘most likely amount’ approach.

Additionally, the effect of time value of money or the fair value of any non-cash consideration should also be

included in the contract price.

⚫ Step 4: Allocate the transaction price to the performance obligations in the contract—an entity typically

allocates the transaction price to each performance obligation on the basis of the relative stand-alone selling

prices of each distinct good or service promised in the contract. If a stand-alone selling price is not observable,

an entity estimates it.

⚫ Step 5: A company would recognise revenue when (or as) it satisfies a performance obligation by transferring

a promised good or service to a customer (which is when the customer obtains control of that good or

service). For each performance obligation identified, an entity must determine at contract inception whether

it satisfied the performance obligation over time, or satisfies the performance obligation at a point in time.

43

ACCA- FR 2021-2022 Part B-Chapter 4

1.3 Performance obligation satisfied at point of time

⚫ To determine the point in time when a customer obtains control of a promised asset and an entity satisfies

a performance obligation, the entity would consider indicators of the transfer of control that include, but are

not limited to, the following:

The entity has a present right to payment for the asset;

The customer has legal title to the asset;

The entity has transferred physical possession of the asset;

The customer has the significant risks and rewards of ownership of the asset; and

The customer has accepted the asset.

1.4. Common types of transactions

Application notes to IFRS 15 provide guidance on how to deal with a number of different transactions.

1.4.1 Principal versus agent

An entity must establish in any transaction whether it is acting as principle or agent.

Principle Agent

If it control as the promised good or

service before it is transferred to the

customer.

If its performance obligation is to arrange for

the provision of goods or services by another

party.

Accounting

treatment

Recognizes revenue in the gross amount

of the consideration

Recognition of revenue in the amount of any

fee or commission

⚫ Indicators that an entity is an agent (and therefore does not control the good or service before it is

provided to a customer) include the following:

Another party is primarily responsible for fulfilling the contract;

The entity does not have inventory risk before or after the goods have been ordered by a customer,

during shipping or on return.

The entity does not have discretion in establishing prices for the other party’s goods or services and,

therefore, the benefit that the entity can receive from those goods or services is limited.

The entity’s consideration is in the form of a commission.

The entity is not exposed to credit risk for the amount receivable from a customer in exchange for

the other party’s goods or services.

44

ACCA- FR 2021-2022 Part B-Chapter 4

1.4.2 Repurchase agreements

⚫ A repurchase agreement is a contract in which an entity sells an asset and also promises or has the

option to repurchase it. This is often not recognised as a sale, but as a secured loan against the asset.

Repurchase agreements generally come in three forms:

an entity’s obligation to

repurchase the asset

an entity’s right to repurchase the

asset

an entity’s obligation to

repurchase the asset at the

customer’s request

a forward a call option a put option

Customer does not obtain control of the asset because the customer

is limited in its ability to direct the use of, and obtain substantially all

of the remaining benefits from, the asset even though the customer

may have physical possession of the asset.

It must consider whether or

not the customer is likely to

exercise that option.

Consequently, the entity shall account for the contract as either of the

following:

(a) a financing arrangement if the entity can or must repurchase the

asset for an amount that is equal to or more than the original

selling price of the asset.

(b) a lease in accordance with IFRS 16 Leases (see chapter 7 lease)

Accounting

treatment

Financing arrangement Real sales

On sales:

Continue to recognise the asset

Recognise the proceeds as a liability rather

than as income

1) recognise gain or loss on sales

2) derecognise asset

On repurchase: Charge interest on carrying value of loan

Repayment of loan Recognise asset

1.4.3 Consignment arrangements

⚫ When an entity delivers a product to another party (such as a dealer or a distributor) for sale to end

customers, the entity shall evaluate whether that other party has obtained control of the product at

that point in time.

Consignment

arrangement

A product that has been delivered to another party may be held in a consignment

arrangement if that other party has not obtained control of the product.

Accounting

treatment

Not recognise revenue upon delivery of a product to another party.

Indicators that an arrangement is a consignment arrangement include, but are not limited to, the

following:

The product is controlled by the entity until a specified event occurs, such as the sale of the product

to a customer of the dealer or until a specified period expires.

The entity is able to require the return of the product or transfer the product to a third party. (such

as another dealer)

The dealer does not have an unconditional obligation to pay for the product.

45

ACCA- FR 2021-2022 Part B-Chapter 4

1.4.4 Bill-and-hold arrangements

⚫ A bill-and-hold arrangement is a contract under which an entity bills a customer for a product but the

entity retains physical possession of the product until it is transferred to the customer at a point in

time in the future. For example, a customer may request an entity to enter into such a contract because

of the customer’s lack of available space for the product or because of delays in the customer’s

production schedules.

⚫ An entity shall determine when it has satisfied its performance obligation to transfer a product by

evaluating when a customer obtains control of that product.

Bill-and-hold arrangements

For some contracts, control is transferred

either when the product is delivered to

the customer’s site or is shipped,

depending on the terms of the contract.

For some contracts, a customer may obtain

control of a product even though that

product remains in an entity’s physical

possession.

Accounting

treatment

Recognise the revenue when customer

obtains control, usually at delivery.

Recognise the revenue when customer

obtains control.

There may be a fee for custodial service,

Recognises a fee for holding the goods on

behalf of the customer. This performance

obligation would be satisfied over time, so

any revenue would be recognised on this

basis.

⚫ For a customer to have obtained control of a product in a bill-and-hold arrangement, all of the

following criteria must be met:

The reason for the bill-and-hold arrangement must be substantive. (for example, the customer has

requested the arrangement)

The product must be identified separately as belonging to the customer.

The product currently must be ready for physical transfer to the customer.

The entity cannot have the ability to use the product or to direct it to another customer.

1.5 Performance obligation satisfied over time

⚫ If the performance obligation is satisfied over time, the revenue allocated to that performance obligation

will be recognised over the period the performance obligation is satisfied. It may include the following

situations:

The customer simultaneously receives and consumes the benefits provided by the entity’s performance

as the entity performs.

The entity’s performance creates or enhances an asset (Eg: WIP) that the customer controls as the asset

is created and enhanced.

The entity’s performance does not create an asset with an alternative use to the entity and the entity has

an enforceable right to payment for performance completed to date.

46

ACCA- FR 2021-2022 Part B-Chapter 4

In FR, a common application of this is likely to be a building company constructing as an asset for a customer.

As along as the building company is not able to use the asset, and has a right to payment for work to date,

revenue would be recognised over time. ( See IFRS15 part Construction Contracts)

1.5.1 Method for measuring progress

Methods for measuring the amount of performance completed to date encompass output methods and

input methods.

⚫ Output methods recognise revenue on the basis of the value to the customer of goods or services

transferred. They include surveys of performance completed, appraisal of units produced or delivered

etc.

⚫ Input methods recognise revenue on the basis of the entity’s inputs, such as resources consumed,

labor hours expended, cost incurred. If using a cost-based method, the cost incurred must contribute

to the entity’s progress in satisfying the performance obligation.

If revenue is earned equally over time, then revenue would be recognized on a straight line basis over that

period.

1.6 Contract costs

⚫ The incremental costs of obtaining a contract are recognised as an asset if the entity expects to recover those

costs. Costs recognised as asset are amortised on a systematic basis consistent with the transfer to the

customer of the goods or services to which the asset relates.

⚫ Costs that would have been incurred regardless of whether the contract was obtained as an expense as

incurred.

1.7 Presentation

When either party to a contract has performed, an entity shall present the contract in the statement of financial

position as a contract asset or a contract liability, depending on the relationship between the entity’s

performance and the customer’s payment.

⚫ A contract liability is an entity’s obligation to transfer goods or services to a customer for which the entity

has received consideration (or an amount of consideration is due) from the customer.

⚫ A contract asset is an entity’s right to consideration in exchange for goods or services that the entity has

transferred to a customer.

2. IFRS15 Part construction contracts

The primary issue in accounting for construction contracts is the allocation of contract revenue and contract

costs to the accounting period in which the construction is performed.

2.1 Definition

⚫ A construction contract is a contract specifically negotiated for the construction of an asset or a combination

of assets that are closely interrelated or interdependent in terms of design, technology and function of their

ultimate purpose or use.

47

ACCA- FR 2021-2022 Part B-Chapter 4

2.2 Identifying contract revenues and costs

Contract revenue Contract costs

Initial amount agreed

Variations

Claims

Incentive payments

Penalties ( reduce from contract revenue)

⚫ Direct costs

• Site labor and supervision

• Materials

• Plant&equipment depreciation

• Moving costs

• Hire charges

• Design

⚫ Attributable costs

• Insurance

• Overheads

2.3 Recognizing contract revenue and costs

⚫ Long-term construction contracts span more than one accounting year-end. This leads to the problem of

determining when and how the contract revenue and contract cost should be recognized.

Outcome can

be estimated

reliably

If the expected outcome is a profit:

Recognise revenue and costs by reference to stage of completion of contract.

If the expected outcome is a loss:

Recognise revenue by reference to stage of completion of contract.

Recognise the whole loss to P/L immediately regardless of the percentage completed.

Outcome

cannot be

estimated

reliably

Recognise contract costs as an expense in period incurred.

Recognise revenue only to the extent of contract costs expected to be recovered.

Thus no profit taken on contract

2.4 Presentation in financial statements

Statement profit or loss Statement of financial position

Revenue

Costs

Profit or loss

Contract asset

Contract liability

Format

Step 1: Calculate project profit or loss

Year 1 Year 2 Year 3

Contract price (including variation)

Less: Total estimated costs

Costs to date (excluding rectification cost )

Estimated costs to complete

Total estimated profit / (loss)

If there is a loss, recognise the whole loss immediately.

48

ACCA- FR 2021-2022 Part B-Chapter 4

Step 2: Calculate % completion to date and cumulative profit

⚫ Input method (eg. cost incurred to date/ total expected costs)

⚫ Output method (eg. work certified to date/ total contract price)

Cumulative profit = % × total expected profit

Current year profit = cumulative profit for the year – cumulative profit in prior year.

Step 3: Statement of profit or loss (if profitable)

Year 1 Year 2 Year 3

Revenue to date (Total price x progress %)

Less: previously recognised

→SPL

COS to date (Total cost x progress %)

Less: previously recognised

→SPL

Recognized profit / (loss) for year

Step 4: Statement of financial position (current asset/liability)

Year 1 Year 2 Year 3

Costs incurred (=cost to date + material unused + rectification

cost)

Add: Recognized profits / (losses) to date

Less: Progress billings

Contract asset / liability →SPF

Example 1– IFRS15 (RECOGNITION)

In January 20X2, customer A enters into two-year contracts with Wireless Company. Wireless Company offers a

handset that has been in the market for 18 months that the company is offering for free (the standalone selling

price is CU 350). Wireless company offers a 400-minuts wireless plan for CU40 per month over a two-year contact

period. For purpose of this example, assume the standalone selling price of the 400-minute wireless plan is CU40

per month.

Require:

How to recognise revenue when Wireless Company sells free handset with 400-miute wireless plan?

Example 2 (AGENCY)

During the year ended 31 March 20X7, Atlas made an amount of $20 million for cash sales and recognised the whole

amount in the total revenue without charging related cost of sales, through retail outlets on behalf of Francis. Atlas,

acting as agent, is entitled to a commission of 10% of the selling price of these goods. By 31 March 20X7, Atlas had

remitted to Francis $18 million (of the $20 million sales) and recorded this amount in cost of sales. Required: How to adjust this transaction for Atlas?

49

ACCA- FR 2021-2022 Part B-Chapter 4

Example 3 (CONSIGNMENT STOCK)

Angelino is a motor car dealer selling vehicles to the public. Most of its new vehicles are supplied on consignment

by manufacturers Monza.

Monza supplies cars on terms that allow Angelino to display the vehicles for a period of three months from the date

of delivery or when Angelino sells the cars on to a retail customer if this is less than three months. Within this period

Angelino can return the cars to Monza or can be asked by Monza to transfer the cars to another dealership (both at

no cost to Angelino). Angelino pays the manufacturer’s list price at the end of the three month period (or at the

date of sale if sooner). In recent years Angelino has returned several cars to Monza that was not selling very well

and has also been required to transfer cars to other dealerships at Monza’s request.

Required:

Describe how the above transactions and events should be treated in the financial statements of Angelino.

Example 4 (REPURCHASE AGREEMENT)

On 1 October 20X6 Atlas made a sale of $10 million of maturing inventory to Xpede, which has been included in the

revenue for the year ended 31 March 20X7. And Atlas has an option to repurchase these goods at any time within

three years of the sale at a price of $10 million plus accrued interest from the date of sale at 10% per annum. At 31

March 20X7 the option had not been exercised, but it is highly likely that it will be before the date it lapses. Required: What journal entry is required to reflect ALL the correct accounting treatment for the sales made to Xpede for the

year ended 31 March 20X7? Example 5 (CONSTRUCTION CONTRACT)

Lobden is a construction contract company involved in building commercial properties. Its policy for determining

the percentage of completion of its contracts is based on the proportion of cost incurred to date compared to the

total expected cost of the contract.

One of Lobden’s contracts has an agreed price of $250 million and estimated total costs of $200 million.

The cumulative progress of this contract is:

Year ended 30 September 20X7 30 September 20X8

$’million $’million

Costs incurred 80 145

Worked certified and billed 75 160

Cash received 70 150

1. What amount of the contract profit or loss would be recognised in Lobden’s statement of profit or loss for the

year ended 30 September 20X7?

A $25 million

B $24 million

C $20 million

D $18 million

50

ACCA- FR 2021-2022 Part B-Chapter 4

2. What amount would be reported in Lobden’s statement of financial position as at 30 September 20X7 for the

contract assets?

A $25 million

B $30 million

C $5 million

D $20 million

3. Most of Lobden’s competitors take profit based on the percentage of completion as determined by the work

certified compared to the contract price.

Assuming Lobden changes its method of determining the percentage of completion of contracts to that used

by its competitors, what percentage of completion would be:

%

4. What amount of the contract profit or loss would be recognised in Lobden’s statement of profit or loss for the

year ended 30 September 20X8 after the change?

A $16.25 million

B $12 million

C $32 million

D $36.25 million

5. What amount would be reported in Lobden’s statement of financial position as at 30 September 20X8 for the

contract assets after the change?

A $15 million

B $10 million

C $27 million

D $17 million

51

ACCA- FR 2021-2022 Part B-Chapter 5

Chapter 5 Other Standards

Syllabus content

1.IAS 8 Accounting policies, changes in accounting estimates and errors

2.IAS 2 Inventories

3.IAS 41 Agriculture

4.IFRS 13 Fair value measurement

Session Content

IAS 8 Accounting policies, changes in accounting estimates and errors

IAS 2 Inventories IAS 41 Agriculture

costs

IFRS 13 Fair value measurement

1. IAS 8 Accounting policies, changes in accounting estimates and errors

1.1 Selection of accounting policies

⚫ When an IAS or an IFRIC Interpretation applies to a transaction, management should select an Accounting

policy that complies with the standard/Interpretation, unless the effect of non-compliance would be

immaterial.

⚫ In the absence of a Standard or Interpretation that applies, management should select accounting policy in

compliance with the principles of the Framework, i.e. they should seek to maximize the relevance and

reliability of the information that is presented. Applicable concepts in the decision will be substance over

form, freedom from bias, and prudence.

⚫ Management may wish to refer to standards issued by other standard-setting bodies, or accounting literature

in general, when making their decision.

1.2 Changes in accounting policies

Accounting policies are normally kept the same from period to period to ensure comparability of financial

statements over time.

1.2.1 IAS 8 allows accounting policies to be changed only if:

⚫ It is required by statute or an accounting standard setting body; or

⚫ The change will result in a more relevant and reliable presentation of events or transactions.

Other standards

52

ACCA- FR 2021-2022 Part B-Chapter 5

1.2.2 A change in accounting policy occurs if there has been a change in:

⚫ Recognition, e.g. An expense is now recognized rather than an asset.

⚫ Presentation, e.g. Depreciation is now included in cost of sales rather than administrative expenses.

⚫ Measurement, e.g. Stating assets at replacement cost rather than historical cost.

1.2.3 The required treatment for dealing with changes policies is that

⚫ The change should be applied retrospectively.

⚫ With an adjustment to the opening balance of retained earnings in the statement of changes in equity.

⚫ Comparative information should be restated unless it is impracticable to do so.

1.2.4 When a change in accounting policy has a material effect on the current period or any prior period

presented, or may have a material effect in subsequent periods, the following disclosures should be

made:

⚫ The reasons for the change.

⚫ The amount of the adjustment recognized in the current period.

⚫ The amount of the adjustment included in each period prior to those included in the financial

statements.

1.3 Changes in accounting estimates

An accounting estimate is a method adopted by an entity to arrive at estimated amounts for the financial

statements.

1.3.1 Examples of changes in accounting estimates

⚫ The residual value of non-current assets

⚫ The useful lives of non-current assets

⚫ The depreciation methods

⚫ Warranty provisions

1.3.2 The effects of any change in accounting estimate should be included in the statement of profit or loss

and other comprehensive income of the period of the change and, if subsequent periods are affected,

in those subsequent periods ,i.e. prospective adjustment.

The effects of the change should be included in the same income statement classification as was used for the

original estimate.

1.4 What is the difference between a policy and an estimate

⚫ A change of accounting policy has occurred where there has been a change to any one of the following:

Measurement basis

Method of presentation

Recognition criteria

⚫ Conversely a change that does not affect any of the above is a change in estimation technique.

53

ACCA- FR 2021-2022 Part B-Chapter 5

Illustration 1 - depreciation of vehicles

An entity has previously depreciated vehicles using the reducing balance method at 40% per year. It now

proposes to depreciate vehicles using the straight-line method over five years. This decision does not involve a

change in any of the three key criteria:

⚫ Recognition

⚫ Measurement

⚫ Presentation

Therefore this is a change in estimate and not a change in accounting policy.

1.5 Correction of prior period errors

⚫ Prior period errors should be corrected retrospectively by adjustment against the opening balance of

retained earnings in the statement of changes in equity and restating the comparative figures.

Example 1 IAS 8

A company has previously failed to depreciate its building. This was due to a local culture of not depreciating both

land and buildings. It now accepts that Financial Reporting Standards require the depreciation of the building and

wishes to start depreciation this year. The building was purchased at the cost of $500,000; ten years prior to the

year start and has an estimated total life of 50 years.

The company also recognises that the application of the reducing balance method does not reflect the use of its

main machine. The machine cost $100,000; two years prior to the year start and a reducing balance of 20% had

been applied. It is now accepted that the machine will have constant use over a period of six years in total.

Required:

Describe the effect of the above on the financial statements.

Example 2 IAS 8

According to IAS 8, how should a material error in the previous financial reporting period be accounted for in the

current period?

A By making an adjustment in the financial statements of the current period through the statement of profit or

loss, and disclosing the nature of the error in a note.

B By making an adjustment in the financial statements of the current period as a movement on reserves, and

disclosing the nature of the error in a note.

C By restating the comparative amounts for the previous period at their correct value, and disclosing the nature

of the error in a note.

D By restating the comparative amounts for the previous period at their correct value, but without the

requirement for a disclosure of the nature of the error in a note.

2. IAS 2 Inventories

2.1 Key definitions

⚫ Inventories are assets:

Held for sale in the ordinary course of business;

In the process of production for such sale; or

In the form of materials or supplies to be consumed in the production process or in the rendering of

services.

54

ACCA- FR 2021-2022 Part B-Chapter 5

⚫ Net realisable value is the estimated selling price in the ordinary course of business less the estimated costs

of completion and the estimated costs necessary to make the sale.

⚫ Cost of inventories is the cost of bringing items of inventory to their present location and condition (including

cost of purchase and costs of conversion).

Cost of purchase comprises:

• Purchase price including import duties, transport and handling costs

• Any other directly attributable costs, less trade discounts, rebates and subsidies.

Cost of conversion comprises:

• Costs which are specifically attributable to units of production, e.g. direct labour, direct expenses

and subcontracted work.

• Production overheads, which must be based on the normal level of activity.

• Other overheads, if any, attributable in the particular circumstances of the business to bringing

the product or service to its present location and condition.

The following costs should be excluded and charged as expenses of the period in which they are

incurred:

Abnormal waste

Storage costs

Administrative overheads which do not contribute to bringing inventories to their present

location and condition.

Selling costs

2.2 Measurement of inventories

2.2.1 The standard states that 'Inventories should be measured at the lower of cost and net realisable value.'

2.2.2 Inventory valuation methods

⚫ IAS 2 deals with three methods of arriving at cost:

Actual unit cost

First in, first out (FIFO)

Weighted average cost (AVCO)

⚫ Where items of inventory are not ordinarily interchangeable, IAS2 requires the actual unit cost

valuation method to be used. Such items should be shown at their actual unit costs.

⚫ Where items are ordinarily interchangeable, the entity must choose between two cost formulae: the

FIFO method and the AVCO method.

2.3 Recognition as an expense

The following treatment is required when inventories are sold.

⚫ The carrying amount is recognised as an expense(specifically cost rather than operating expenses in SPL) in

the period in which the related revenue is recognised.

55

ACCA- FR 2021-2022 Part B-Chapter 5

⚫ The amount of any write-down of inventories to NRV and all losses of inventories are recognized as an

expense in the period the write-down or loss occurs.

⚫ The amount of any reversal of any write-down of inventories, arising from an increase in NRV, is recognised

as a reduction in the amount of inventories recognised as an expense in the period in which the reversal

occurs.

Example 1 Inventory valuation

Value the following items of inventory.

(a)Materials costing $12,000 bought for processing and assembly for a profitable special order. Since buying these

items, the cost price has fallen to $10,000.

(b)Equipment constructed for a customer for an agreed price of $18,000. this has recently been completed at a cost

of $16,800. It has now been discovered that , in order to meet certain regulations, conversion with an extra cost

of $4,200 will be required. The customer has accepted partial responsibility and agreed to meet half the extra

cost.

3. IAS 41 Agriculture

3.1 Key definitions

⚫ Agricultural activity is the management by an entity of the biological transformation and harvest of biological

assets for sale or for conversion into agricultural produce or into additional biological assets.

⚫ Agricultural produce is the harvested product of the entity’s biological assets.

⚫ A biological asset is a living animal or plant.

⚫ A group of biological assets is an aggregation of similar living animals or plants.

⚫ Harvest is the detachment of produce from a biological asset or the cessation of a biological asset’s life

processes.

⚫ Costs to sell are the incremental costs directly attributable to the disposal of an asset, excluding finance costs

and income taxes.

3.2 Recognition

⚫ An entity shall recognise a biological asset or agricultural produce when, and only when:

The entity controls the asset as a result of past events;

It is probable that future economic benefits associated with the asset will flow to the entity; and

The fair value or cost of the asset can be measured reliably.

In agricultural activity, control may be evidenced by, for example, legal ownership of cattle and the branding or

otherwise marking of the cattle on acquisition, birth, or weaning. The future benefits are normally assessed by

measuring the significant physical attributes. 3.3 Measurement

⚫ A biological asset shall be measured on initial recognition and at the end of each reporting period at its fair

value less costs to sell, except for the case stated in IAS 41 paragraph 30* where the fair value cannot be

measured reliably.

56

ACCA- FR 2021-2022 Part B-Chapter 5

*IAS 41 paragraph 30 stated in IAS 41 is : There is a presumption that fair value can be measured reliably for

a biological asset. However, that presumption can be rebutted only on initial recognition for a biological asset

for which quoted market prices are not available and for which alternative fair value measurements are

determined to be clearly unreliable. In such a case, that biological asset shall be measured at its cost less any

accumulated depreciation and any accumulated impairment losses. Once the fair value of such a biological

asset becomes reliably measurable, an entity shall measure it at its fair value less costs to sell. Once a non-

current biological asset meets the criteria to be classified as held for sale (or is included in a disposal group

that is classified as held for sale) in accordance with IFRS 5 Non-current Assets Held for Sale and Discontinued

Operations, it is presumed that fair value can be measured reliably.

⚫ Agricultural produce harvested from an entity’s biological assets shall be measured at its fair value less costs

to sell at the point of harvest. Such measurement is the cost at that date when applying IAS 2 Inventories or

another applicable Standard.

⚫ The fair value measurement of a biological asset or agricultural produce may be facilitated by grouping

biological assets or agricultural produce according to significant attributes; for example, by age or quality. An

entity selects the attributes corresponding to the attributes used in the market as a basis for pricing.

Example 1

If a biological asset whose quoted market prices are not available, the biological asset shall be measured

at_________

A its cost less any accumulated depreciation and any accumulated impairment losses.

B its realizable value

C its historical cost

D can’t be measured reliably

Example 2 (Refered Jun 2015 Q10)

To which of the following items does IAS 41 Agriculture apply?

(i) A change in the fair value of a herd of farm animals relating to the unit price of the animals.

(ii) Logs held in a wood yard.

(iii) Farm land which is used for growing vegetables.

(iv) The cost of developing a new type of crop seed which is resistant to tropical diseases.

A All four

B (i) only

C (i) and (ii) only

D (ii) and (iii) only

Example 3

How is a gain or loss arising on a biological asset recognized in accordance with IAS 41?

A Included in profit of loss for the year.

B Adjusted in retained earnings.

C Shown under other comprehensive income.

D Deferred and recognized over the life of the biological asset.

57

ACCA- FR 2021-2022 Part B-Chapter 5

4. IFRS 13 Fair value measurement

4.1 Definitions

⚫ Fair value is the price that would be received to sell an asset or paid to transfer a liability in an orderly

transaction between market participants at the measurement date.

4.2 Valuation techniques

Three widely used valuation techniques are the market approach, the cost approach and the income approach.

⚫ The market approach uses prices and other relevant information generated by market transactions involving

identical or comparable (i.e similar) assets, liabilities or a group of assets and liabilities, such as a business.

⚫ The cost approach reflects the amount that would be required currently to replace the service capacity of an

asset (often referred to as current replacement cost).

⚫ The income approach converts future amounts (e.g. cash flows or income and expenses) to a single current

(i.e. discounted) amount.

4.3 Inputs to valuation techniques

IFRS 13 establishes a hierarchy that categories the inputs to valuation techniques used to measure fair value as

follows:

⚫ Level 1 inputs comprise quoted prices (‘observable’) in active markets for identical assets and liabilities at

the measurement date. This is regarded as providing the most reliable evidence of fair value and is likely to

be used without adjustment.

⚫ Level 2 inputs are observable inputs, other than those included within Level 1 above, which are observable

directly or indirectly. This may include quoted prices for similar (not identical) assets or liabilities in active

markets, or prices for identical or similar assets and liabilities in inactive markets. Typically, they are likely to

require some degree of adjustment to arrive at a fair value measurement.

⚫ Level 3 inputs are unobservable inputs for an asset or liability, based upon the best information available,

including information that may be reasonably available relating to market participants. An asset or liability is

regarded as having been measured using the lowest level of inputs that is significant to its valuation.

Example 1

Which ONE of the following statements regarding IFRS13 is not true?

A Level 1 inputs are likely to be used without adjustment.

B Level 3 inputs are based on the best information available to market participants and are therefore regarded

as providing the most reliable evidence of fair value.

C Level 2 inputs may include quoted prices for similar (but not identical) assets and liabilities in active market.

D Level 1 inputs comprise quoted prices in active markets for identical assets and liabilities at the reporting date.

58

ACCA- FR 2021-2022 Part B-Chapter 6

Chapter 6 Financial Instruments

Content

1.Need for IFRS Standards

2.IAS 32 Financial instruments: Presentation

3.IFRS 9 Financial instruments

1. Need for IFRS Standards

⚫ The significant growth of financial instruments over recent years has outstripped the development of

guidance for their accounting.

⚫ The topic is of international concern, other national standard-setters are involved as well as the IASB.

⚫ There have been recent high-profile disasters involving derivatives which, while not caused by accounting

failures, have raised questions about accounting and disclosure practices.

2. IAS 32 Financial instruments: Presentation

2.1 Definition

⚫ A financial instrument is any contract that gives rise to a financial asset of one entity and a financial liability

or equity instrument of another entity.

2.1.1 Financial assets

⚫ Financial asset is any asset that is:

Cash

A contractual right to receive cash or another financial asset from another entity.

A contractual right to exchange financial assets / liabilities with another equity under conditions that

are potentially favorable.

⚫ Examples

Trade receivables

Share Options

Investment in equity shares

2.1.2 Financial liabilities

⚫ Financial liability is any liability that is a contractual obligation:

To deliver cash or another financial asset to another entity, or

To exchange financial assets / liabilities with another entity under conditions that are potentially

unfavorable or

That will or may be settled in the entity’s own equity instrument.

⚫ Examples

Trade payables

Debenture loans

Redeemable preference shares

59

ACCA- FR 2021-2022 Part B-Chapter 6

2.1.3 Illustration - Financial instruments:

Identify which of the following are financial instruments:

(a) Inventories

(b) Investment in ordinary shares

(c) Prepayments for goods and services

(d) Liability for income taxes

(e) A share option (an entity’s obligation to issue its own shares).

Solution:

(a) Inventory (or any other physical asset such as non-current assets) is not a financial instrument since

there is no present contractual right to receive cash or other financial instrument.

(b) An investment in ordinary shares is a financial asset since it is an equity instrument of another entity.

(c) Prepayment of goods and service are not financial instruments since the future economic benefit will

be the receipt of goods or services rather than a financial asset.

(d) A liability for income taxes is not a financial instrument since the obligation is statutory rather than

contractual.

(e) A share option is a financial instrument since a contractual obligation does exist to deliver an equity

instrument. Note, however, that an option to buy or sell an asset other than a financial instrument (e.g. a

commodity) would not qualify as a financial instrument.

3. IFRS 9 Financial instruments

3.1 Initial recognition

⚫ An enterprise should recognize a financial asset or a financial liability on its statement of financial position

when, and only when, it becomes a party to the contractual provisions of the instrument and measurement

at cost i.e. the fair value of the consideration given or received for it plus or minus transaction costs.

⚫ As examples of this principle:

An unconditional receivable should be recognized as an asset when the enterprise becomes a party to

the contract so that it has a legal right to receive cash.

A forward contract is recognized as an asset or liability on the commitment date, rather than waiting until

the closing date when the exchange actually takes place. 3.2 Financial assets

3.2.1 Classification

⚫ IFRS 9 classifies financial assets into the following three categories:

Amortised cost;

Fair value through other comprehensive income.

Fair value through profit or loss.

⚫ The classification is made on the basis of both:

The entity’s business model for managing the financial assets and

The contractual cash flows characteristics of the financial assets.

60

ACCA- FR 2021-2022 Part B-Chapter 6

⚫ A financial asset is classified as measured at amortised cost where:

The objective of the business model within which the asset is held is to hold assets in order to collect

contractual cash flows and

The contractual terms of the financial asset give rise on specified dates to cash flows that are solely

payments of principal and interest on the principal outstanding.

⚫ Financial asset is classified as measured at fair value that is other than a financial asset which is

measured at amortised cost. As for a further comment, if the fair value financial asset is for trading

purpose, it is classified as fair value through profit or loss. Otherwise (not for trading) ,it is classified

as fair value through other comprehensive income.

3.2.2 Initial measurement

⚫ Financial asset are initially measured at the fair value of the consideration given (ie, cost) plus

transaction costs that are directly attributable to the acquisition of the financial instrument.

3.2.3 Subsequently measurement

Financial Assets:

Categories Examples Transaction cost Subsequent

Measurement

Amortised cost with

interest recognized

in profit or loss

Redeemable preference

share

Debt instrument held to

maturity to gain interest

and principal.

Capitalise as CV

Amortised cost

⚫ should be valued at

amortized cost using

the effective interest

rate.

⚫ the effective interest

amount is credited to

the profit or loss.

FV through profit or

loss

Short term investment in

equity instrument (Shares)

or debt instrument held for

FV change (debentures)or

Derivative.

Expensed to P/L The changes in fair value

are recognized in profit

or loss.

FV through other

comprehensive

income

Long term investment in

equity instrument (Shares)

or debt instrument held for

FV change (debentures)

Capitalise as CV

The changes in fair value

are recognized in other

comprehensive income

until the asset is

derecognized when the

cumulative gain or loss

recognized in OCI is

recognized in retained

earnings.

61

ACCA- FR 2021-2022 Part B-Chapter 6

Illustration: Financial assets:

(1) A company invests $ 5,000 in 10% debentures. The debentures are repayable at a premium after 3

years. The effective rate of interest is 12%.

What amounts will be shown in Statement of profit or loss and Statement of financial position for

years 1-3?

(2) A company invested in 10,000 shares of a listed company in November 20X7 at a cost of $ 4.2 per

share. At 31 December 20X7, the shares have a market value of $ 4.9. The company is planning on

selling these shares in April 20X8.

Prepare extracts from Statement of profit or loss for 31 December 20X7 and Statement of financial

position at that date.

(3) A company invested in 20,000 shares of a listed company in October 20X7 at a cost of $3.8 per share.

At 31 December 20X7 the shares have a market value of $ 3.4. The company is not planning on selling

these shares in the short term.

Prepare extracts from Statement of profit or loss for 31 December 20X7 and Statement of financial

position at that date.

Solution:

(1) Assumed held to maturity

Statement of profit or loss

1 2 3

Investment income 600 612 625

Statement of financial position

1 2 3

Non-current asset

Investment 5,100 5,212 0

Working

Year opening investment income12% Cash received 10% closing

1 5000 600 (500) 5100

2 5100 612 (500) 5212

3 5212 625 (500)

(5337)

0

(2) This is a fair value through profit or loss asset as it is held for sale in the short term. Change in value

recognized in profit or loss.

Statement of profit or loss

Investment income (10,000 x (4.9-4.2) 7,000

Statement of financial position

Current Assets:

Investment (10,000 x 4.9) 49,000

(3) This is an available for sale investment.

Statement of profit or loss and other comprehensive income

Other comprehensive income

Loss on Investment: (8,000)

Statement of financial position

62

ACCA- FR 2021-2022 Part B-Chapter 6

Non-current asset

Investment (20,000 x 3.4) 68,000

Equity:

Other reserve: (8,000)

3.2.4 Factoring of receivables

Factoring of receivable is where a company transfers it receivables balances to another organisation (a

factor) for management and collection, and receivables an advance on the value of those receivables in

return.

Accounting problem

A genuine sale or a secured loan?

⚫ Accounting treatment and indicators

Legal: factoring with pass of legal title at sales

Substance: risk and rewards transferred at sales

Yes (without recourse) No (with recourse)

legal=substance, a real sale Legal=substance, a secured loan

Accounting

treatment

on sales:

1)Derecognize the receivables

2)Charge the difference between the

proceeds and the face value of the

receivables factored to the income

statement as irrecoverable debts

1)Continue to recognize the asset

2)Recognize the proceeds of sale as a

liability in the statement of financial

position

Subsequent

charges No

3) Interest and other factoring cost

should be accrued.

Indicators:

payment and

charges(slow-

moving risk)

Factor's transfer is for a single non-

returnable fixed sum deducting

charges

Finance cost is varying with collecting

speed

Recourse(bad

debt risk)

Factor has no recourse to seller for

loss Factor has full recourse to seller for loss

residual

amount

returnable

Factor is not required to return

residual amount above initial transfer

Factor is required to return residual

amount above initial transfer

3.3 Financial liabilities

3.3.1 Classification

⚫ IFRS 9 requires that financial liabilities into two categories

Fair value through profit or loss;

Amortised cost;

⚫ All financial liabilities should be measured at amortised cost, with the exception of financial liabilities

at fair value through profit or loss.

⚫ Financial liabilities are measured at fair value through profit or loss if

It is held for short-term profit-taking.

Or a derivative except it is for hedging purpose.

63

ACCA- FR 2021-2022 Part B-Chapter 6

3.3.2 Initial measurement

⚫ Financial liability are initially measured at the fair value of the consideration received (ie, cost) minus

transaction costs that are directly attributable to issue of the financial instrument.

3.3.3 Subsequently measurement

Financial liabilities

Categories Examples Transaction cost Subsequent Measurement

FV

derivative Expensed to P/L Gain or loss on FV change is

taken to profit or loss

Amortised cost

Payable/

Bank loan/

Debenture/

Redeemable

preference share

Capitalised to CV

(deduct the debt

capital)

Amortised cost

Carrying value of liability is

adjusted to achieve a

constant effective interest

rate over life of the liability.

3.4 Equity and liabilities

IAS 32 requires the classification of a financial liability, or its component parts, as a liability or as equity according

to the substance of the contractual arrangement.

⚫ An equity instrument is any contract that evidences a residual interest in the assets of an entity after deduct

all of its liabilities.

⚫ A financial instrument is a debt instrument if it contains obligations to transfer economic benefits even if its

legal form is shares i.e. redeemable preference shares or preference shares paying a fixed rate of dividend

are in substance debt and should be accounted for as such.

⚫ As convertible loan stock contains an embedded derivative (the conversion rights) so it will be necessary to

split the derivative from the host contract and account for both separately.

3.4.1 Compound instruments:

⚫ Some financial instruments – sometimes called compound instruments – have both a liability and an

equity component from the issuer's perspective. In that case, IAS 32 Financial Instruments:

Presentation requires that the component parts be accounted for and presented separately according

to their substance based on the definitions of liability and equity. For example: a convertible bond:

The value of a convertible bond consists of a liability component – the bond and.

An equity component – the value of the right to convert in due course to equity.

⚫ The two elements must be separately recognized in SOFP

The liability element

The equity element

3.4.2 Preference shares

⚫ If preference shares are irredeemable, they are classified as equity. Equity dividends declared are

reported directly in equity.

⚫ If preference shares are redeemable, they are classified as a financial liability.

This is because the terms of a preferred share provide for its redemption for a fixed or determinable

amount on a fixed or determinable date. Dividends on redeemable preference shares classified as a

liability are an expense in the I/S.

⚫ The accounting treatment is the same as straight debt as described before.

64

ACCA- FR 2021-2022 Part B-Chapter 6

Example 1 IFRS 9 Financial assets with FV

A company Jones buys quoted company’s shares for $40 million for trading purpose. Transactions cost are $1 million.

At the year end the value of the asset has risen to $60 million. Shortly after the year end the asset is sold for $70

million.

Required:

How should this be accounted for?

Example 2 IFRS 9 Financial assets with FV

If the above company is not planning on selling these shares in the short term, in other words, it doesn’t hold the

shares for trading purpose, how to account for it?

Example 3 IFRS 9 Financial assets with amortised cost

A company invests three bonds, all with a nominal value of $10,000 redeemable in two years. The effective rate of

interest is 10%. The company intends to collect the contractual cash flows which consist solely for repayments of

interest and capital.

a) The first has a coupon rate is 0% and the debenture is redeemed at a premium of $2,100.

b) The second has coupon rate of 0% and the debenture is issued at a discount of $1,736.

c) The third has a coupon rate of 2% and the debenture is issued at a discount of $500 and redeemed at a premium

f $1,075. Required:

How should these bonds be accounted for?

Example 4 IFRS 9 Financial liabilities with amortised cost

A company issues a bond with a nominal value of $10,000 redeemable in two years. The coupon rate is 2% and the

debenture is issued at a discount of $300, cost of issuing of $200 and redeemed at a premium of $1,075. the effective

interest rate is 10%.

Required:

How should the bond be accounted for?

Example 5 IAS 32 Past exam Jun 2008 Q5 Convertible loan

Pingway issued a $10 million 3% convertible loan note at par on 1 April 20X7 with interest payable annually in arrears.

Three years later, on 31 March 20Y0, the loan note is convertible into equity shares on basis of $100 of loan note

for 25 equity shares or it may be redeemed at par in cash at the option of the loan note holder. One of the company’s

financial assistants observed that the use of a convertible loan note was preferable to a non-convertible loan note

as the latter would have required an interest rate of 8% in order to make it attractive to investors. The assistant has

also commented that the use of a convertible loan note will improve the profit as a result of lower interest costs

and, as it is likely that the loan note holders will choose the equity option, the loan note can be classified as equity

which will improve the company’s high gearing position.

The present value of $1 receivable at the end of the year, based on discount rate of 3% and 8% can be taken as: 3% 8%

End of year 1 0.97 0.93 2 0.94 0.86 3 0.92 0.79

65

ACCA- FR 2021-2022 Part B-Chapter 6

Required:

Comment on the financial assistant’ observations and show how the convertible loan note should be accounted for

in Pingway’s income statement for the year ended 31 March 20X8 and statement of financial position as at that date.

(10 marks)

Example 6 Redeemable preference shares

On 1 October 20x4, a company issued 50,000 redeemable preference shares with a par value of $ 100 each to

investors at $ 55. The shares are redeemable at par on 30/09/20x9 and have a coupon rate of 2%. The effective rate

of interest is 15.62%.

Required:

how to show in SFP and I/S for the year ending 30/09/20x5 and 30/09/20x6?

66

ACCA- FR 2021-2022 Part B-Chapter 7

Chapter 7 Leases

Content

1.Objective of IFRS 16 leases

2.Accounting by lessee

3.Account treatment

1. Objective of IFRS 16 leases

International Financial Reporting Standard 16 Leases (IFRS 16) sets out the principles for the recognition,

measurement, presentation and disclosure of leases. The objective is to ensure that lessees and lessors provide

relevant information in a manner that faithfully represents those transactions.

2. Accounting by lessee

2.1 Identifying a lease

⚫ A contract is, or contains, a lease if the contract conveys the right to control the use of an identified asset for

a period of time in exchange for consideration. The contract may contain other elements which are not lease,

such as a service contract. These other components must be separated out from the lease and separately

accounted for, allocating the consideration on the basis of the stand-alone prices of the lease and non-lease

components.

The right to control the use of an identified asset depends on the lessee having:

⚫ The right to obtain substantially all of the economic benefits from use of the identified asset

⚫ The right to direct the use of the identified asset. This arises if either:

The customer has the right to direct how and for what purpose the asset is used during the whole of it

period of use, or

The relevant decisions about use are pre-determined and the customer can operate the asset without

the supplier having the right to change those operating instructions.

⚫ A lessee does not control the use of an identified asset if the lessor can substitute the underlying asset for

another asset during the lease term and whole benefit economically from doing so.

2.2 Recognition

⚫ At the commencement date, a lessee shall recognise a right-of-use asset and a lease liability.

⚫ Recognition exemptions

Instead of applying the recognition requirements of IFRS 16 describe below, a lease may elect to account for

lease payments as an expenses on a straight-line basis over the lease term or another systematic basis for

the following two types of leases.

Short-term leases. There are leases with a lease term of twelve months or less. This election is made by

class of underlying asset. A lease that contains a purchase option cannot be a short-term lease.

67

ACCA- FR 2021-2022 Part B-Chapter 7

Low value leases. These are lease where the underlying asset has a low value when new. This election

can be made on a lease-by-lease basis. An underlying asset qualifies as low value only if two conditions

apply:

The lease can benefit from using the underlying asset.

The underlying asset is not highly dependent on, or highly interrelated with, other assets.

2.3 Measurement

2.3.1 Initial measurement

⚫ Initial measurement of the right-of-use asset

At the commencement date, a lessee shall measure the right-of-use asset at cost. This comprises:

The amount of the initial measurement of the lease liability.

Any lease payments made at or before the commencement date, less any lease incentives received.

Any initial direct costs incurred by the lessee.

Any cost which the lessee will incur for dismantling and removing the underlying asset. (the

obligations for such costs are recognised and measured applying IAS 37 provision, contingent

liabilities and contingent assets)

⚫ Initial measurement of the lease liability

At the commencement date, a lessee shall measure the lease liability at the present value of the

lease payments, including any expected payments at end of the lease, discounted using the interest

rate implicit in the lease, If that rate cannot be readily determined, the lessee shall use the lessee’s

incremental borrowing rate.

2.3.2 Subsequent measurement

⚫ Subsequent measurement of the right-of-use asset

After the commencement date, a lessee shall measure the right-of-use asset applying a cost model

(IAS 16), unless it is an investment property or belongs to a class of assets to which the revaluation

model applies.

If the lease transfers ownership of the

underlying asset at end of the lease term or if

the cost of reflects a purchase option which the

lessee is expected to exercise, the right-of-use

asset should be depreciated over the useful life

of the underlying asset.

If there is no transfer of ownership and no

purchase option, the right-of-use asset should

be depreciated from the commencement date

to the earlier of the end of the useful life and

the end of the lease term.

⚫ Subsequent measurement of the lease liability

After the commencement date the carrying amount of the lease liability is increased by interest

charges on the outstanding liability and reduced by lease payments made.

68

ACCA- FR 2021-2022 Part B-Chapter 7

Payment in arrears

Year Balance b/d Interest subtotal Payment Balance c/d

1 X X X (X) X

2 X X X (X) X

(NCL)

Payments in advance

Year Balance b/d Payment subtotal Interest Balance c/d

1 X (X) X X X

2 X (X) X X X

(NCL)

2.4 Presentation

A lessee shall either present in the statement of financial position, or disclose in the notes.

⚫ Right-of-use assets can be presented on a separate line under non-currents or they can be included in the

total of corresponding underlying assets and disclosed in the note.

⚫ Lease liabilities should be either presented separately from other liabilities or disclosed in the notes.

2.5 Disclosure

A lessee shall disclose the following amounts for the reporting period:

⚫ Depreciation charge for right-of-use assets.

⚫ Interest expense on lease liabilities.

⚫ Expense relating to short-term and low-value leases.

⚫ Additions to right-of-use assets.

⚫ The carrying amount of right-of-use assets at the end of the reporting period by class of underlying asset.

3. Accounting treatment

3.1 Sale and leaseback

3.1.1 Transfer of the asset is a sale

⚫ If the transfer of an asset by the seller (lessee) satisfies the requirements of IFRS 15 to be accounted

for as a sale of the asset:

The seller(lessee) shall measure the right-of-use asset arising from the leaseback at the proportion

of the previous carrying amount of the asset that relates to the right of use retained by the seller

(lessee).

The seller (lessee) only recognise only the amount of any gain or loss that relates to the rights

transferred to the buyer (lessor).

⚫ If the fair value of the consideration for the sale of an asset does not equal the fair value of the asset,

or if the payments for the lease are not at market rates, the following adjustments should be made:

Any below-market terms shall be accounted for as a prepayment of lease payments. (the shortfall

in consideration received from the lessor is treated as a lease payment made by lessee)

69

ACCA- FR 2021-2022 Part B-Chapter 7

Any above-market terms shall be accounted for as additional financing provided by the buyer (lessor)

to the seller(lessee)(the additional amount paid by the lessor is treated as additional liability, not as

gain on the sales)

3.1.2 Transfer of the asset is not a sale

⚫ If the transfer does not satisfy the requirements of IFRS 15 to be accounted for as a sale of the asset,

the seller (lessee) shall continue to recognise the transferred asset and shall recognise a financial

liability equal to the transfer proceeds. It shall account for the financial liability applying IFRS 9. The

transaction is more in the nature of a secured loan.

Example 1 (Identifying a lease)

ZBG is a lessee that operates in a number of countries. ZBG has:

(a) Approximately 20, 000 leases of vehicles (Le. cars and trucks) throughout the group, with non-cancellable lease

terms of between three and five years.

(b) A relatively small number of property leases (approximately 60) used for corporate purposes, with non-

cancellable lease terms of between 5 and 12 years.

(c) A large number of leases of low value office equipment such as personal computers, desktops, phone and

desktop printers.

Which of the above leases should be applied by the requirements in IFRS 16 Leases?

A (a), (b) and (c)

B Only (a)

C (a) and (b)

D (b) and (c)

Example 2 IFRS 16

Basic has two leases contracts for machinery, all of which commenced on the first day of the year. Contract A requires

the annual installments to be paid in advance on the first day of each year and B requires the annual installments

to be paid in arrears on the last day of each year. The details are as follows:

Contract A

$000

Initial lease liability 1,000

Installments in advance 240

Contract life 5 years

Implicit interest 10%

Contract B

$000

Initial lease liability 1,100

Installments in arrears 290

Contract life 5 years

Implicit interest 10%

Required:

Show the effect of the above transactions on the financial statements.

70

ACCA- FR 2021-2022 Part B-Chapter 7

Example 3 Sales and leaseback

Mini Co entered into a sale and leaseback on 1 Jan 20X9. It sold a machine with carrying amount of $800,000 for

cash $900,000 which is the fair value at that date. Mini Co leased it back over a three-year period which equivalent

to its remaining useful life and Annual payments are $300,000 payable at the end of each year. The rate of interest

implicit in the lease is 4%. The transaction constitutes a sale in accordance with IFRS 15.

Required:

What are the amounts to be recognised at 1 January 20X9 in respect of this transaction?

71

ACCA- FR 2021-2022 Part B-Chapter 8

Chapter 8 Taxation

Syllabus content

1.IAS 12 Income tax

2.Current tax

3.Defer tax

1. IAS 12 Income tax

IAS 12 covers both current and deferred tax.

2. Current tax

2.1 Principles

⚫ Current tax is the amount of income taxes payable (recoverable) in respect of the taxable profit (tax loss) for

a period.

⚫ Normally, current tax is recognized as income or expense and included in the net profit or loss for the period,

except for the tax relates to an item that has been accounted for in equity.

2.2 In the statement of profit or loss the tax charge will be made up of:

Estimated income tax on taxable profits x

Add: under provision for tax charges in the previous year x

Less: overprovision for tax charge in the previous year (x)

x

Dr. P/L-tax expense

Cr. Current tax

3. Deferred tax

3.1 Definition

⚫ Deferred tax: Deferred tax is taxation payable or recoverable in the future based on the “temporary

differences” existing at the reporting date.

⚫ Permanent differences:

One-off differences between accounting and taxable profits caused by certain items not being

taxable/allowable.

Differences which only impact on the tax computation of one period.

Difference which have no deferred tax consequences whatever.

⚫ Examples of permanent difference could be the situation when revenue is never taxable, such as tax-exempt

income or expenses is not allowable expenditure for tax purpose, such as penalty & fines.

⚫ Temporary differences: differences between the carrying amount of an asset or liability in the statement of

financial position and its tax base (the amount attributed to that asset or liability for tax purposes). These

occur when items of revenue are included in both accounting profits and taxable profits, but not for the same

accounting period.

72

ACCA- FR 2021-2022 Part B-Chapter 8

⚫ Example of temporary differences could be straight-line depreciation for accounting purpose but accelerated

depreciation/tax allowance for tax purpose.

⚫ Temporary differences may be either taxable temporary differences or deductible temporary differences.

⚫ Taxable temporary differences: Taxable temporary differences are temporary differences that will result in

taxable amounts in determining taxable profit (tax loss) of future periods when the carrying amount of the

asset or liability is recovered or settled.

⚫ Deductible temporary difference: Deductible temporary differences are temporary differences that will

result in amounts that are deductible in determining taxable profit (tax loss) of future periods when the

carrying amount of the asset or liability is recovered or settled.

⚫ Deferred tax liabilities: Deferred tax liabilities are the amounts of income taxes payable in future periods in

respect of taxable temporary differences.

⚫ Deferred tax assets: Deferred tax assets are the amounts of income taxes recoverable in future periods in

respect of:

Deductible temporary differences

The carry forward of unused tax losses

The carry forward of unused tax credits

⚫ Tax base: The tax base of an asset or liability is the amount attributed to that asset or liability for tax purposes.

3.2 Taxable temporary differences

You may have found the definition of taxable temporary differences somewhat confusing. In order to fully

understand the definition, the following is illustration of circumstances that give rise to taxable temporary

differences.

Illustration 1

A non-current asset costing $2,000 was acquired at the start of year 1. It is being depreciated straight line over

four years, resulting in annual depreciation charges of $500. Thus a total of $2,000 of depreciation is being

charged. The capital allowances granted on this asset are:

$

Year 1 800

Year 2 600

Year 3 360

Year 4 240

Tax rate 25%

The differences between accounting and tax depreciation on the equipment will be:

Y1 Y2 Y3 Y4

$ $ $ $

Accounting depreciation 500 500 500 500

Tax depreciation(Capital allowance) 800 600 360 240

Taxable difference 300 100 (140) (260)

Cumulative difference 300 400 260 0

73

ACCA- FR 2021-2022 Part B-Chapter 8

This will give the following differences between the carrying amount and the tax base of the asset at the end of

each year.

Year Carrying value

(Cost - accumulated

depreciation)

Tax base

(Cost - accumulated

capital allowances)

Temporary

difference

$ $ $

1 1,500 1,200 300

2 1,000 600 400

3 500 240 260

4 Nil Nil Nil

Table 1 shows the carrying value of the asset, the tax base of the asset and therefore the taxable temporary

difference* at the end of each year.

* In the above illustration, when the capital allowances are greater than the depreciation expense in years 1

and 2, the entity has received tax relief early. This is good for cash flow in that it delays (ie defers) the payment

of tax. However, the difference is only a temporary difference and so the tax will have to be paid in the future.

In years 3 and 4, when the capital allowances for the year are less than the depreciation charged, the entity is

being charged additional tax and the temporary difference is reversing. Hence the temporary differences can

be said to be taxable temporary differences.

3.3 Deferred tax liabilities

The deferred tax liabilities (items in SOFP) are calculated based on taxable temporary differences as follows:

Y1 Y2 Y3 Y4

$ $ $ $

Carrying amount at Y/E 1,500 1,000 500 0

Tax base at Y/E 1,200 600 240 0

Taxable temporary differences 300 400 260 0

Deferred tax liabilities at Y/E @25% 75 100 65 0

The movements in the liability are recorded in the statement of profit or loss as part of the tax expense (items

in SPL/OCI).

Year 1 2 3 4

$ $ $ $

Opening deferred tax liability 0 75 100 65

Increase/(decrease) in the year(tax expense) 75 25 (35) (65)

Closing deferred tax liability 75 100 65 0

3.4 Revaluations of non-current assets

⚫ Revaluations of non-current assets (NCA) are a further example of a taxable temporary difference. When an

NCA is revalued to its current value within the financial statements, the revaluation surplus is recorded in

equity (in a revaluation reserve) and reported as other comprehensive income. While the carrying value of

the asset has increased, the tax base of the asset remains the same and so a temporary difference arises.

74

ACCA- FR 2021-2022 Part B-Chapter 8

⚫ Tax will become payable on the surplus when the asset is sold and so the temporary difference is taxable.

Since the revaluation surplus has been recognised as other comprehensive income, to comply with matching,

the tax charge on the surplus is also charged to other comprehensive income. Suppose that in Illustration 1,

the asset is revalued to $2,500 at the end of year 1.

Revalued asset at the end of year 1 (illustration 1)

Year2 Carrying value

(Cost - accumulated

depreciation)

Tax base

(Cost - accumulated

capital allowances)

Temporary

difference

$ $ $

Opening balance 2,000 2,000

Depreciation charge

/capital allowance this year

(500) (800)

Balance before revaluation 1,500 1,200 300

Revaluation 1,000 - 1,000

Balance after revaluation

(closing balance)

2,500 1,200

(remain the same)

1,300

The double entry is:

Dr P/L-Tax expense $ 300*25% $75

Dr Revaluation reserve $1,000*25% $250

Cr Deferred tax liability $1,300*25% $325

3.5 Deductible temporary differences

Again, in order to fully understand the definition, let’s revise the capital allowance of illustration 1 of

circumstances that give rise to deductible temporary differences.

Revised capital allowance

$

Year 1 300

Year 2 400

Year 3 600

Year 4 700

The differences between accounting and tax depreciation on the equipment will be:

Y1 Y2 Y3 Y4

$ $ $ $

Accounting depreciation 500 500 500 500

Tax depreciation(Capital allowance) 300 400 600 700

Deductible difference (200) (100) 100 200

Cumulative difference (200) (300) (200) 0

75

ACCA- FR 2021-2022 Part B-Chapter 8

This will give the following differences between the carrying amount and the tax base of the asset at the end of

each year.

Year Carrying value

(Cost - accumulated

depreciation)

Tax base

(Cost - accumulated

capital allowances)

Temporary

difference

$ $ $

1 1,500 1,700 (200)

2 1,000 1,300 (300)

3 500 700 (200)

4 Nil Nil Nil

Above table shows the carrying value of the asset and the tax base of the asset therefore the deductible

temporary difference* occurs at the end of each year.

* In the revised illustration 1, when the capital allowances are lower than the depreciation expense in years 1

and 2, the entity has to pay tax early. This is bad for cash flow. However, the difference is only a temporary

difference and so the tax will have to be relieved in the future. In years 3 and 4, when the capital allowances for

the year are greater than the depreciation charged, the entity is being relieved additional tax and the temporary

difference is reversing. Hence the temporary differences can be said to be deductible temporary differences.

3.6 Deferred tax assets

The deferred tax assets (items in SOFP) are calculated based on deductible temporary differences as follows:

Y1 Y2 Y3 Y4

$ $ $ $

Carrying amount at Y/E 1,500 1,000 500 0

Tax base at Y/E 1,700 1,300 700 0

Deductible temporary differences 200 300 200 0

Deferred tax asset at Y/E @25% 50 75 50 0

The movements in the liability are recorded in the statement of profit or loss as part of the tax expense (items

in SPL/OCI).

Year 1 2 3 4

$ $ $ $

Opening deferred tax asset 0 50 75 50

Increase/(decrease) in the year(tax expense) 50 25 (25) (50)

Closing deferred tax asset 50 75 50 0

3.7 Summary

Assets and net assets

CA>TB Taxable temporary difference Deferred tax liability

CA<TB Deductible temporary difference Deferred tax asset

76

ACCA- FR 2021-2022 Part B-Chapter 8

Liabilities (which is contrast with assets)

CA>TB Deductible temporary difference Deferred tax asset

CA<TB Taxable temporary difference Deferred tax liability

A deferred tax asset is recognised for an unused tax loss carry forward or unused tax credit if, and only if, it is

considered probable that there will be sufficient future taxable profit against which the loss or credit carry

forward can be utilised.

Example 1 IAS 12

Peter Ltd was incorporated on 1 April 20X1. In the year ended 31 March 20X2 the company make a profit before tax

of $100,000 (after charging depreciation of $10,000) and made the following capital additions:

Plant $48,000

Motor vehicles $12,000

Income tax is chargeable at the rate of 30%, and tax allowance are available at 25%.

Required:

a ) The current tax payable for the year ended 31 March 20X2.

b ) The deferred tax charge for the year on the basis that full provision is made for all temporary differences,

showing also the relevant extracts from the financial statement.

Example 2 IAS 12

A non-current asset cost $100,000 when purchased and depreciation totalling $40,000 has been charged up to the

balance sheet date - 31 March 20X1. The company has claimed total tax allowances on the asset of $50,000. On 31

March 20X1 the asset is revalued to $90,000. Assume tax rate is 30%. Required:

Explain the deferred tax implications.

Example 3 IAS 12

There is an incorporated listed company, the following is an extract of the trail balance at 31 March 20X5:

$000 $000

Deferred income tax (note 1) 5,000

Tax payable (note 2) 8,000

The following information is relevant

(1)Deferred tax is calculated on all temporary differences under the comprehensive (full provision) method. The tax

rate is 25%.

(2)The provision for current income tax for the year to 31 March 20X6 is estimated at $10m and the tax payment in

current year is $7m.

Temporary difference at 31 March 20X6 was:

$000

Accelerated tax depreciation 18,400

Other temporary difference (4,400)

Required:

Prepare the extract of the statement of profit or loss and other comprehensive income for the year to 31 March

20X6, and the statement of financial position as at 31 March 20X6 ( Note: Cash is NOT required to present in the

extract of financial statements)

77

ACCA- FR 2021-2022 Part B-Chapter 9

Chapter 9 Foreign Currency Transactions and Entities

Content

1.Why adjustments for foreign currency transactions are necessary

2.Functional and presentation currency

3.Accounting for individual transactions designated in a foreign currency

SESSION CONTENT

Functional and

presentation currency

Why necessary

Translation

1. Why adjustments for foreign currency transactions are necessary

If a company trades overseas, it will buy or sell assets in foreign currencies. For example, a Chinese company

might buy materials from Canada, and pay for them in US dollars, and then sell its finished goods in Germany,

receiving payment in Euros, or perhaps in some other currency. If the company owes money in a foreign currency

at the end of the accounting year, or holds assets which were bought in a foreign currency, those liabilities or

assets must be translated into the local currency, in order to be shown in the books of account. 2. Functional and presentation currency

An entity maintains its day-to-day financial records in its functional currency.

⚫ The functional currency is the currency of the primary economic environment where the entity operates. In

most cases this will be the local currency.

An entity should consider the following when determining its functional currency.

The currency that mainly influences sales prices for goods and services .

The currency of the country whose competitive forces and regulations mainly determine the sales price

of goods and services.

The currency that mainly influences labour, material and other costs of providing goods and services.

⚫ The presentation currency is the currency in which the entity presents its financial statements. This can be

different from the functional currency.

3. Accounting for individual transactions designated in a foreign currency

Where an entity enters into a transaction denominated in a currency other than its functional currency, that

transaction must be translated into the functional currency before it is recorded.

⚫ The exchange rate used to initially record transactions should be either:

The spot exchange rate on the date the transaction occurred, or

An average rate over a period of time, providing the exchange rate has not fluctuated significantly.

Foreign currency

transactions

78

ACCA- FR 2021-2022 Part B-Chapter 9

Monetary Items

⚫ At subsequent each reporting dates , the following process must be applied :

Foreign currency monetary items (debtors , creditors , cash , loans) must be retranslated using the closing

rate . The closing rate is the exchange rate at the reporting date.

Foreign currency non-monetary items (fixed assets , investments , stock) carried at historical cost should

be reported using exchange rate at transaction date, which are not retranslated .

Foreign currency non-monetary items carried at fair value should be reported at rate when fair value is

determined. Foreign currency element is not recognised separately. When gain or loss of fair value is

recognised in P/L, exchange component is also taken to P/L. If gain or loss of fair value is recognised in

other comprehensive income on a non-monetary asset, any exchange gain is also recognised in other

comprehensive income. (eg: IAS16 revaluation model)

Exchange differences on settlement of monetary items or on retranslating monetary items are recognized

in P/L.

Example 1 – Monetary items

On 7 May 20X6 an entity with a functional currency of dollars ($)sold goods to a German entity for €48,000. On this

date, the rate of exchange was $1 = € 3.2.

The sale is translated into the functional currency using the exchange rate in place on the transaction date.

$

Dr Receivables (€48,000/3.2) 15,000

Cr Revenue 15,000

By the reporting date of 31 July 20X6, the invoice had not been settled.

On this date, the rate of exchange was $1 = €3.4.

Units of currency held and assets

and liabilities to be received or paid

in a fixed or determinable number

of units of currency, e.g. cash,

receivables, payables, loans.

Other items in the statement of

financial position, e.g. non-current

assets, inventory, investments.

Retranslate using the closing

rate (year end exchange rate)

Do not retranslate

If an item is carried at fair value, the

fair value should be translated on

the date it was determined.

Non-monetary Items

79

ACCA- FR 2021-2022 Part B-Chapter 9

Example 2 – Non monetary items with cost

Olympic, which has a functional and presentation currency of the dollar ($), accounts for land using the cost model

in lAS 16. On 1 July 20X5, Olympic purchased a plot of land in another country for 1.2 million dinars.

Relevant exchange rates: Dinars to $1

1 July 20X5 4.0

30 June 20X6 3.0

Required:

Explain how to account for the land?

Example 3 – Non monetary items with fair value

Pallot, which has a functional and presentation currency of the dollar ($), accounts for land using the revaluation

model in lAS 16. On 1 July 20X5, Pallot purchased a plot of land in another country for 1.2 million dinars. At 30 June

20X6, the fair value of the plot of land was 1.5 million dinars.

Relevant exchange rates: Dinars to $1

1 July 20X5 4.0

30 June 20X6 3.0

Required:

Explain how to account for the land?

80

ACCA- FR 2021-2022 Part B-Chapter 10

Chapter 10 Events after the Reporting Period, Provision and Contingencies

Syllabus content

1.Events after the reporting period

2.IAS 37 Provision, contingent assets and contingent liabilities

1. IAS 10 Events after the reporting period

The statement of financial position shows the state of affairs at the reporting date but accounts are not finalized

until some time later.

So that users of accounts are not misled about the company's financial position, accounts must reflect

information which comes to light or events which occur in between the year end and the date on which accounts

are finalized.

1.1 Key Definitions

⚫ Events after the reporting period: Events occurring between the end of the reporting period and the date

on which the financial statements are authorised for issue.

⚫ Adjusting events: Events after the reporting period which provide evidence of conditions that existed at the

end of the reporting period.

Examples:

Allowances for inventory losses and doubtful debts.

Amounts received or receivables in respects of insurance claims which were being negotiated at the end

of reporting period.

⚫ Non-adjusting events: events after the reporting period which provide are indicative of conditions that arose

after the reporting period. In other words, they are events.

Events after the Reporting Period

An event which occurs after the year end

but before the FS are approved

Gives new evidence on condition

which did not existed at the year end

Gives new evidence on condition

which existed at the year end

Adjusting events-apply relevant

accounting treatment

But impacts going

concern assumption

Any other, disclose as non-adjusting

events

Nature

Estimate of financial effect

81

ACCA- FR 2021-2022 Part B-Chapter 10

Examples:

Natural disasters such as flood and fire.

The issue of new shares or loan stock.

Purchase, disposal or closure of a business.

1.2 Accounting treatment:

⚫ Adjusting events required the adjustment of amounts.

⚫ Non-adjusting events should be disclosed by note if they are of such importance.

⚫ Going concern – if an event after the reporting dates indicates that the going concern assumption is

inappropriate for the entity, then the SOFP should be prepared on a break-up basis.

⚫ If an entity declares dividends after the reporting period, the entity shall not recognize those dividends as a

liability at the end of the reporting date. That is a non-adjusting event.

Example 1 The following events after the reporting period have occurred. Identify whether they are adjusting or non-adjusting:

Adjusting Non-adjusting

1.Evidence of a permanent diminution in property value

prior to the year end.

2.Uninsured loss of stock in a fire.

3.Sale of inventory after the reporting period for less than its

carrying value at the year end.

4. Amounts received or paid in respect of legal or insurance

claims which were in negotiation at the year end.

5. Litigation commenced after the reporting period. Example 2 IAS 10 Past Exam Jun 2009 Q4 (b) Waxwork’s current year end is 31 March 20X9. Its financial statements were authorized for issue by its directors

on 6 May 20X9 and the AGM (annual general meeting) will be held on 3 June 20X9. The following matters have been

brought to your attention: (i) On 12 April 20X9 a fire completely destroyed the company’s largest warehouse and the inventory it contained.

The carrying amounts of the warehouse and inventory were $10 million and $6 million respectively. It appears that

the company has not updated the value of its insurance cover and only expects to be able to recover a maximum

$9 million from its insurers. Waxwork’s trading operations have been severely disrupted since the fire and it expects

large trading losses from some time to come.

(4 marks)

(ii) A single class of inventory held at another warehouse was valued at its cost of $460,000 at 31 March 20X9. In

April 20X9 70% of this inventory was sold for $280,000 on which Waxworks’ sales staff earned commission of 15%

of the selling price

(3 marks)

Required:

Explain the required treatment of the items(i) and (ii) by Waxwork in its financial statements for the year ended 31

March 20X9.

82

ACCA- FR 2021-2022 Part B-Chapter 10

2. IAS 37 Provision, contingent assets and contingent liabilities

2.1 Provisions and creative accounting

⚫ The objective of the standard is to ensure that:

Provisions are recognized and measured consistently.

Sufficient information is disclosed to enable a user of the accounts to understand the nature, timing and

amount of any provisions included in the accounts.

⚫ A provision is defined as a liability of uncertain timing or amount.

⚫ Prudence suggests that if a future cash outflow is foreseen we should provide for it, i.e. put a liability (a

provision) in the balance sheet and the corresponding debit as a cost in the profit and loss.

⚫ The problem is that prudence not only ensures companies show bad news in the accounts as soon as possible;

it also allows the accounts to be manipulated.

⚫ By careful use of provisions, managers can retain profits from very good years, by making provisions for items

that may or may not be required (e.g. redecorating the head office) and

⚫ Then releasing them in years when the results are poor and the provision turned out not to be needed after

all. This is known as 'smoothing profits'. Crucially it reduces the level of risk that shareholders attribute to

the company and therefore their required return.

IAS 37 attempts to prevent this manipulation and get the accounts to show a true and fair view without 'fudging'

by setting strict criteria for the recognition of a provision. This is why an accounting standard on provisions is

necessary. 2.2 Recognition of a provision

The recognition criteria are derived from the definition of a liability and the recognition criteria laid down in

the Framework. A provision should be recognized when:

⚫ An enterprise has a present obligation (legal or constructive) as a result of a past event;

⚫ It is probable that an outflow of resources embodying economic benefits will be required to settle the

obligation; and

⚫ A reliable estimate can be made of the amount of the obligation.

IAS 37 Provisions and Contingencies

Provisions Contingent liability Contingent asset

Only provide if, Obligation legal or constructive; Probable transfer of economic benefit

resulting from a past event;& Can be reliably measured

Potential liability Assess likelihood of liability Remote-ignore Possible-disclose Probable-disclose/provision

Potential asset

Assess likelihood of asset

Remote-ignore

Possible-ignore

Probable-disclose

83

ACCA- FR 2021-2022 Part B-Chapter 10

If any of these conditions are not met then the enterprise should show a contingent liability unless the

probability of an outflow is remote in which case nothing should be disclosed.

2.2.1 Present obligation

A provision may be necessary as a result of a legal or a constructive obligation.

⚫ A legal obligation is an obligation that derives from:

A contract

Legislation

Other operation of law.

⚫ A constructive obligation is an obligation that derives from an entity's actions where:

By an established pattern of past practice, published policies or a sufficiently specific current

statement, the entity has indicated to other parties that it will accept certain responsibilities, and

As a result, the entity has created a valid expectation on the part of those other parties that it will

discharge those responsibilities.

⚫ Past events

This is an event, which creates a legal or constructive obligation and that results in an entity having

no realistic alternative to settling that obligation.

2.3 Measurement of a provision

⚫ The amount recognized as a provision should be the best estimate of the expenditure required to settle the

present obligation at the reporting date, that is, the amount that an enterprise would rationally pay to settle

the obligation at the reporting date or to transfer it to a third party.

⚫ Provisions for one-off events (restructuring, environmental clean-up, settlement of a lawsuit) are measured

at the most likely amount.

⚫ Provisions for large populations of events (warranties, customer refunds) are measured at a probability-

weighted expected value.

⚫ The estimate should take into account:

Risks associated with the cash flows

Expected future events (e.g. new technology or new legislation)

The time value of money, if it has a material effect

· If the time value of money is material then the provision should be discounted.

· The amortisation of the discount is a finance cost, and it should be disclosed separately on the face

of the income statement.

2.3.1 Changes in provisions

⚫ Review and adjust provisions at the end of each reporting period.

⚫ If outflow no longer probable, reverse the provision to income.

2.3.2 Reimbursement

If some or all of the expenditure required to settle a provision is to be reimbursed by a third party, this

⚫ should only be recognized when it is virtually certain to be received, and

⚫ should be treated in SFP as a separate asset rather than a reduction in the provision. (a separate liability

and a separate asset)

⚫ In I/S, however, the net amount may be shown.

84

ACCA- FR 2021-2022 Part B-Chapter 10

2.4 Application of Provisions

2.4.1 Environmental provisions

⚫ Recognising an asset when recognising a provision (environmental provision)

Normally the setting up of a provision should be charged to the income statement immediately.

However, in the unusual case where the incurring of the present obligation give rise to future

economic benefits, then the provision for the obligation and the asset for the future benefits should

be recognized.

These are often referred to as clean-up costs because they usually relate to the cost of

decontaminating and restoring an industrial site when production has ceased.

⚫ Recognition

Merely, causing damage or intending to clean-up a site will not create an obligation.

Generally, a provision will only be recognized if there is a legal obligation to repair environmental

damage.

However, some entities publish specific corporate guidelines that include environment awareness.

If these guidelines create a constructive obligation to clean-up then a provision will be required.

⚫ Measurement

The full cost of an environmental provision should be recognized as soon as the obligation arises,

while the debit side is to the cost of related asset and depreciated over expected useful life of the

asset under IAS 16.

Because it may be many years before the costs relating to the provision are incurred,then provision

is normally recognized at its present value.

As the day of payment draws nearer, the discount unwinds and the provision increases.

The increase in the provision will be charged to the income statement as a finance cost.

Example 1 IAS 37

On 1 October 20X7, Promoil acquired a newly constructed oil platform at a cost $30 million together with

the right extract oil from an offshore oilfield under a government licence. The terms of the licence are that

Promoil will have to remove the platform (which will then have no value) and restore the sea bed to an

environmentally satisfactory condition in 10 years’ time when the oil reserves have been exhausted. The

estimated cost of this on 30 September 20Y7 will be $15 million. The present value of $1 receivable in 10

years at the appropriate discount rate for Promoil of 8% is $0.46.

Required:

(i) Explain and quantify how the oil platform should be treated in the financial statement of Promoil for

the year ended 30 September 20X8; (7 marks)

(ii) Describe how your answer to (i) would change if the government licence did not required an

environmental clean up. (3 marks)

(10 marks)

85

ACCA- FR 2021-2022 Part B-Chapter 10

2.4.2 Warranties

⚫ A warranty is often given in manufacturing and retailing business.

⚫ Present obligation: obligation to make good or replace faulty products.

⚫ Past event:

A warranty provision is required at the time of the sale rather than the time of the

repair/replacement as the making of the sale is the past event which gives rise to an obligation.

An outflow of resources embodying economic benefits in settlement--it may well be probable that

some outflow of resources will be needed to settle the class of obligations as a whole.

⚫ Reliable estimate:

It is measured at a probability-weighted expected value. This requires the seller to analyze past

experience so as to be able to estimate:

How many claims will be made.

How much each repair will cost.

⚫ The provision set up at the time of sale (obligation arises):

Is the number of repairs expected in the future times at the expected cost of each repair.

Should be reviewed at the end of each accounting period in the light of further experience.

Illustration

A company offers a warranty on the sale of all its goods for one year after the date of sale.

The probability of a warranty claim on an individual item is estimated at 15%.

If a warranty claim is made then on average the repair or replacement costs 40% of the original sales

value. Sales for the year ended 31 December 20X8 totaled $2 million.

Warranty claims in respect of those sales prior to 31 December 20X8 totaled $40,000.

Solution:

The total expected warranty claims in respect of 20X8 sales are $120,000 ($2 million 15% 40%).

Claims of $40,000 have been made before the year end.

Therefore a provision is required for expected future claims of $80,000 ($120,000 - $40,000).

2.4.3 Guarantees

⚫ In some instances (particularly in groups) one company will make a guarantee to another to pay off a

loan if the other company is unable to do so.

⚫ This guarantee should be provided for if it is probable that the payment will have to be made.

⚫ It may otherwise require disclosure as a contingent liability.

Illustration

During 20X5, X guarantees borrowings of Y. At the time of the guarantee Y's financial position was

good.

How should this be treated in the financial statements?

Solution:

1 Present obligation? Yes. The guarantee has given rise to a legal obligation.

2 Transfer of benefits probable? No.

3 Reliable estimate? Yes.

Conclusion - Do not recognize a provision as not all conditions are met.

86

ACCA- FR 2021-2022 Part B-Chapter 10

2.4.4 Future Operating losses

⚫ No provision may be made for future operating losses or repairs because they arise in the future and

can be avoided and therefore no obligation exists.

2.4.5 Onerous contracts

⚫ An onerous contract is a contract in which the unavoidable costs of meeting the obligations under the

contract exceed the economic benefits expected to be received under it.

⚫ The least net cost of an onerous contract should be recognized as a provision.

⚫ The least net cost is the lower of

The cost of fulfilling the contract.

The cost of terminating it and suffering any penalty payments.

Illustration

A company has ten years left to run on the lease of a property that is currently unoccupied. The present

value of the future rentals at the reporting date is $50,000. Subletting possibilities are limited but the

directors feel that likely future subletting rentals could have a present value of $10,000. A provision of

$50,000 would be required in respect of this onerous contract.

2.4.6 Future repairs and refurbishment

⚫ Some fixed assets need substantial expenditure every few years for major refits or refurbishment and

the replacement of major components. Before IAS 37 was issued. it was usual for entities to set up

provisions for the future expenditure. This was justified by the prudence concept: provision must be

made for all known liabilities at the balance sheet date. However, such provisions are no longer allowed

by IAS 37 because they are avoidable and do not represent an obligation to the entity.

⚫ Instead as required by IAS 16, the entity should capitalize the cost of the expenditure when it is incurred

and depreciate it over the useful life. This matches the expenditure with the accounting periods in

which it is effectively incurred.

Illustration: Rockbuster

Rockbuster has recently purchased an item of earth moving plant at a total cost of $24 million. The plant

has an estimated life of 10 years with no residual value, however its engine will need replacing after every

5,000 hours of use at an estimated cost of $7·5 million. The directors of Rockbuster intend to depreciate

the plant at $2·4 million ($24 million/10 years) per annum and make a provision of $1,500 ($7·5

million/5,000 hours) per hour of use for the replacement of the engine.

Required:

Explain how the plant should be treated in accordance with IFRS Standards and comment on the Directors’

proposed treatment. (5 marks)

Answer:

The Directors’ proposed treatment is incorrect. The replacement of the engine is an example of what has

been described as cyclic repairs or replacement. Whilst it may seem logical and prudent to accrue for the

cost of a replacement engine as the old one is being worn out, such practice leads to double counting.

Under the Directors’ proposals the cost of the engine is being depreciated as part of the cost of the asset,

albeit over an incorrect time period. The solution to this problem lies in IAS 16 ‘Property, Plant and

87

ACCA- FR 2021-2022 Part B-Chapter 10

Equipment’. The plant constitutes a ‘complex’ asset i.e. one that may be thought of as having separate

components within a single asset. Thus part of the plant $16·5 million (total cost of $24 million less $7·5

assumed cost of the engine) should be depreciated at $1·65 million per annum over a 10-year life and the

engine should be depreciated at $1,500 per hour of use (assuming machine hour depreciation is the most

appropriate method). If a further provision of $1,500 per machine hour is made, there would be a double

charge against profit for the cost of the engine. IAS 37 also refers to this type of provision and says that

the future replacement of the engine is not a liability. The reasoning is that the replacement could be

avoided if, for example, the company chose to sell the asset before replacement was due. If an item does

not meet the definition of a liability it cannot be provided for.

2.4.7 Restructurings (Optional)

2.4.7.1 Examples of events that may fall under the definition of restructuring:

⚫ The sale or termination of a line of business.

⚫ The closure of business locations in a country or region or the relocation of business activities from

one country region to another.

⚫ Changes in management structure, for example, eliminating a layer of management.

⚫ Fundamental reorganisations that have a material effect on the nature and focus of the entity’s

operations.

2.4.7.2 A provision may only be made if:

⚫ A detail formal and approved plan exists, and

⚫ The plan has been announced to those affected.

2.4.7.3 Restructuring provisions should include only direct expenditures caused by the restructuring, not costs

that associated with the ongoing activities of the enterprise.

Redundancies

Write-down on PPE

⚫ Future costs associated with ongoing activities must not be provided for.

Retraining or relocating staff

Marketing

Investment in new systems and distribution networks

2.5 Accounting for contingent assets and liabilities

2.5.1 A contingent asset is:

⚫ A possible asset that arises from past events and whose existence will be confirmed only by the

occurrence of one or more, uncertain future events not wholly within the entity's control. A contingent

asset only recognized if its receipt is virtually certain.

⚫ Contingent assets should not be recognized - but should be disclosed where an inflow of economic

benefits is probable. When the realisation of income is virtually certain, then the related asset is not a

contingent asset and its recognition is appropriate.

88

ACCA- FR 2021-2022 Part B-Chapter 10

2.5.2 A contingent liability is:

⚫ A possible obligation that arises from past events and whose existence will be confirmed only by the

occurrence of one or more uncertain future events not wholly within the entity's control; or

⚫ A present obligation that arises from past events but is not recognized because:

It is not probable that a transfer of economic benefits will required to settle the obligation.

The amount of the obligation cannot be measured with sufficient reliability.

⚫ Enterprises should not recognize contingent liabilities - unless the possibility of an outflow of resources

embodying economic benefits is remote. The required disclosures are:

Brief description of the nature of the contingent liability

An estimate of its financial effect

An indication of the uncertainties relating to the amount or timing of any outflow; and

The possibility of any reimbursement. 2.6 Summary of recognition criteria by probability:

Probability Contingent Asset Contingent Liability

Virtually certain Asset is SOFP Liabilities in SOFP

Probable Disclose in note Provision in SOFP

Possible Nothing Disclose in note

Remote Nothing Nothing

Example 2 IAS 37

Which of following should be recognized provision for the year end 31 December 1999?

(i) Under new legislation, an entity is required to fit smoke filters to its factories by 30 June 2000. The entity has

not fitted the smoke filters.

(ii) On 12 December 1999 the board of an entity decided to close down a factory making a particular product.

(iii) A company owns buildings that will require repair expenditure of approximately $6 million for next two years.

(iv) Company has a policy of self insuring the risk of fire damage to its buildings. As a result it saves an annual

premium of $10,000 per year. The company wishes to annually provide for this sum so that the cost of any fire

damage as and when incurred can be written off to the provision.

A None

B (i),(ii),(iii) and (iv)

C (iv)

D (iii) and (iv)

Example 3 IAS 37

An oil company has erected an oil rig in the North Sea on 1 January 20X5. It is estimated that it will cost $200 million

to dismantle in twenty years time. The installation costs are $50 million and the cost of construction $400 million.

Suppose the discount rate is 10%.

Required:

Should a provision be made for the environmental costs for year ended 31 December 20X5, and if so how should

they be accounted for?

89

ACCA- FR 2021-2022 Part B-Chapter 11

Chapter 11 Earning per Share

Content

1.Introduction

2.Basic EPS

3.Diluted earnings per share

4.The importance of EPS & DEPS

5.Limitations of EPS as a performance indicator

SESSION CONTENT

Uses &

Diluted EPS limitations

Options/ Convertible

warrants loan note

Rights Full market Bonus

issue value issue issue 1. Introduction

The basic EPS calculation is simply Earnings

Share

⚫ Here ‘earnings’ means group profit after tax, less non-controlling interests and irredeemable preference

dividends; and

⚫ 'Shares' means the weighted average number of outstanding during the period.

2. Basic EPS

EPS =Net profit for year after irredeemable preference dividend but before ordinary dividend

Weighted average number of ordinary shares in issue

Example 1 – GERARD (1)

Draft statement of profit or loss for the year ended 31 December 20X4

$000 $000

Profit before tax 5,060

Taxation (2,300)

Profit after tax 2,760

Dividends information:

Paid- preference interim dividend 276

Paid -ordinary interim dividend 828

Declared – preference final dividend 276

EPS

90

ACCA- FR 2021-2022 Part B-Chapter 11

On 1 January 20X4 the issued share capital of Gerard was 9,200,000 6% preference shares of $1 each and

8,280,000 ordinary shares of $1 each.

Required:

Calculate the earnings per share (EPS) in respect of the year ended 31 December 20X4 on the basis that there

was no change in the issued share capital of the company during the year ended 31 December 20X4.

⚫ Different calculations are used for the three different types of share issues:

Issue at full market price

Bonus issue

Right issue

2.1 Issue of shares at market price:

⚫ Shares issued at full price generate additional cash for the company, which can be invested to earn additional

profits in the future.

⚫ The number of shares used is the weighted average number of ordinary shares in issue—time apportionment.

Example 2 – GERARD (2)

Gerard issued 200,000 shares at full market price ($3) on 1 Jul 20X8.

Relevant information:

20X8 20X7

Ordinary profit attributable to the ordinary shareholders

for the year ending 31 Dec $550,000 $460,000

No. of ordinary shares in issue at 31 Dec 1,000,000 800,000

Required:

Calculate the earnings per share (EPS) in respect of the year ended 31 December 20X8.

2.2 Bonus issues:

⚫ A bonus (or capitalization) issue takes place when a company issues shares, for no consideration, to the

existing shareholders in proportion to their existing holdings.

⚫ No new funds come into the business.

⚫ In the current year, the bonus shares are deemed to have been issued at the start of the year.

⚫ Comparative figures are restated to allow for the proportional increase in share capital caused by the bonus

issue. Doing this treats the bonus issue as if it had always been in existence.

⚫ Restatement of EPS calculations:

Where a company issues shares without a corresponding change in resources (a bonus issue or a right

issue), then the previous EPS figures need to be restated to reflect the bonus element in the issue.

If no restatement is made then EPS computations in successive years will not be comparable.

91

ACCA- FR 2021-2022 Part B-Chapter 11

Example 3 – GERARD (3)

Gerard made a bonus issue of one new share for every five existing shares held on 1 Jul 20 x 8.

Relevant information:

20 x 8 20 x 7

Ordinary profit attributable to the ordinary shareholders

for the year ending 31 Dec $550,000 $460,000

No. of ordinary shares in issue at 31 Dec 1,200,000 1,000,000

Required:

Calculate the earnings per share (EPS) in respect of the year ended 31 December 20X8.

2.3 Rights issues

⚫ A rights issue is an issue of shares to the existing shareholders in proportion to their existing holdings at a

price that is less than the full current market price.

⚫ A right issue combines the characteristics of an issue at full market price with a bonus issue.

⚫ The newly issued shares should be split into:

The bonus element for the full year ⎯ no time apportionment plus

The full price element ⎯ time apportionment

⚫ The comparative EPS will be restated* (see above ‘* Restatement of EPS calculations’)

Format of calculation

Step 1: Calculate theoretical ex-rights price (TERP) = (shares before rights x MV + rights x price)

Shares after rights

Step 2: Calculate bonus fraction = actual cum rights share price (MV)

TERP

Where the actual cum rights price is the share price just before the rights issue and the theoretical ex rights

price is the expected share price just after the rights issue.

Step 3: Calculate current year weighted average number of shares

Add together:

Before right issue = No. of shares before rights issue x time apportion x bonus fraction

After right issue = No. of shares after rights issue x time apportion

Step 4:Basic EPS

Step 5: Adjust comparative EPS

EPS x TERP

Actual cum rights price

92

ACCA- FR 2021-2022 Part B-Chapter 11

Example 4 – GERARD (4)

Gerard issued one new share for every two existing shares held by way of rights at $1.5 per share on 1 Jul 20×8.

The pre-issue market price was $3 per share.

Relevant information:

20X8 20X7

Ordinary profit attributable to the ordinary shareholders

for the year ending 31 Dec $550,000 $460,000

No. of ordinary shares in issue at 31 Dec 1,200,000 800,000

Required:

Calculate the earnings per share (EPS) in respect of the year ended 31 December 20X8.

3. Diluted earnings per share

⚫ Diluted EPS (FDEPS) attempts to alert shareholders to the potential impact on EPS.

⚫ Financial instruments such as convertible loan stock, Share options and share warrants may affect investors’

economic decisions when they are eventually issued.

⚫ Assumptions--The worst possible future dilution has already happened on the first day of the accounting

period, or on the date of issue, if later.

⚫ No restatement of prior year diluted EPS is required.

Where potential ordinary shares are actually issued the basic EPS will be affected due to two factors:

⚫ The number of shares in issue will increase.

⚫ The profits available to the shareholders may increase - for example due to the saving of interest on

convertible debt instruments.

Therefore the standard requires - as a historical performance measure - the disclosure of the diluted EPS on the

assumption that the potential ordinary shares were issued on the first day of the accounting period or, if later,

on the day the relevant potential ordinary shares came into existence.

3.1 Convertible Bonds

Normally, most investors opt for conversion rather than repayment.

This means that existing ordinary shareholders must be informed of the potential effects on EPS of conversion.

The EPS will be affected in two ways:

⚫ Earnings will be increased, because the finance costs will cease on conversion.(post-tax interest saved)

⚫ The number of equity shares would increase.

Example 5 – GERARD (5)

On 1 Apr 20×1, Gerard issued $1,250,000 8% convertible unsecured bonds for cash at par. Each $100 nominal

of the bonds will be convertible in 20×6/20×9 into the no. of ordinary shares set out below:

On 31 Dec 20×6 124 shares

On 31 Dec 20×7 120 shares

On 31 Dec 20×8 115 shares

On 31 Dec 20×9 110 shares

93

ACCA- FR 2021-2022 Part B-Chapter 11

Relevant information:

The company has in issue four million 25 cent ordinary shares.

The income tax rate is 25%.

Trading results for the year end 31 Dec.

20×2

$

Profit before interest and tax 1,050,000

Interest on 8% convertible unsecured bonds (100,000)

Profit before tax 950,000

Income tax (237,500)

712,500

Required:

Calculate basic EPS and diluted EPS in 20×2.

3.2 Share options and warrants

Where a company has potential ordinary shares in issue then the diluted EPS needs to consider the effect on

earnings and on the number of shares issued if the potential ordinary shares are converted into ordinary shares.

In the case of share options and warrants it is not immediately obvious what the effect on future earnings is

likely to be. Therefore the standard requires that the assumed proceeds from the issue of shares under the

option or warrant should be considered as comprising:

⚫ The issue of a number of ordinary shares at fair value - having no dilutive effect.

⚫ The issue of a number of ordinary shares for no consideration with a consequential dilutive impact on the

number of shares in issue.

Summary format

Earnings + notional extra earnings

Number of shares + notional extra shares

Example 6 – Gerard(6)

During 20×3 Gerard has 200,000 shares under option. The options can be exercised for $18 per share,

compared with an average FV during 20×3 of $25. the company had 700,000 shares already in issue, and made

a net profit of $1.9m

Required:

Calculate basic EPS and diluted EPS in 20X3.

3.3 Presentation

⚫ An entity should present basic and diluted EPS on the face of I/S (and also basic and diluted EPS from

continuing operations, if relevant) for each class of ordinary share.

⚫ An entity that reports a discontinued operation must disclose the basic and diluted EPS for the operation,

either on the face of I/S or in the notes.

⚫ IAS 33 also requires basic and diluted losses per share to be disclosed.

94

ACCA- FR 2021-2022 Part B-Chapter 11

4. The importance of EPS & DEPS

The importance of EPS:

⚫ The EPS figure is used to compute the major stock market indicator of performance, the price earnings ratio

(P/E ratio). The calculation is as follows:

P/E ratio =Market value of share

EPS

⚫ Although EPS is based on profit on ordinary activities after taxation, the trend in EPS may be a more accurate

performance indicator than the trend in profit.

⚫ EPS measures performance from the perspective of investors and potential investors and shows the amount

of earnings available to each ordinary shareholder, so that it indicates the potential return on individual

investments.

The importance of DEPS:

⚫ It shows what the current year's EPS would be if all the dilutive potential ordinary shares in issue had been

converted.

⚫ It can be used to assess trends in past performance.

⚫ In theory, it serves as a warning to equity shareholders that the return on their investment may fall in future

periods.

5. Limitations of EPS as a performance indicator

Although EPS is believed to have a real influence on the market price of shares, it has several important

limitations as a performance measure.

⚫ It does not take account of inflation. Apparent growth in, earnings may not be real.

⚫ It is based on historic information and therefore it does not necessarily have predictive value.

⚫ An entity's earnings are affected by the choice of its accounting policies. Therefore it may not always be

appropriate to compare the EPS of different companies.

⚫ DEPS is only an additional measure of past performance despite looking at future potential shares.

Example 7 IAS 33 Past exam Dec 2009 Q5

The profit after for Barstead for the year ended 30 September 20X9 was $15 million. At 1 October 20X8 the

company had in issue 36 million equity shares and a $10 million 8% convertible loan note. The loan note will

mature in 20Y0 and will be redeemed at par or converted to equity shares on basis of 25 shares for each $100

of loan note at the loan-note holder’s option. ON 1 January 20X9 Barsteda made a fully subscribed rights issue

of one new share for every four shares held at a price of $2.8 each. The market price of the equity shares of

Barstead immediately before the issue was $3.8. The earnings per share(EPS) reported for the year ended 30

September 20X8 was 35 Cents. Barstead’s income tax rate is 25%.

Required:

Calculated the (basic) EPS figure for Barstead (including comparatives) and the diluted EPS (comparative not

required) that would be disclosed for the year ended 30 September 20X9. (6 markes)

95

ACCA- FR 2021-2022 Part B-Chapter 12

Chapter 12 Statement of Cash Flows

Content

1.The need for a statement of cash flows

2.Definitions

3.The need for a statement of cash flows

4.Interpretation of cash flow

5.Limitations of the statement of cash flow

Session Content

Format & Contents

Usefulness & Interpretation Notes

1. The need for a statement of cash flows

⚫ IAS 7 was introduced because it was generally felt that the financial statements required an additional

primary statement with the same importance as an income statements and statement of financial position

to emphasize the difference between profit and cash.

⚫ One reason why the additional statement was considered necessary is that final profit figures are relatively

easy to manipulate.

The advantages of cash flows are:

⚫ It is difficult to manipulate cash flows. They are real and possess the qualitative characteristic of objectivity

(as opposed to subjective profits).

⚫ Cash flows are an easy concept for users to understand. Indeed many users misinterpret income statement

items as being cash flows.

⚫ Cash flows help to assess a company’s liquidity, solvency and financial adaptability. Healthy liquidity is vital

to a company’s going concern.

⚫ Many business investment decisions and company valuations are based on projected cash flows.

⚫ The ‘quality’ of a company’s operating profit is said to be confirmed by closely correlated cash flows. Some

analysts take the view that if a company shows a healthy operating profit, but has low or negative operating

cash flows, there is a suspicion of profit manipulation or creative accounting.

2. Definitions

⚫ Cash – Cash on hand (including overdrafts) and on demand deposits.

⚫ Cash equivalents – Short-term, highly liquid investments that are readily convertible into know amounts of

cash and are subject to insignificant risk of changes in value.

Cash Flow

96

ACCA- FR 2021-2022 Part B-Chapter 12

3. Preparation of statement of cash flows

⚫ IAS 7 requires statements of cash flows to report cash flows during the period classified by operating,

investing and financing activities.

⚫ For operating cash flows, IAS 7 offers a choice of method for this part of the statement of cash flows.

3.1 Cash flows under indirect method

This method determines the net cash flow from operating activities by adjusting net profit or loss for the effects

of:

⚫ Non-cash items such as depreciation, provisions, deferred taxes, unrealized foreign currency gains and losses,

undistributed profit of associates, and minority interests;

⚫ Changes during the period in inventories and operating receivables and payables;

⚫ All other items for which the cash effects are investing or financing cash flows;

Pro forma Statement of cash flows for the year ended xx

$ $

Cash flows from operating activities

Profit Before Tax (PBT) X

Add: Amortization/depreciation charge for the year X

Add: Increase in provision X

Add: Loss on sale of non current assets X

Add: interest expense X

Less: Provision released (X)

Less: investment income (X)

Less: profit on the sale of non current assets (X)

Operating profit before working capital changes X

(Increase)/decrease in trade receivables (X)/X

(Increase)/decrease in inventories (X)/X

(Decrease)/increase in trade payables (X)/X

Cash generated from operations X

Interest paid (including interest element of leases) (X)

Income tax paid (X)

Net cash from operating activities X

Cash flows from investing activities

Payments for of non-current assets (X)

Proceeds from sale of non current assets X

Investment income received (interest/dividend received) X X

97

ACCA- FR 2021-2022 Part B-Chapter 12

Cash flows from financing activities

Proceeds from issue of shares X

Proceeds from long-term borrowings X

Payment of capital element of lease (X)

Payment of loan (X)

Dividends paid (X) X

Net increase/decrease in cash and cash equivalents X/(X)

Cash and cash equivalents at beginning of period X/(X)

Cash and cash equivalents at end of period X/(X)

Analysis of cash and cash equivalents

$ $

Cash on hand and balances with banks X X

Short-term investments X X

Cash and cash equivalents X X

Workings:

W1 taxation $

Balance c/f – corporation tax X

- deferred tax X

Less: P/L charge (X)

Less: balance b/f – corporation tax (X)

- deferred tax (X)

Taxation paid (X)

W2 dividend paid

Balance c/f X

Less: charged to retained earnings (X)

Less: balance b/f (X)

Dividend paid (X)

W3 lease

Balance c/f (current and non current liability) X

Less: new lease (X)

Less: balance b/f (current and non current) (X)

Payments for lease (X)

W4 government grant

Balance c/f (current and non-current liability) X

Add: released/amortized to income statement (X)

Less: balance b/f (current and non-current liability) (X)

Receipt of government grants (X)

98

ACCA- FR 2021-2022 Part B-Chapter 12

W5 share capital and share premium

Balance c/f X

Less: conversion of loan stock (X)

Less: balance b/f (X)

Proceeds from issue of share (X)

W6 non-current assets

Balance b/f (NBV) X

Revaluations X

Add: New lease asset X

Less: disposals (NBV) (X)

Less: depreciation/impairment for the year (X)

Balance c/f X

Cash purchase (X)

3.2 Cash flows under direct method (optional)

Direct method: disclose major classes of gross cash receipts and gross cash payments. Formulas used in

calculating cash flows under direct method are as follows:

$

Cash received from customers X

Cash paid to suppliers (X)

Cash paid to or on behalf of employee (X)

Other cash expenses (X)

Cash generated from operations X

4. Interpretation of cash flow

4.1 Cash generation from trading operations

The figure should be compared to the operating profit. The reconciliation note to the cash flow statement is

useful in this regard. Overtrading may be indicated by:

⚫ High profits and low cash generation.

⚫ Large increases in inventory, receivables and payables.

When discussing this area, comments should be made regarding working capital management, giving any

potential reasons for movements I in inventory, receivables and payables, discussing the impact this may have

on cash flow and customer/supplier relations going forwards.

4.2 Dividend and interest payouts

⚫ These can be compared to cash generated from trading operations to see whether the normal operations

can sustain such payments. If cash generated from operations cannot cover these, the business may have

problems continuing as a going concern.

⚫ If the cash generated from operations can cover this, then any cash left over is free cash, and comments

should be made about what the business has done with this (such as buying assets, repaying debt paying a

dividend).

99

ACCA- FR 2021-2022 Part B-Chapter 12

4.3 Investing activities

The nature and scale of a company's investment in non-current assets is clearly shown. A simple test may be to

compare investment and depreciation.

⚫ If investment> depreciation, the company is investing at a greater rate than its current assets are wearing

out - this suggests expansion.

⚫ If investment = depreciation, the company is investing in new assets as existing ones wear out. The company

appears stable.

⚫ If investment < depreciation the non-current asset base of the company is not being maintained. This is

potentially worrying as non-current assets are generators of profit.

4.4 Financing activities

⚫ The current and future implications should be considered when looking at I the financing section. If new

loans have been received, then there will be higher interest going forwards, and regular repayments required.

Conversely, if loans have been repaid, this will help cash flow in future periods.

⚫ If shares have been issued, there is no requirement for this to be repaid, and no interest. However,

shareholders may expect regular dividends which could have to be paid indefinitely.

4.5 Cash flow

⚫ The statement clearly shows the end result in cash terms of the company's operations in the year. Do not

overstate the importance of this figure alone however.

⚫ A decrease in cash in the year may be for very sound reasons (e.g. there was surplus cash last year) or may

be mainly the result of timing (e.g. a new loan was raised just after the end of the accounting period).

⚫ To help in determining the future cash position, other areas of the published accounts should be considered.

4.6 Cash shortfall

⚫ Raising money through a share issue (the company must have a good record of profitability and of dividend

growth, and the share price must be high).

⚫ Tightening credit and stock control, paying payables later.

⚫ Limiting capital expenditure.

⚫ Entering into sale and, leaseback arrangements (it will need some assets which are not already charged).

⚫ Selling some assets (for example investments, or parts of the business which are less related to the main

trade).

⚫ Purchasing a cash-rich company by issuing shares in consideration.

The company should consider most of these measures in preference to:

⚫ Reducing dividends;

⚫ Reducing its level of activity;

100

ACCA- FR 2021-2022 Part B-Chapter 12

5. Limitations of the statement of cash flow

Statement of Cash flows should normally be used in conjunction with income statements and balance sheets

when making an assessment of future cash flows.

⚫ Statement of Cash flows are based on historical information.

⚫ There is some scope for manipulation of cash flows. For example, a business may delay paying payables until

after the year-end.

⚫ Cash flow is necessary for survival in the short term, but in order to survive in the long term a business must

be profitable. It is often necessary to sacrifice cash flow in the short term in order to generate profits in the

long term (e.g. by investment in fixed assets). A huge cash balance is not a sign of good management if the

cash could be invested elsewhere to generate profit.

Example1 – HOLLYMOOD

The financial statements of Hollywood are given below.

Statement of financial position

30 September 20X3 30 September 20X2

$000 $000 $000 $000

Non-current tangible assets 634 510

Current assets:

Inventory 420 460

Trade receivables 390 320

Interest receivable 4 9

Investments 50 0

Cash in bank 75 0

Cash in hand 7 5

946 794

Total assets 1,580 1,304

Capital and reserves:

Ordinary shares $0.50 each 363 300

Share premium account 89 92

Revaluation reserve 50 0

Retained Earnings 63 (70)

562 322

Non-current liabilities:

10% loan notes 0 40

5% loan notes 329 349

329 389

Current liabilities:

Bank overdraft 0 70

Trade payables 550 400

Income tax 100 90

Accruals 36 689 33 593

1,580 1,304

101

ACCA- FR 2021-2022 Part B-Chapter 12

Income statement for the year to 30 September 20X3

$000 $000

Revenue 2,900

Cost of sales (1,734)

Gross profit 1,166

Administrative expenses (342)

Distribution costs (520) (862)

Profit from operations

Income from investments 5

Finance cost (19) (14)

Profit before tax 290

Income tax expense (104)

Net profit for the period 186

Additional information:

1. On 1 October 20X2, Hollywood issued some shares. The proceeds were used to repayment of all its 10% loan

notes and some of its 5% loan notes, both at par.

2. The current asset investment was a 30-day government bond.

3. Non-current tangible assets include certain properties which were revalued in the year.

4. Non-current tangible assets disposed of in the year had carrying value of $75,000; cash received on disposal

was $98,000. Depreciation charge for the year was $87,000.

5. The accruals balance includes interest payable of $33,000 at 30 September 20X2 and $6,000 at 30 September

20X3.

Required:

Prepare the following for Hollywood for the year ended 30 September 20X3, in accordance with IAS 7 Statement of

Cash flows, a cash flow statement, using the indirect method and an analysis of cash and cash equivalents.

Example 2 - Lala

LaLa Co has been trading for a number of years.

The following items were recorded during the year ended 30 September 20X7.

Purchase a new plant by $230.

Interest income received by $20.

Interest paid by $40.

A property revaluation upwards by $150.

An items of building, with a carrying amount of $200, was sold for $250.

What amount would be shown in Lala’s statement of cash flows for net cash used in investing activities for year

end 30 September 20X7?

A $40

B $100

C $560

D $250

102

ACCA- FR 2021-2022 Part B-Chapter 12

Example 3 Presentation of single company financial statement

The following trial balance relates to Quincy as at 30 September 20X4:

(Referred Specimen 2014 Q3)

The following notes are relevant:

(i) On 1 October 20X3, Quincy sold one of its products for $10 million (included in revenue in the trial balance).

As part of the sale agreement, Quincy is committed to the ongoing servicing of this product until 30 September

20X6 (i.e. three years from the date of sale). The value of this service has been included in the selling price of

$10 million. The estimated cost to Quincy of the servicing is $600,000 per annum and Quincy’s normal gross

profit margin on this type of servicing is 25%. Ignore discounting. (ii) Quincy issued a $25 million 6% loan on 1 October 20X3. Issue costs were $1 million and these have been

charged to administrative expenses. Interest is paid annually on 30 September each year. The loan will be

redeemed on 30 September 20X6 at a premium which gives an effective interest rate on the loan of 8%. (iii) Non-current assets:

Quincy had been carrying buildings at depreciated cost, but due to a recent rise in property prices, it decided

to revalue its property on 1 October 20X3 to market value. An independent valuer confirmed the value of the

building at $60 million as at that date and the directors accepted this valuation. The property had a remaining

life of 20 years at the date of its revaluation. Quincy will make a transfer from the revaluation reserve to

retained earnings in respect of the realisation of the revaluation. Ignore deferred tax on the revaluation.

No depreciation has yet been charged on any non-current asset for the year ended 30 September 20X4. All

depreciation is charged to cost of sales.

$’000 $’000

Revenue (note (i)) 213,500

Cost of sales 136,800

Distribution costs 17,500

Administrative expenses (note (ii)) 19,000

Loan note interest paid (note (ii)) 1,500

Investment income 400

Equity shares of 25 cents each 60,000

6% loan note (note (ii)) 25,000

Retained earnings at 1 October 20X3 4,300

Buildings at cost (note (iii)) 50,000

Accumulated depreciation at 1 October 20X3: buildings 8,000

Equity financial asset investments (note (iv)) 17,000

Inventory at 30 September 20X4 74,800

Trade receivables 28,500

Bank 2,900

Current tax (note (v)) 1,100

Deferred tax (note (v)) 1,200

Trade payables 36,700

349,100 349,100

103

ACCA- FR 2021-2022 Part B-Chapter 12

(iv) The investments had a fair value of $15.7 million as at 30 September 20X4. There were no acquisitions or

disposals of these investments during the year ended 30 September 20X4.

(v) The balance on current tax represents the under/over provision of the tax liability for the year ended 30

September 20X3. A provision for income tax for the year ended 30 September 20X4 of $7·4 million is required.

At 30 September 20X4, Quincy had taxable temporary differences of $5 million requiring a provision for

deferred tax. Any deferred tax adjustment should be reported in profit or loss. The income tax rate of Quincy

is 20%.

Required:

(a) Prepare the statement of profit or loss and other comprehensive income for Quincy for the year ended 30

September 20X4.

(b) Prepare the statement of financial position of Quincy as at 30 September 20X4.

104

ACCA- FR 2021-2022 Part C-Chapter 13

Part C CONSOLIDATION ACCOUNTS

Chapter 13 The Consolidation Statement of Financial Position

Content

1.The nature of business combination

2.Accounting issues

3.The single entity concept

4.Basic principle of consolidation financial statement

5.Basic for consolidation statement of financial position

6.Explanations of group account--IFRS 3 business combination

Session content

1. The nature of business combination

1.1 What is a “group”?

Every entity is a separate legal entity. Each one is required to prepare its own financial statements that will

provide useful information for making economic decisions. However, sometimes a number of entities (known

as subsidiaries) will operate under the control of another entity (known as the parent). Together they form a

group, and the group operates as a single economic entity.

1.1.1 Definition

⚫ A parent is an entity that has one or more subsidiaries.

⚫ A subsidiary is an entity that is controlled by another entity (known as the parent).

⚫ Control is

• Power over the investee, where the investor has current ability to direct activities that

significantly affect the investee’s return.

IFRS 3

Business combinations

Exclusions

From

consolidation

Mid-year

acquisitions

CSFP

Dividends

Intra – group

cancellations

Unrealised

Profit in inventories Goodwill

Fair values Deferred

consideration

IFRS10

105

ACCA- FR 2021-2022 Part C-Chapter 13

• Exposure, or rights to ,variable returns from involvement in the investee.

• The ability to use power over the investee to affect the amount of the investor’s returns.

⚫ Acquisition method: All groups are now consolidated using the acquisition method. This “freezes” the

pre-acquisition reserves of subsidiaries. This means that the group’s equity will be less than the sum

of the individual companies’ equity. As a result, groups appear to be more highly geared than their

constituent companies.

2. Accounting issues

⚫ If one company owns more than 50% of the ordinary shares of another company this will usually give the

first company ‘control’ of the second company.

⚫ This is because the first company (P say) has enough voting power to appoint all the directors of the second

company (S say). P is, in effect, able to manage S as if it were merely a department of P, rather than a separate

entity. In strict legal terms P and S remain distinct, but in economic substance they can be regarded as a

single unit (a 'group').

⚫ The key principle underlying group accounts is the need to reflect the economic substance of the relationship.

⚫ To reflect the true economic substance of a group of companies we need to produce group accounts in

addition to the individual accounts prepared for each company within the group. One of the main methods

of doing this is to prepare 'consolidated' accounts and this is the subject of the present chapter.

⚫ Consolidated accounts present the group as though it were a 'single economic entity'.

3. The single entity concept

Business combinations consolidate the results and net assets of group members so as to display the group's

affairs as those of a single economic entity. As already mentioned, this conflicts with the strict legal position that

each. Applying the single entity concept is a good example of the accounting principle of showing economic

substance over legal form.

The group as a single entity

Group

Group

Exemption from consolidation

IFRS10 requires that a parent need not present consolidated financial statements if and only if all of the following

hold.

⚫ The parent is itself a wholly-owned subsidiary or it is a partially owned subsidiary of another entity and its

other owners, including those not otherwise entitled to vote, have been informed about, and do not object

to, the parent not presenting consolidated financial statements.

P

Controls

S

106

ACCA- FR 2021-2022 Part C-Chapter 13

⚫ Its securities are not publicly traded.

⚫ It is not in the process of issuing securities in public securities markets.

⚫ The ultimate or intermediate parent publishes consolidated financial statements that comply with

International Financial Reporting Standards.

4. Basic principle of consolidation financial statement

⚫ A group comprises at minimum a parent company and a subsidiary.

⚫ A subsidiary relationship exists where the parent company controls the subsidiary, control is normally

achieved by the parent owning a majority (>50%) of the ordinary share (voting rights) in the subsidiary.

⚫ All business combination must be accounted for the Acquisition method. The pooling of interest method is

prohibited.

⚫ The basic rule with the parent and subsidiary financial statement is that it shows all assets and liability

⚫ Intra-group items are excluded

ASSET

Non-current assets

Cost of investment

Goodwill

Current assets

EQUITY & LIABILITY

Share capital

Reserve

Current liability

Non-current liability

H

X

X

X

X

X

X

X

X

X

S

X

X

X

X

X

X

X

X

Group

X

X

X

X

X

X

X

X

X

5. Basic for consolidation statement of financial position

The group account is not equal to the sum of individual accounts

(W1) Establish the group structure

H

Date of acq 80%

S

This indicates that holding company owns 80% of the ordinary share of S and when they were acquired.

107

ACCA- FR 2021-2022 Part C-Chapter 13

(W2) Net assets of subsidiary

At date of acquisition At date of consolidation Movement

$ $

Share capital X X

Share premium X X

Retained earnings X X X

Policy adjustment X X

Unrealized profits (X) (X)

Fair Value Adjustments X X

Additional depreciation (X) (X)

X* X** X

(W3) Goodwill on acquisition

There are two methods in which Goodwill may be calculated following the update to IFRS 3:

(1) Partial Goodwill (Old method)

Cost of investment X

Less: P’s share of FV of S’s NA at DOA (X)

Goodwill on consolidation (P’s share) X

Less: impairment to date (X)

X

(2) Full Goodwill (new method)

$

Cost of investment X

Non-controlling interest at Fair Value X

FV of the subsidiary’s net identifiable assets (X)

Goodwill X

Less: impairment to date (X)

Goodwill on consolidation X

Or, this can be presented by:

Cost of investment X

Less: P’s share of FV of S’s NA at DOA (X)

Goodwill on consolidation (P’s share) X

Non-controlling interest at Fair Value X

NCI share of FV of S’s NA at DOA (X)

Goodwill on consolidation (NCI share) X

Total Goodwill X

108

ACCA- FR 2021-2022 Part C-Chapter 13

(W4) Non-controlling interest

⚫ Old Method

NCI% of S’s NA at date of consolidation

⚫ New Method

$

FV of NCI at Date of Acquisition X

NCI share of post-acquisition retained earnings NCI% x (X**- X*) X

Less: Goodwill impairments to date (X)

Total Non-controlling interest X

(W5) Group Retained earnings

$

P’s Retained earnings (100%) X

Add:P’s group share of S’ post-acquisition reserve S% × (X**-X*) X

Less: Goodwill impairments to date (X)

X

6. Explanations of group account--IFRS 3 business combination

6.1 Cost of investment

The cost of investment is used to purchase the net assets of subsidiary. In general, to acquire a company is to

acquire its net assets (equity). This must be accounted for at fair value.

6.2 Value

The cost of acquisition includes the following elements

⚫ Cash paid / deferred cash

⚫ Share for share exchange

⚫ Fair value of any other consideration, such as debentures

⚫ Contingent considerations

6.3 Deferred consideration should be:

⚫ Discounted at PV of future cash flow.

⚫ Including in non-current liability in H’s book.

⚫ Interest on deferred consideration deducted from group reserve.

6.4 Share for share exchange:

⚫ Calculate the value of consideration (use the market price)

⚫ Add extra share capital and share premium to group account

Issue cost should be deducted from proceeds of issue (i.e. share premium) not included in the cost of the

acquisition.

Professional fees and similar incremental costs incurred directly in making the acquisition—all such costs

should be expensed.

109

ACCA- FR 2021-2022 Part C-Chapter 13

6.5 Contingent consideration

⚫ The revised standard requires the acquirer to recognize the acquisition date fair value of contingent

consideration as part of the consideration for the acquiree. This ‘fair value’ approach is consistent with the

way in which other forms of consideration are valued, and fair value is defined as: ‘The amount for which an

asset could be exchanged, or a liability settled, between knowledgeable, willing parties in an arm’s length

transaction’.

⚫ In an exam question, the acquisition date fair value (or how to calculate it) of any contingent consideration

would be given. The payment of contingent consideration may be in the form of equity or a liability (issuing

a debt instrument or cash) and should be recorded as such under the rules of IAS 32, Financial Instruments:

Presentation, or other applicable standard.

⚫ Classification either as liability or equity under IAS 32 affects post-acquisition reported as follows:

A contingent consideration liability is subsequently remeasured at fair value through profit or loss until

settled for within equity.

Contingent consideration classified as equity is not remeasured and its subsequent settlement is

accounted for within equity.

Illustration

On 1/7/20X6, Company H acquires 24 million shares (80%) of the ordinary share of Company S by offering a

share for share exchange. H offer every 2 shares for every 3 shares acquired in S and a cash payment of $1 per

share payable 3 years later (discount factor 0.75). H’s cost of capital is 10% per annum. The current market

price of share in H is $2 and the nominal value is $1.

Required:

Calculate the cost of investment, and show the interest unwinding on deferred consideration on 31/12/20X6

when H prepares the CSOFP.

Answer:

Cash: deferred consideration

PV of deferred consideration: 24m×$1×0.75 = $18m

Share:

(24m×2/3)×$2 = $32m (accounted in share capital and share premium of H’s book)

Total cost of investment = 18+32 = $50m

On date of acquisition-1/7/20X6

The double entry for the consideration

Dr. Cost of investment 50m

Cr. Non-current liability 18m

Cr Share capital 16m

Cr Share premium 16m

On date of consolidation-31/12/20X6

Interest on deferred consideration: $18m x 10% x 6/12=$0.9 m

Dr. Group reserve 0.9m

Cr. Non-current liability 0.9m

110

ACCA- FR 2021-2022 Part C-Chapter 13

6.6 Fair value adjustment of subsidiary

⚫ As mentioned above, acquisition is to acquire the net asset of subsidiary. Therefore, the net asset of

subsidiary should be stated at fair value.

⚫ At acquisition date, put an adjustment into the Net Assets working of the subsidiary to bring the net assets

to fair value.

⚫ At the consolidation date, any adjustments remaining from acquisition must be accounted for both on the

face of the Statement of Financial Position and in the net assets working. Any changes in adjustment (e.g.

depreciation) should also be reflected.

Working: fair value adjustment

At date of

acquisition

At date of

consolidation

Movement

Ordinary share + reserve X X X

Fair value adjustments:

Land (no depreciation) X X -

Non-current assets X X -

Less: Extra depreciation/ amortization - (X) (X)

Inventory X - (X)

Loan (X) (X) -

Interest payment - (X) (X)

X X X

Fair value adjustment is only available when the related asset is still held by the group at the consolidation date.

We always assume that the inventory is sold at the consolidation date unless the question says no. 6.7 Inter-company transactions

6.7.1 Original double entries

H sells goods to S:

In H’s book In S’s book

Dr. Cash Dr. P/L – Cost of sale

Dr. Receivables Cr. Cash

Cr. P/L -Sale Cr. Payables

6.7.2 Outstanding balance of transactions:

In group account, the outstanding balance in receivables and payable (current account) should be

cancelled:

Dr. Payables

Cr. Receivables

6.7.3 Cash/goods in transit

Adjust for the cash/goods and then eliminate the inter-company balance

Cash in transit adjusting entry Goods in transit adjusting entry

Dr. cash

Cr. Receivable

Dr. Inventory

Cr. Payable

111

ACCA- FR 2021-2022 Part C-Chapter 13

6.7.4 Provision for un-realized profit (purp)

⚫ In the buying company's accounts, inventory will be valued at acquisition cost which now includes the profit

element earned by the selling company. The problem is that from the group’s point of view this profit has

not yet been realised because no sale has been made outside the group and therefore closing inventories

are overstated by the profit element.

⚫ Remember that the objective of consolidated accounts is to reflect the financial results and position of the

group as a single entity. To achieve this, we need to make an adjustment.

Un-realized profit in inventory

If H sold goods to S: If S sold goods to H

Dr. Group retained earnings Dr. Group retained earnings

Cr. Group inventory (whole) Dr. Non-controlling interest

Cr. Group inventory (whole)

Unrealized profit in non-current asset

Apart from the difference between the selling price and NBV, unrealised price should be also included the

depreciation. So, the easiest way to calculate the adjustment required is to compare carrying value of the

asset before and after the transaction.

Format:

Before

transfer After transfer Difference

Carrying Value at transfer X X*

(transfer price)

Depreciation from transfer to B/S date (X) (X)

Carrying Value at reporting date X X X(purp)

6.7.5 Inter-group loan

Normally Parents Company lends the loan to subsidiary, from the group account point of view, Group

Company does not issue the loan outside, and therefore the inter-group loan should be cancelled.

⚫ Original entries

H: Dr. Investment / Receivables S: Dr. Cash

Cr. Cash Cr. Loan

⚫ Reverse entries

In group B/S eliminate both: Dr. Loan

Cr. Investment/Receivables

Loan interest: as interest is normally paid by cash, no elimination in CSFP.

If H sold non current assets to S: If S sold non current assets to H

Dr. Group retained earnings Dr. Group retained earnings

Cr. Group non-current assets Dr. Non-controlling interest

Cr. Group non-current assets

112

ACCA- FR 2021-2022 Part C-Chapter 13

6.8 Treatment of goodwill and NCI

6.8.1 The rationale of the ‘new’ method

⚫ It has long been argued (mainly by academics) that the traditional (‘old’) method of calculating goodwill

only recognizes the goodwill acquired by the parent, and is based on the parent’s ownership interest

rather than the goodwill controlled by the parent. In other words, any goodwill attributable to the NCI

is not recognized.

Illustration

6.8.2 Old method:

Parent pays $100m for 80% of Subsidiary which has net assets with a fair value of $75m.

Goodwill of $40m (100m - (80% x $75m)) would be recognized, and the NCI would be $15m (20% x $75m).

Hypothetically, if we presumed that purchasing 100% of Subsidiary would have cost proportionately more,

the consideration would have been $125m ($100m/80%) and goodwill would then be $50m ($125m -

$75m) and there would be no NCI.

This demonstrates that, where an NCI exists, the traditional consolidation method only records the

parent’s share of the goodwill, and the NCI is carried at its proportionate share of the fair value of the

subsidiary’s net assets (which excludes any attributable goodwill). The argument goes that as we

consolidate the whole of a subsidiary’s other assets (and liabilities), why should goodwill be any different?

After all, it is an asset.

6.8.3 New method

Progressing the above example, assuming that the value of the goodwill of the NCI is proportionate to

that of the parent, consolidated goodwill of $50m would be recognized (this includes both the controlling

($40m) and the NCI ($10m) in goodwill) and the NCI would be $25m ($15m + $10m attributed goodwill).

In effect, consolidated goodwill and the NCI are ‘grossed up’ by the NCI share of goodwill ($10m, in this

case).

Although this may seem new, it is in fact an extension of the methodology in IAS 36, Impairment of Assets

when calculating the impairment of goodwill of a cash generating unit (CGU) where there is an NCI.

IFRs3 requires positive goodwill arising on acquisition to be capitalised as an intangible non-current asset

and then tested annually for possible impairments. Amortization of goodwill is not permitted by the

standard.

Negative goodwill arises where the cost of the investment is less than the valued of net assets purchased.

The negative goodwill is very rare. Once it is established that negative goodwill has arisen it should be

credited directly to the income statement.

Dr. Goodwill

Cr. Group retained earnings

If asked to describe goodwill, traditional aspects such as product reputation, skilled workforce, site

location, market share, and so on, all spring to mind. These are perfectly valid, but in an acquisition,

goodwill may contain other factors such as a premium to acquire control, and the value of synergies (cost

savings or higher profits) when the subsidiary is integrated within the rest of the group. While the NCI can

legitimately lay claim to its share of the more traditional aspects of goodwill, it is unlikely to benefit from

the other aspects, as they relate to the ability to control the subsidiary.

Thus, it may not be appropriate to value the NCI’s share of goodwill proportionately with that of the parent.

113

ACCA- FR 2021-2022 Part C-Chapter 13

The revised IFRS 3 seeks to resolve this problem (under the ‘new’ method) by requiring the NCI to be

measured at its ‘fair value’, rather than at ‘its proportionate share of the (fair value of the) acquiree’s

identifiable net assets’. The difference between these two values is, effectively, the NCI share of goodwill

which may or may not be proportionate to the parent’s share of goodwill.

The standard illustrates the calculation of consolidated goodwill as:

‘Consideration paid by parent + non-controlling interest - fair value of the subsidiary’s net identifiable

assets = consolidated goodwill’.

Note that the NCI in the above formula may be valued at its proportionate share of the subsidiary’s net

identifiable assets, in which case consolidated goodwill would be that relating to the parent only (the ‘old’

method). Alternatively, the NCI may be at its fair value (the ‘new’ method), in which case the consolidated

goodwill represents that of both the parent and the NCI.

The formula represents a different approach to the previous, more traditional, method of calculating

goodwill which was calculated as the difference between the consideration paid by the parent and its

share of the fair value of the subsidiary’s net identifiable assets. This method did not refer to the NCI

because it was only intended to recognize the parent’s share of goodwill.

It is important to realise that the new ‘formula’ only applies at the date of acquisition. Subsequent to

acquisition, both the NCI and the fair value of the subsidiary’s net assets will have changed.

The standard recognizes that there may be many ways of calculating the fair value of the NCI and does

not go into detail on this matter, but it recognizes that the market price of the subsidiary’s shares prior to

the acquisition may be a reasonable basis on which to value the shareholding of the NCI.

6.9 Group reserve

Group reserve includes:

⚫ H’s reserve at reporting date + share of profit created by subsidiary post acquisition Or

⚫ H’s reserve at acquisition date + group profit post acquisition (CIS method)

Unrealised profit should be excluded from group reserve.

Goodwill impairment should be excluded from group reserve.

6.10 Inter-company dividend

⚫ Inter-company’s dividend is the dividend that subsidiary paid to parent company. If the dividend is payable

and has not been paid, the outstanding balance in CSFP should be cancelled. If the dividend is paid by cash,

no adjustment is needed.

Example:

H owns 80% of ordinary share of S, at the year end, S proposes to pay H dividend of 10,000, but the dividend

has not been paid, and H recorded the dividend receivables in its books.

In S’s book: In H’s book

Dr. Reserve 10000 Dr. Dividend receivable 8000

Cr. Proposed dividend –parent 8000 Cr. Reserve 8000

- NCI. 2000

114

ACCA- FR 2021-2022 Part C-Chapter 13

Eliminate the inter-company dividend:

Dr. Proposed dividend – parent 8000

Cr. Dividend receivable 8000

Note: only payable and receivable dividend should be eliminated, if the dividend has been already paid, no

elimination needed. Students should read the question carefully.

Illustration: dividend is paid by cash (same example)

In S’s book: In H’s book

Dr. Reserve 10000 Dr. Cash 8000

Cr. Cash–parent 8000 Cr. Reserve 8000

- NCI 2000

6.11 Detailed format of consolidation--summary

(W1) Cost of investment

⚫ Cash

⚫ Share for share exchange

• Calculate the value of consideration

• Add extra share capital and share premium in group account

⚫ Deferred consideration

• Discounted at PV of future cash flow

• Including in non-current liability in H’s book

• Interest on deferred consideration deducted from group reserve

⚫ Contingent consideration—FV at DOA

(w2) URP

⚫ Inventory

Identify H—S or S—H

Calculate the whole URP.

If H sold goods to S: If S sold goods to H

Dr. Group reserve Dr. Group reserve

Cr. Group inventory (whole) Dr. NCI

Cr. Group inventory (whole)

If transfer of non-current asset

Before transfer After transfer Difference

CV at transfer X X*(transfer price)

Depreciation from transfer

to B/S date (X) (X)

CV at B/S date

X X X(URP)

115

ACCA- FR 2021-2022 Part C-Chapter 13

If H sold goods to S

Dr. Group reserve

Cr. Group non-current assets

If H sold goods to S:

If S sold goods to H

Dr. Group reserve

Dr. NCI

Cr. Group non-current assets

Note: For the convenience of calculation, we put the URP (S—H) into W3 Net asset of subsidiary; put the URP

(H—S) into W6 Group reserve. See below for details.

(W3) Net asset of subsidiary

Share capital

Share premium

Reserve

Fair value adjustments:

Inventory

Non-current asset

- Extra depreciation

Loan (FV= PV of int. and capital)

- interest payment

Less: Any errors

Less: URP (S – H)

Net asset at Acq

$

X

X

X

X

X

X

-

(X)

-

-

X*

Net asset at B/S

$

X

X

X

X

-

X

(X)

(X)

X/(X)

(X)/X

X**

Any error:

In some exam question, some items subsidiary recorded incorrectly or forgets to record, and these items will

influence the net asset, students should adjust these errors in this form.

e.g. Subsidiary did not record the central cost charged by holding company of $4m.

In this form the net assets at B/S should be deducted by $4m

(W4) Goodwill on acquisition (new Method)

(1) Partial Goodwill (Old method)

Cost of investment X

Less: P’s share of FV of S’s NA at DOA (X)

Goodwill on consolidation (P’s share) X

Less: impairment to date (X)

X

116

ACCA- FR 2021-2022 Part C-Chapter 13

(2) Full Goodwill (new method)

$

Cost of investment X

Non-controlling interest at Fair Value X

FV of the subsidiary’s net identifiable assets (X)

Goodwill on consolidation X

Less: impairment to date (X)

X

Or , this can be presented by:

Cost of investment X

Less: P’s share of FV of S’s NA at DOA (X)

Goodwill on consolidation (P’s share) X

Non-controlling interest at Fair Value X

NCI share of FV of S’s NA at DOA (X)

Goodwill on consolidation (NCI share) X

Total Goodwill X

(W5) Non-controlling interest

(1) Old Method NCI% of S’s NA at date of consolidation X

(2) New Method

$

FV of NCI at Date of Acquisition X

NCI share of post-acquisition retained earnings NCI% x (X**- X*) X

Less: Goodwill impairments to date (X)

Total Non-controlling interest X

(W6) Group reserve

$

H’s reserve (100%) X

Add: P’s group share of post-acquisition reserve G% × (X**-X*) X

Less: URP (H—S) (X)

Less: Goodwill impairment to date (X)

Less: interest on deferred consideration (if any) (X)

X

117

ACCA- FR 2021-2022 Part C-Chapter 13

Questions

Example 1 – PIPER AND SWANS (CSOFP-2 methods)

Draft balance sheets of Piper and Swans on 31 December 20x1 are as follows.

Piper Swans

$000 $000

Non-current assets 90 100

Investment in Swans at cost 110

Current assets 50 30

250 130

Equity and liabilities

Capital and reserves

Ordinary share capital $1 100 100

Retained earnings 120 20

220 120

Current liabilities 30 10

250 130 Piper had bought 80% of the ordinary shares of Swans on 1 January 20X1 when the retained earnings of Swans had

a balance of $10,000. No impairment of goodwill has occurred to date.

Required:

1. Prepare a consolidated statement of financial position as at 31 December 20X1, assuming FV of NCI@ ACQ

date=$24,000.

2. Prepare a consolidated statement of financial position as at 31 December 20X1, assuming that the Piper group

values the NCI using the proportion of NA method.

Example 2 HOT & SOUR (CSOFP-FV adjustment & Impairment loss)

Hot (H) acquired 75% of the ordinary share capital of Sour (S), one year ago, on 1 January. At that

date the fair value of S's fixed assets was $40,000 greater than their net book value and the

balance of RE was $42,000. The remaining useful economic life of fixed assets from acquisition

was ten years. An impairment test on the value of goodwill has resulted in the need to write

goodwill down by $11,000.

The statement of financial position of both companies at 31 December are given below.

H S

$000 $000

Non-current assets 126 56

Investment in S 102 Nil

228 56

Receivable 32 13

Inventory 18 15

Cash 12 5

62 33

290 89

118

ACCA- FR 2021-2022 Part C-Chapter 13

Ordinary shares 100 10

Retained earnings 170 69

270 79

Payable 20 10

290 89

Required:

Prepare H's consolidated statement of financial position as at 31 December, assuming FV of NCI

at date of acquisition is $13,000.

Example 3 Calculation for cost of investment

On 1/7/20X6, Company H acquires 24 million shares (80%) of the ordinary share of Company S by offering a share

for share exchange. H offer every 2 shares for every 3 shares acquired in S and a cash payment of $1 per share

payable 3 years later (discount factor 0.75). H’s cost of capital is 10% per annum. The current market price of share

in H is $2 and the nominal value is $1.

Required:

Calculate the cost of investment, and show the interest unwinding on deferred consideration on 31/12/20X6 when

H prepares the CSOFP.

Example 4 Intercompany Trading Balance

In EXAMPLE 2 HOT & SOUR, the extracts from the SOFP of H and its subsidiary S at y/e are

H S

Receivable due from S 22,000

Payable due to H (5,000)

The current accounts do not agree due to:

1, Inventory in transit from H to S of $5,000

2, Cash in transit from S to H of $10,000

3, Adm fee charged from H to S $2,000

Required:

Show how the above information would be reflected in the CSOFP at 31/12/20X8.

Example 5 Unrealised Profit in inventory

Further adjustment for H & S. In current year transfers are:

1, From H to S, $20,000, mark-up 25%, half in stock.

2, From S to H, $150,000, mark-up 50%, 20% in stock.

3, H sold goods to S at a price of $12,000. These goods cost H $9,000. During the year S sold $10,000 (at the cost to

S) of these goods for $15,000.

Required:

Show how the above information would be reflected in the CSOFP.

119

ACCA- FR 2021-2022 Part C-Chapter 13

Example 6 NCA Transits between Intercompany

Further adjustment for H & S. H company transfers an item of plant to S company for $6,000 at the start of current

year and it has been used for 3 years. The plant originally cost H $10,000 and has been depreciated at the rate 20

% on a straight basis.

Required:

Show how the above information would be reflected in the CSOFP.

Example 7 Intercompany dividend

Further adjustment for H & S. At the year end, S proposes to pay dividend of 10,000, but the dividend has not been

paid, and H recorded the dividend receivables in its books.

Example 8 RUSH & ALDRIDGE (Mid-year Acquisition and Negative goodwill)

Rush acquired 80% of equity shares of Aldridge on 30 September 20X8, for $58,000. On 1 January

20X8, the retained earnings and equity shares of Aldridge was $48,000 and $12,000. During the

year to 31 December 20X8 Aldridge made a profit after tax of $20,000.

Required:

Calculate the goodwill of Aldridge on date of acquisition.

What should be the correct accounting treatment for negative goodwill? Example 9 – HARDLY (EXAM – STYLE QUESTION)(自我练习题目,无视频讲解,可参考答案解析)

On 1 April 20X5 Hardly acquired 4 million of Sibling ordinary shares, paying $4.50 each. At the date

of acquisition, the retained earnings of Sibling were $8,400,000.

Reproduced below are the draft statement of financial position of the two companies at 31 March

20X8:

Hardly Sibling

$000 $000 $000 $000

Non-current assets

Land and buildings 22,000 12,000

Plant and equipment 20,950 10,220

Investment in Sibling plc

Equity 18,000

60,950 22,220

Current assets

Inventory 9,850 6,590

Receivables 11,420 3,830

Cash and bank 490 -

21,760 10,420

82,710 32,640

120

ACCA- FR 2021-2022 Part C-Chapter 13

Equity

Ordinary shares of $1 each 10,000 5,000

Retained earnings 51,840 14,580

61840 19580

Non-current liabilities

10% Debentures 20Y1 12,000 6,000

Current liabilities

Creditors 5,600 3,810

Bank overdraft - 770

Provision for taxation 2,470 1,980

Provision for dividends

Ordinary 800 500

8,870 7,060

82,710 32,640

The following information is relevant:

(1) Included in the land and buildings of Sibling is a large area of development land at its cost of

$5 million. Its fair value at the date Sibling was acquired was $7 million and by 31 March 20X8

this had risen to $8.5 million. The group valuation policy for development land is that it should

be carded at fair value and not depreciated.

(2) Also at the date of Sibling plc's acquisition the plant and equipment included plant that had a

fair value of $4 million in excess of its carrying value. This plant had a remaining life of five

years. The group calculates depreciation on a straight-line basis. The fair value of Sibling plc's

other net assets approximated to their carrying values.

(3) During the year Sibling sold goods to Hardly for $1.8 million. Sibling adds a 20% mark-up on

cost to all its sales. A quarter of the goods were included in Hardly plc's stock at 31 March 20X8.

The balance on the current accounts of the parent and subsidiary was $240,000 on 31 March

20X8.

(4) Hardly has not accounted for any dividends receivable from Sibling. Dividends were proposed

before the year end.

(5) Consolidated goodwill has suffered an impairment of $1,488,000.

Required:

(a) Prepare the consolidated statement of financial position of Hardly at 31 March 20X8.

(20 marks)

121

ACCA- FR 2021-2022 Part C-Chapter 14

Chapter 14 The Consolidated Statement of Profit or Loss and Other

Comprehensive Income

Content

1.Accounting for subsidiary companies

2.Accounting for disposal of a subsidiary

Session content

Disposal of subsidiary

1. Accounting for subsidiary companies

1.1 Control and ownership

⚫ A statement of profit or loss and other comprehensive income (SPL/OCI) shows the profit generated by

resources disclosed in the related statement of financial position. In the same way, the consolidated

statement of profit or loss and other comprehensive income (CSPL/OCI) shows all income generated by the

group's resources (i.e. by the net assets shown in the consolidated statement of financial position).

⚫ To reflect this we must prepare the CSPL/OCI on a basis consistent with the consolidated statement of

financial position. In particular, the CSPL/OCI must show incomes generated from the net assets under P's

control.

⚫ To do this, we include in the CSPL/OCI all of S's income and expenses (100%), line by line, down to and

including net profit for the period (profit after tax). This is the case even if our equity share in S is less than

100%.

⚫ To reflect our ownership in S we must then show the amounts attributable to the ‘Equity holders of the

parent' the parent's shareholders and the amount attributable to the "Non-controlling interest".

⚫ If S has paid dividends to P these should be cancelled against P’s investment income. The only dividends to

be shown on the SPL/OCI will be those of the parent company.

⚫ Indeed, this is just one example of intra-group transactions that must be eliminated from the consolidated

accounts. We have already dealt with such transactions in the consolidated statement of financial position

(e.g. unrealized profit in closing inventories). We now look at the impact of intra-group transactions on the

CSPL/OCI.

Mid –year

acquisition

IFRS 3

Business

combinations Dividends

Intra-group

Transactions

Unrealised

Profit in inventory

CSPL/OCI

122

ACCA- FR 2021-2022 Part C-Chapter 14

1.2 Intra-group transactions

⚫ Intra-group transactions are those which take place within the group and do not involve outside entities.

⚫ The objective of consolidated accounts is to display group as a single entity, Intra-group transactions have a

nil effect on the group as a whole and must be excluded from the consolidated accounts.

Sales and purchase

URP in inventory

Inter-company interest

Inter-company dividend income and dividend proposed

1.2.1 Sales and purchase

Eliminated in full in CSPL

Dr Revenue

Cr Cost of sales

1.2.2 URP in inventory if any

Step 1: Eliminate transaction (sales and purchase)

Dr Revenue

Cr Cost of sales

Step 2: Adjust for URP inventory

P to S S to P

Dr cost of sales

Cr inventory (CSFP item)

Dr cost of sales

Cr inventory (CSFP item)

1.2.3 Inter-company interest

If there is a loan outstanding between group companies the effect of any loan, interest received and paid

must be eliminated from the consolidated SP/L.

Dr Group investment income

Cr Group finance costs

1.2.4 Inter-company dividend income and dividend proposed

Eliminated in full

Dr P’s investment income (group)

Cr S’s RE

Dividend proposed from S is directly reduced in RE, thus it doesn’t make effect on PAT attributed to NCI.

1.3 A pro-forma CSPL/OCI

⚫ Add results of the subsidiary of those of the parent on a line by line basis even if the interest in the subsidiary

is less than 100%

⚫ The NCI in the subsidiary’s profit and total comprehensive income is shown after PAT. It is calculated as NCI %

x Subsidiary’s PAT or total comprehensive income.

123

ACCA- FR 2021-2022 Part C-Chapter 14

Consolidated statement of profit or loss and other comprehensive income for the year ended XXX

$ $

Revenue (H+S – All inter-group transaction) X

Cost of sale (H+S – All inter-group transaction) X

Add: URP (H to S) and (S to H) X

Add: addition depreciation in S X

(X)

Gross profit X

Less: distribution cost (H+S) (X)

Less: administrative expense (H+S) (X)

Less: goodwill impairment (only S) (X)

Investment income (external only) X

Finance cost (external only) (X)

Income from associate (see Chapter 15)

G% of A’s PAT X

Less: goodwill impairment of A (X)

Less: A urp (A to H & H to A) (X)

X

Profit for the year X

Other comprehensive income

Revaluation surplus of property plant and equipment X

Fair value gain on financial assets X

X

Total comprehensive income X

Profit attributable to:

Equity holders of the Parent X

Non-controlling interest

PAT of S X

Less: urp (S to H) (X)

Less: additional depreciation (X)

Less: others (X)

Adjusted profit of S X

Adjusted profit of S×(1-G %) X

X

Total comprehensive income attributable to:

Equity holders of the Parent X

Non-controlling interest ((adjusted profit + OCI of S) x (1-G%)) X

X

124

ACCA- FR 2021-2022 Part C-Chapter 14

1.4 Acquisition of a subsidiary in mid-year

If we acquire a subsidiary in mid-year the CSPL/OCI includes S's results only from the date of acquisition, i.e. the

date on which control is obtained.

We adopt the following:

⚫ Consolidate S from date of acquisition.

⚫ Identify the net assets of S at the date of acquisition in order to calculate goodwill.

⚫ Assume that revenue and expenses accrue evenly over the year unless the contrary is indicated.

⚫ Time-apportion the results of S in the year of acquisition and deduct the post acquisition intra-group items

as normal.

2. Accounting for disposal of a subsidiary

During the year, one entity may sell some or all of its shares in another entity causing a loss of control. In FR

syllabus, it restricts to disposals of the parent’s entire investment in the subsidiary.

2.1 Disposals in the individual financial statements

The profit on disposal in the investing entity's, i.e. parent’s individual financial statements is calculated as

follows:

$

Sales proceeds X

Carrying amount (usually cost) of shares sold (X)

Profit/ (Ioss) on disposal X/(X)

2.2 Disposals in the consolidated financial statements

When the parent disposes the entire investment in the subsidiary, that is on disposal of the subsidiary, we shall

derecognise its assets, liabilities, goodwill and non-controlling interest and calculate a profit or loss on disposal.

A proforma calculation of disposal gain (loss) in the consolidated financial statements is shown below.

$

Sales proceeds X

FV of NCI @disposal date X

Less:

Goodwill (X)

FV of net assets of subsidiary @disposal date (X)

Gain/(loss) of disposal to group X/(X)

Example 1

A acquires 80% of the share capital of B on 01/08/20*6 and is preparing its group financial statement for the year

ended 31/12/20*6.

How will B’s results be in included in the group statement of profit or loss?

A 80% of B’s revenue and expenses for the year end 31/12/20*6

B 100% of B’s revenue and expenses for the year end 31/12/20*6

C 80% of B’s revenue and expenses for the period 01/08/20*6- 31/12/20*6

D 100% of B’s revenue and expenses for the period 01/08/20*6- 31/12/20*6

125

ACCA- FR 2021-2022 Part C-Chapter 14

Example 2

Which ONE of the following statements is incorrect?

A Parent include in the CSPL/OCI all of subsidiary's income and expenses (100%), line by line, down to and

including net profit for the period (profit after tax).

B If subsidiary has paid dividends to parent these should be cancelled against parent’s investment income. Any

dividend income shown in CSPL/OCI must arise from investments other than those in subsidiaries or associates.

C When Inter-company trading incurs, such trading will be included in the sales revenue of one group company

and the purchases of another. Cancel these transactions on consolidation only when the inventories are unsold.

D To reflect ownership in subsidiary, parent must then show the amounts attributable to the ‘Equity holders of

the parent' and the amount attributable to the 'Non-controlling interest'.

Example 3

A acquires 80% of the share capital of B on 01/10/20*6. During the year, B sold the inventory to A for $200,000 per

month. B made a mark-up on cost of 25% on these sales. At 31/12/20*6, A still held $100,000 inventory provided

from B after acquisition.

What is the amount of the unrealized profit attributable to NCI?

A there is no impaction on the profit attributable to NCI

B profit attributable to NCI will be decreased by $4,000

C profit attributable to NCI will be increased by $4,000

D profit attributable to NCI will be decreased by $20,000

Example 4 – Disposal of subsidiary

Rock has held a 70% investment in Dog for two years. Goodwill has been calculated using the full goodwill method.

There have been no goodwill impairments to date.

Rock disposes of all of its shares in Dog. The following information has been provided:

$

Cost of investment 2,000

Dog - Fair value of net assets at acquisition 1,900

Dog- Fair value of the non-controlling interest at acquisition 800

Sales proceeds 3,000

Dog - Net assets at disposal 2,400

Required:

Calculate the profit or loss on disposal in:

(a) Rock's individual financial statements

(b) the consolidated financial statements.

126

ACCA- FR 2021-2022 Part C-Chapter 15

Chapter 15 Accounting for Associates

Content

1.Defining an associated undertaking

Session content

1. Defining an associated undertaking

1.1 Associates and subsidiaries

⚫ A group of companies consists of a parent company and its subsidiaries. Subsidiaries are those companies

over which the parent exercises control generally by virtue of holding more than 50% of the equity shares.

⚫ Often a group will be able to exercise significant influence over another company by virtue of a substantial

shareholding which falls short of 50%. Such a company is called an associated undertaking of the group.

When we use the term 'group companies' we are referring to the parent company and its subsidiaries; an

associated undertaking is not a group company.

⚫ An associated undertaking is therefore defined as an entity over which the group exercises significant

influence, but not control. If the group holds 20% or more of the company's equity we presume that

significant influence exists however this presumption can be rebutted if there is clear evidence to the

contrary.

30%

A

1.2 Definitions

⚫ Significant influence—is the power participate in the financial and operating policy decisions of an entity in

which an investment is held, but is not control over these policies.

⚫ Equity method—a method of accounting whereby the investment is initially recorded at cost and adjusted

thereafter for the post-acquisition change in the investor’s share of net assets of the associate.

The rules on accounting for associated undertakings are contained mainly IAS 28 Investments in associates.

CSOFP

CIS

PARENT

+

SUB

+

ASSOCIATE

P

Controls

S

127

ACCA- FR 2021-2022 Part C-Chapter 15

⚫ IAS 28 requires the use of the equity method to account for associates. The investing group will now have to

disclose its share of associate’s profit after tax on the face of the SPL/OCI as ‘Share of profit of associates’

and disclose its share of the net assets of the associate plus any unimpaired goodwill on the statement of

financial position as ‘Investments in associates’.

⚫ In the investing company’s individual accounts the statement of financial position shows the investment in

associated undertaking(s) as a non-current asset investment, stated (usually) at cost or in accordance with

IFRS 9 (at fair value). The individual company's SPL/OCI shows dividend income received from associates

under the heading 'investment income'.

1.3 The equity method of accounting

1.3.1 The consolidated statement of financial position

⚫ In non-current asset investments, we replace the investment shown in the individual company

statement of financial position with the group share of A's net assets at the reporting date, in one line.

The item is 'Investments in associates'.

⚫ In group reserves, we include P’s share of A's post - acquisition reserves (just as we do for a subsidiary).

⚫ Next, we cancel the investment in A in the individual company's books against the share of A's net

assets acquired (at fair value). The difference is a premium or discount on acquisition (in effect,

goodwill).

⚫ To calculate these amounts, do a net assets working for A (as we did for a subsidiary).

⚫ Goodwill must be calculated and treated in the same way as for subsidiaries but presented within the

'investments in associates' item and separately disclosed.

⚫ Dividends from associates are excluded from the CSPL/OCI, the group share of the associate’s profit is

included instead.

Relevant workings

⚫ Associates in the consolidated statement of financial position

The associate is included as a non-current asset investment calculated as:

$000

Cost of investment X

Share of post acquisition profits X

Less: impairment losses (X)

Less: PURP (P = seller) (X)

X

The group share of the associate's post acquisition profits or losses and the impairment of goodwill will

also be included in the group retained earnings calculation.

Or:

$000

Parents shares of associates net assets at reporting date X

Carrying Goodwill/Premium X

Less: PURP (P = seller) (X)

X

128

ACCA- FR 2021-2022 Part C-Chapter 15

⚫ Fair values and the associate

If the fair value of the associate's net assets at acquisition are materially different from their book

value the net assets should be adjusted in the same way as for a subsidiary.

⚫ Balances with the associate

Generally the associate is considered to be outside the group. Therefore balances between group

companies and the associate will remain in the consolidated statement of financial position.

If a group company trades with the associate, the resulting payables and receivables will remain in

the consolidated statement of financial position.

⚫ Unrealised profit in inventory

Unrealised profit on trading between group and associate must be eliminated to the extent of the

investor's interest (i.e. % owned by parent).

Adjustment must be made for unrealised profit in inventory as follows.

-Investor selling to associate --- downstream

Dr Share of profit of associate (Group R/E)

Cr Investment in associate

-Associate selling to investor --- upstream

Dr Share of profit of associate (Group R/E)

Cr Investment in associate

(The same as downstream)

1.3.2 Associates in the consolidated statement of profit or loss

Equity accounting

⚫ No dividends from the associate.(Excluded from the consolidated SP/L)

⚫ Include group share of the associate's profit after tax less any impairment of the associate in the year.

(included below group profit from operations).

Trading with the associate

⚫ Generally the associate is considered to be outside the group.

⚫ Therefore:

• Any sales or purchases between group companies and the associate are not normally eliminated

and will remain part of the consolidated figures in the SP/L.

• Adjust for the unrealised profit in inventory.

Example 1 Consolidated statement of profit or loss & Associate

(Referred Dec 2016 Q31) On 1 January 20X6, Laurel Co acquired 60% of the equity share capital of Rakewood Co in a share exchange in which

Laurel Co issued three new shares for every five shares it acquired in Rakewood Co. The share issue has not yet been

recorded by Laurel Co. Additionally, on 31 December 20X6, Laurel Co will pay to the shareholders of Rakewood Co

$1.62 per share acquired. Laurel Co's cost of capital is 8% per annum. At the date of acquisition, shares in Laurel Co and Rakewood Co had a market value of $7.00 and $2.00 each

respectively.

129

ACCA- FR 2021-2022 Part C-Chapter 15

Statements of profit or loss for the year ended 30 September 20X6

Laurel Co

$'000

Rakewood Co

$'000

Revenue 84,500 52,000

Cost of sales (58,200) (34,000)

Gross profit 26,300 18,000

Distribution costs (2,000) (1,600)

Administrative expenses (4,100) (2,800)

Investment income (note (iv)) 500 400

Finance costs (300) nil

Proft before tax 20,400 14,000

Income tax expense (4,800) (3,600)

Profit for the year 15,600 10,400

Equity as at 1 October 20X5:

$'000 $'000

Equity shares of $1 each 20,000 15,000

Retained earnings 72,000 25,000 The following information is relevant:

(i) At the date of acquisition, Laurel Co conducted a fair value exercise on Rakewood Co's net assets which were equal to their carrying amounts with the following exceptions:

- An item of plant had a fair value of $4m above its carrying amount. At the date of acquisition it had a remaining life of two years

- Inventory of $800,000 had a fair value of $1m. All of this inventory had been sold by 30 September 20X6

(ii) Laurel Co's policy is to value the non-controlling interest at fair value at the date of acquisition. For this purpose Rakewood Co's share price at 1 January 20X6 can be deemed to be representative of the fair value of the shares held by the non-controlling interest.

(iii) Laurel Co had traded with Rakewood Co for many years before the acquisition. Sales from Rakewood Co to Laurel Co throughout the year ended 30 September 20X6 were consistently $1.2m per month. Rakewood Co made a mark-up on cost of 20% on these sales. Laurel Co had $1.8m of these goods in inventory as at 30 September 20X6.

(iv) Laurel Co's investment income consists of:

- Its share of a dividend of $500,000 paid by Rakewood Co in August 20X6

- A dividend of $200,000 received from Artic Co, a 25% owned associate which it has held for several years. The profit after tax of Artic Co for the year ended 30 September 20X6 was $2.4m

(v) Assume, except where indicated otherwise, that all items of income and expense accrue evenly throughout the year.

(vi) An impairment review was undertaken at year end. It was decided that goodwill hadreduced in value by 10%. Goodwill impairments are charged to administrative expense.

Required:

(a) Calculate the consolidated goodwill at the date of acquisition of Rakewood Co. (7 marks)

(b) Prepare the consolidated statement of profit or loss for Laurel Co for the year ended 30 September 20X6.

(13marks)

130

ACCA- FR 2021-2022 Part D-Chapter 16

Part D ANALYSING AND INTERPRETING FINANCIAL

STATEMENTS OF SINGLE ENTITIES AND GROUPS

Chapter 16 Interpretation of Financial Statements

Session content

Interpretation of financial statements

Interpretation of financial statements

Profitability

Ratios

Liquidity

Ratios

Long-term

Financial

Stability Ratio

Investor

Ratios

Limitations Current Cost

and current

Purchasing

power accounts

Related party

transactions

1. Interpreting financial information

Financial statements on their own are of limited use. In this chapter we will consider how to interpret them and

gain additional useful information from them.

Users of financial statements

When interpreting financial statements it is important to ascertain who are the users of accounts and what

information they need:

⚫ Shareholders and potential investors – primarily concerned with receiving an adequate return on their

investment, but it must at least provide security and liquidity.

⚫ Suppliers and lenders – concerned with the security of their debt or loan.

⚫ Management – concerned with the trend and level of profits, since this is the main measure of their success.

Other potential users include:

⚫ Bank managers

⚫ Financial institutions

⚫ Employees

⚫ Professional advisors to investors

⚫ Financial journalists and commentators

Content

1.Interpreting financial information

2.Ratio analysis

131

ACCA- FR 2021-2022 Part D-Chapter 16

2. Ratio analysis

A number of ratios can be calculated to help interpret the financial statements.

In an examination question you will not have time to calculate all of the ratios presented in this chapter so you

must make a choice:

⚫ Choose those relevant to the situation;

⚫ Choose those relevant to the party you are analyzing for;

⚫ Make use of any additional information given in question to help your choice;

Analysis will, in practice, be limited in the analysis that can be performed by the amount of information available.

They are unlikely to have access to all of the facts which are available to a company’s managements.

In the examination the information which can be provided about a company in any one question will be limited.

The first part of such a question may ask you to interpret the available information, while you may also be asked

to state what further information you require.

If you are asked to request further information, make the request specific to the scenario given and state the

reason you would be looking for that particular piece of information.

2.1 Commenting on ratios

Ratios are of limited use on their own, thus most of the marks in an examination question will be available for

sensible, well-explained and accurate comments on the key ratios.

If you doubt that you have anything to say, the following points should serve as a useful checklist:

⚫ What does the ratio literally mean?

⚫ What does a change in the ratio mean?

⚫ What is the norm?

⚫ What are the limitations of the ratio?

2.2 Profitability ratio

2.2.1 Gross profit margin

Gross Profit / Sales Revenue x 100%

This is the margin that the company makes on its sales, and would be expected to remain reasonably

constant.

Since the ratio is affected by only a small number of variables, a change may be traced to a change in:

⚫ Selling prices – normally deliberate through sometimes unavoidable, e.g. because of increased

competition.

⚫ Sales mix – often deliberate.

⚫ Purchase cost – including carriage or discounts.

⚫ Production cost – materials, labor or production overheads.

⚫ Inventory – errors in counting, valuing or cut-off, inventory shortages.

132

ACCA- FR 2021-2022 Part D-Chapter 16

Comparing gross profit margin over time

If gross profit has not increased in line with sales revenue, you need to establish why not. Is the

discrepancy due to:

⚫ Increased ‘purchase’ costs: if so, are the costs under the company’s control (i.e. does the company

manufacture the goods sold?)

⚫ Inventory write-offs (likely where the company operates in a volatile marketplace, such as fashion

retail)? Or

⚫ Other costs being allocated to cost of sales – for example, research and development (R&D)

expenditure?

Inter-company comparison of gross profit margin

⚫ Inter-company comparison of margins can be very useful but it is especially important to look at

businesses within the same sector. For example, food retailing is able to support to low margins

because of the high volume of sales. A manufacturing industry would usually need higher margins to

offset lower sales volumes.

⚫ Low margins usually suggest poor performance but may be due to expansion costs (launching a new

product) or trying to increase market share. Lower margins than usual suggest scope for improvement.

⚫ Above-average margins are usually a sign of good management although unusually high margins may

make the competition keen to join in and enjoy the rich pickings.

2.2.2 Net profit margin

The net profit margin or operating profit margin in calculated as:

PBIT / Sales Revenue x 100%

⚫ Any changes in net profit margin should be considered further:

Are they in line with changes in gross profit margin?

Are they in line with changes in sales revenue?

As many costs are fixed they need not necessarily increase / decrease with a change in revenue.

Look for individual cost categories that have increased/decreased significantly.

⚫ This is affected by more factors than the gross profit margin but it is equally useful and if the company

does not disclose a cost of sales it may be used on its own in lieu of the gross profit percentage.

⚫ One of the many factors affecting the trading profit margin is depreciation, which is open to

considerable subjective judgment. Inter-company comparisons should be made after suitable

adjustments to align accounting policies.

⚫ By the time you have reached operating (net) profit, there are many more factors to consider. If you

are provided with a breakdown of expenses you can use this for further line-by-line comparisons.

Bear in mind that:

Some costs are fixed or semi-fixed (e.g. property costs) and therefore not expected to change in

line with revenue.

Other costs are variable (e.g. packing and distribution, and commission).

133

ACCA- FR 2021-2022 Part D-Chapter 16

2.2.3 Return on capital employed

ROCE = Profit / Capital employed x 100%

⚫ Profit is measured as:

Operating (trading) profit, or

The PBIT, i.e. the profit before taking account of any returns paid to the providers of long-term

finance.

⚫ Capital employed is measured as:

Equity, plus interest-bearing finance, i.e. the long-term finance supporting the business or

Total assets less current liabilities.

⚫ ROCE for the current year should be compared to:

The prior year ROCE;

A target ROCE;

The cost of borrowing;

Other companies ”ROCE” in the same industry;

⚫ Once calculated, ROCE should be compared with:

Previous years’ figures – provided there have been no changes in accounting policies, or suitable

adjustments have been made to facilitate comparison (note, however that the effect of not

replacing non-current assets is that their value will decrease and ROCE will increase)

The company’s target ROCE-where the company’s management has determined a target return as

part of its budget procedure, consistent failure by a part of the business to meet the target may

make it a target for disposal.

The cost of borrowings – if the cost of borrowing is say 10% and ROCE 7%, then further borrowings

will reduce EPS unless the extra money can be used in areas where the ROCE is higher than the

cost of borrowings.

Other companies in same industry – care is required in interpretation, because of the possibility,

noted above, of different accounting policies, ages of plant, etc.

The ratio also shows how efficiently a business is using its resources. If the return is very low, the business

may be better off realizing its assets and investing the proceeds in a high interest bank account! (this

may sound extreme, but should be considered particularly for a small, unprofitable business with

valuable assets such as property.) Furthermore, a low return can easily become a loss if the business

suffers a downturn.

⚫ Further points:

Treatment of associates and investments: where the profit excludes investment income the

statement of financial position carrying amounts for associates and investments should be

excluded from the capital employed.

This gives accurate measure of trading performance. If associations and investments are not

excluded, the overall profit figure should include income from investments and associates.

134

ACCA- FR 2021-2022 Part D-Chapter 16

Large cash balance are not contributing to profits and some analysts therefore deduct them from

capital employed (to compare operating profits with operating assets). However, it is usually

acceptable not to make this adjustments as ROCE is a performance measure and management

have decided to operate with that large balance.

2.2.4 Net asset turnover

Sales revenue / Capital employed (net assets) = times pa

It measures management’s efficiency in generating revenue from the net assets at its disposal: The higher,

the more efficient.

⚫ Note that this can be further subdivided into:

Non-current asset turnover (by making non-current assets the denominator) and

Working capital turnover (by making net current assets the denominator).

⚫ Relationship between ratios:

ROCE can be subdivided into profit margin and asset turnover.

⚫ Profit margin x Assets turnover = ROCE

PBIT / Sales Revenue x Sales Revenue / Capital employed = PBIT / Capital employed

Profit margin is often seen as an indication of the quality of products or services supplied (top-of –

range products usually have higher margins).

Assets turnover is often seen as a measure of how intensively the assets are worked.

⚫ A trade-off may exist between margin and asset turnover.

Low – margin businesses (e.g. food retailers) usually have a high asset turnover.

Capital – intensive manufacturing industries usually have relatively low asset turnover but high

margins. (e.g. electrical equipment manufactures).

⚫ Two completely different strategies can achieve the same ROCE.

Sell goods at a high profit margin with sales volume remaining low (e.g. designer dress shop);

Sell goods at a low profit margin with very high sales volume(e.g. discount clothes store);

2.3 Liquidity and working capital ratios:

2.3.1 Current ratio

Current assets / Current Liabilities

⚫ The current ratio measures the adequacy of current assets to meet the liabilities as they fall due. A

high or increasing figure may appear safe but should be regarded with suspicion as it may be due to:

High levels of inventory and receivables (check working capital management ratios);

High cash levels which could be put to better use (e.g. by investing in non-current assets);

⚫ Traditionally, a current ratio of 2:1 or higher was regarded as appropriate for most businesses to

maintain creditworthiness. However, more recently a figure of 1.5:1 is regarded as the norm.

135

ACCA- FR 2021-2022 Part D-Chapter 16

⚫ The current ratio should be looked at in the light of what is normal for the business. For example,

supermarkets tend to have low current ratios because:

There are few trade receivables;

There is a high level of trade payables;

There is usually very tight cash control, to fund investment in developing new sites and improving

sites;

⚫ It is also worth considering:

Availability of further finance, e.g. is the overdraft at the limit? – very often this information is

highly relevant but is not disclosed in the accounts;

Seasonal nature of the business – one way of doing this is to compare the interest charges in the

income statement with the overdraft and other loans in the statement of financial position; if the

interest rate appears abnormally high, this is probably because the company has had higher levers

of borrowings during the year;

Long-term liabilities, when they fall due and how will they be financed;

Nature of the inventory – where inventories are slow moving, the quick ratio probably provides a

better indicator of shore-term liquidity;

2.3.2 Quick ratio/Acid test ratio:

(Current assets – Inventories) / Current liabilities

⚫ The quick ratio is also known as the acid test ratio because by eliminating inventory from current

asses it provides the acid test of whether the company has sufficient liquid resources (receivables and

cash) to settle its liabilities. Normal levels for the quick ratio range from 1:1 to 0.7:1. Like the current

ratio it is relevant to consider the nature of the businesses (again supermarkets have very low quick

ratios).

⚫ Some times the quick ratio is calculated on the basis of a six-week time-frame (i.e. the quick assets

are those which will turn into cash in six weeks; quick liabilities are those which fall due for payment

within six weeks). This basis would usually include the following in quick assets:

Bank, cash and short-term investments;

Trade receivables;

Thus excluding prepayments and inventory.

⚫ Quick liabilities would usually include:

Bank overdraft which is repayable on demand;

Trade payables, tax and social security;

Dividends;

⚫ Income tax liabilities may be excluded:

When interpreting the quick ratio, care should be taken over the status of the bank overdraft. A

company with a low quick ratio may actually have no problem in paying its amounts due if sufficient

overall overdraft facilities are available.

136

ACCA- FR 2021-2022 Part D-Chapter 16

2.3.3 Inventory turnover period

Inventory / Costs of goods sold x 365 days

This is normally expressed in days: An alternative is to express the inventory turnover period as a number

of times:

Cost of sales / Inventory = times pa

⚫ An increasing number of days (or a diminishing multiple) implies that inventory is turning over less

quickly which is regarded as a bad sign as it may indicate:

Lack of demand for the goods;

Poor inventory control;

An increase in costs (storage, obsolescence, insurance, damage);

⚫ However, it may not necessarily be bad where management are:

Buying inventory in larger quantities to take advantage of trade discounts, or

Increasing inventory levels to avoid stock outs;

⚫ Inventory days:

Year-end inventory is normally used in the calculation of inventory turnover. An average (based on

the average of year-start and year-end inventories) may be used to have smoothing effect,

although this may dampen the effect of a major change in the period.

Inventory turnover ratios very enormously with the nature of the business. For example, a

fishmonger selling fresh fish would have an inventory turnover period of 1-2 days, whereas a

building contractor may have an inventory turnover period of 200 days. Manufacturing companies

may have an inventory turnover ratio of 60-100 days: this period is likely to increase as the goods

made become larger and more complex.

For large and complex items (e.g. rolling stock or aircraft) there may be sharp fluctuations in

inventory turnover according to whether delivery took place just before or just after the year end.

⚫ A manufacturer should take into consideration:

Reliability of suppliers: if the supplier is unreliable it is prudent to hold more raw materials;

Demand: if demand is erratic it is prudent to hold more finished goods;

2.3.4 Receivables collection period:

Trade receivables / credit sales x 365 days

⚫ The collection period should be compared with:

The stated credit policy;

Previous period figures;

⚫ Increasing accounts receivables collection period is usually a bad sign suggesting lack of proper credit

control which may lead to irrecoverable debts.

It may, however, be due to:

A deliberate policy to attract more trade, or

A major new customer being allowed different terms.

137

ACCA- FR 2021-2022 Part D-Chapter 16

⚫ Falling receivables days is usually a good sign, though it could indicate that the company is suffering

a cash shortage.

⚫ Receivable days:

The trade receivables used may be a year-end figure or the average for the year. Where an average

is used to calculate the number of days, the ratio is the average number of days’ credit taken by

customers.

⚫ For many business total sales revenue can safely be used, because cash sales will be insignificant. But

cash-based businesses like supermarkets make the substantial majority of their sales for cash, so the

receivables period calculated by reference to credit sales only.

⚫ The result should be compared with the stated credit policy. A period of 30 days or ‘at the end of the

month following delivery’ are common credit terms.

⚫ The receivables days ratio can be distorted by:

Using year-end figures which do not represent average receivables.

Factoring of accounts receivables which result in very low trade receivables.

Sales on unusually long credit terms to some customers.

2.3.5 Payable payment period:

Trade payables / Credit purchases x 365 days

This represents the credit period taken by the company from its suppliers.

⚫ The ratio is always compared to previous years:

A long credit period may be good as it represents a source of free finance;

A long credit period may indicate that the company is unable to pay more quickly because of

liquidity problems;

⚫ If the credit period is long:

The company may develop a poor reputation as a slow payer and may not be able to find new

suppliers;

Existing suppliers may decide to discontinue suppliers;

The company may be losing out on worthwhile cash discounts;

⚫ In most sets of financial statements (in practice and in examinations) the figure for purchases will not

be available therefore cost of sales is normally used as an approximation in the calculation of the

accounts payable payment period. 2.4 Long-term financial stability

The main points to consider when assessing the longer-term financial position are:

⚫ Gearing

⚫ Over trading

138

ACCA- FR 2021-2022 Part D-Chapter 16

2.4.1 Gearing

⚫ Gearing ratios indicate:

The degree of risk attached to the company; and

The sensitivity of earnings and dividends to changes in profitability and activity level.

⚫ Preference share capital is usually counted as part of debt rather than equity since it carriers the right

to a fixed rate of dividend which is payable before the ordinary shareholders have any right to a

dividend.

High and Low gearing:

⚫ In highly geared business:

A large proportion of fixed-return capital is used;

There is a greater risk of insolvency;

Returns to shareholders will grow proportionately more if profits are growing;

⚫ Low-geared business:

Provide scope to increase borrowings when potentially profitable projects are available;

Can usually borrow more easily;

⚫ Not all companies are suitable for a highly-geared structure. A company must have two fundamental

characteristics if it is to use gearing successfully.

⚫ Loan stock interest must be paid whether or not profit are earned. A company with erratic profits

may have insufficient funds in a bad year with which to pay the interest. This would result in the

appointment of a receiver and possibly the liquidation of the company.

⚫ Most issues of loan capital are secured on some or all of the company’s assets which must be suitable

for the purpose. A company with most of its capital invested in fast depreciating assets or inventory

subject to rapid changes in demand and price would not be suitable for high gearing.

⚫ The classic examples of companies that are suited to high gearing are those in property investment

and the hotel / leisure services industry. These companies generally enjoy relatively stable profits and

have assets which are highly suitable for charging. Nonetheless, these are industries that could be

described as cyclical.

⚫ Companies not suited to high gearing would include those in the extractive, and high-tech, industries

where constant changes occur. These companies could experience erratic profits and would generally

have inadequate assets to pledge as security.

These are two methods commonly used to express gearing as follows.

⚫ Debt / equity ratio:

Loans + Preference share capital / Ordinary share capital + Reserves + Non-controlling interest

⚫ Percentage of capital employed represented by borrowings:

Loans + Preference share capital / ordinary share capital + Reserves + Non-controlling interest +

Loans + Preference share capital

139

ACCA- FR 2021-2022 Part D-Chapter 16

2.4.2 Interest cover:

Interest cover = PBIT / Interest Payable

⚫ Interest cover indicates the ability of a company to pay interest out of profits generated:

Low interest cover indicates to shareholders that their dividends are at risk (because most profits

are eaten up by interest payments) and

The company may have difficulty financing its debts if the profits fall.

Interest cover of less than two is usually considered unsatisfactory.

⚫ A business must have a sufficient level of long-term capital to finance its long-term investment in non-

current assets. Part of the investment in current assets would usually be financed by relatively

permanent capital with the balance being provided by credit from suppliers and other short term

borrowings. Any expansion in activity will normally require ad broadening of the long-term capital

base, without which “overtrading may develop”.

⚫ Suitability of finance is also a key factor. A permanent expansion of a company’s activities should not

be financed by temporary, short-term borrowing. On the other hand, a short-term increase in activity

such as the “January sales” in a retail trading company could ideally be financed by overdraft.

⚫ A major addition to non-current assets such as the construction of a new factory would not normally

be financed on a long-term basis by overdraft. It might be found, however, that the expenditure was

temporarily financed by short-term loans until construction was completed, when the overdraft

would be “funded” by a long-term borrowings secured on the completed building.

2.4.3 Overtrading:

⚫ Overtrading arises where a company expands its sales revenue fairly rapidly without securing

additional long-term capital adequate for its needs. The symptoms of overtrading are :

Inventory increasing, possibly more than proportionately to revenue;

Receivables increasing, possibly more than proportionately to revenue;

Cash and liquid assets declining at a fairly alarming rate;

Trade payables increasing rapidly;

⚫ The symptoms of overtrading simply imply that the company has expanded without giving proper

thought to the necessity to expand its capital base. It has consequently continued to rely on its trade

payables and probably its bank overdraft to provide the additional finance required. It will reach a

stage where suppliers will withhold further supplies and bankers will refuse to honour further

cheques until borrowings are reduced. The problem is that borrowings cannot be reduced until sales

revenue is earned, which in turn cannot be achieved until production is completed, which in turn is

dependent upon materials being available and wages paid. Overall result – deadlock and rapid

financial collapse.

⚫ This is a particularly difficult stage for small – to medium – sized companies. They have reached a

stage in their life when conventional payables and overdraft facilities are being stretched to the

maximum, but they are probably too small to manage a flotation. In many cases, by proper planning,

the company can arrange fixed-term loan funding from the bank rather than relying exclusively on

overdraft finance.

140

ACCA- FR 2021-2022 Part D-Chapter 16

2.5 Investor ratios:

2.5.1 Earnings per share (EPS)

EPS is used primarily as a measure of profitability, so an increasing EPS is seen as a good sign. EPS is also

used to calculate the price earnings ratio which is dealt with below.

The Limitation of EPS may be listed as follows.

⚫ In times of rising prices EPS will increase as profits increase. Thus any improvement in EPS should be

viewed in the context of the effect of price level changes on the company’s profits.

⚫ Where there is a new share issue for cash, the shares are included for, say, half the year on the

grounds that earnings will also increase for half of the year. However, in practice a new project funded

by that cash does not begin generating normal returns immediately, so a new share issue is often

accompanied by a decrease in EPS.

⚫ EPS is dependent on an earnings figure which is subject to many judgments. Some elements off that

earnings figure, such as movements on provisions, are particularly sensitive to different judgments.

⚫ A single earnings figure should not be used as a key performance measure. This is to take a far too

simplistic approach to the analysis of performance.

⚫ EPS cannot be used as a basis of comparison between companies, as the number of shares in issue in

any particular company is not related to the amount of capital employed. For example, two

companies may have the same amount of capital employed but one company has 100,000 $ 1 shares

in issue and reserves of $ 4,900,000. Another company may have 5 million 50c shares in issue and

reserves of $ 2,500,000. If earnings are the same, EPS is different.

⚫ EPS is an historical figure based on historical accounts. This is a disadvantage where it is used for a

forward-looking figure such as the price earnings ratio.

⚫ The diluted EPS (DEPS) is a theoretical measure of the effect of dilution on the basic EPS. DEPS should

serve as a warning to equity shareholders that their future earnings will be affected by diluting factors.

Thus, notes in the accounts relating to convertible loan stock, convertible preference shares and share

options should all be analysed carefully.

2.5.2 P/E ratio:

P/E ratio = Current share price / Latest EPS

⚫ Represents the market’s view of the future prospects of the share.

⚫ High P/E suggests that high growth is expected.

This is the most widely referred to stock market ratio, also commonly described as an earning multiple.

It is calculated as the “purchase of a number of years’ earnings, but it represents the market’s consensus

of the future prospects of that share. The higher the P/E ratio, the faster the growth the market is

expecting in the company’s future EPS. Correspondingly, the lower the P/E ratio, the lower the expected

future growth.

2.5.3 Dividend yield:

Dividend Yield = Dividend per share / Current share price

⚫ Can be compared to the yields available on other investment possibilities.

⚫ The lower the dividend yield, the more the market is expecting future growth in the dividend, and

vice versa.

141

ACCA- FR 2021-2022 Part D-Chapter 16

2.5.4 Dividend cover:

Dividend cover = Profit after tax / Dividends

⚫ This is the relationship between available profits and the dividends payable out of the profits.

⚫ The higher the dividend cover, the more likely it is that the current dividend level can be sustained in

the future.

2.6 Limitations of financial statements and ratio analysis:

⚫ Historical cost accounts

Ratios are a tool to assist analysis.

They help to focus attention systematically on important areas and summarize information in an

understandable form.

They assist in identifying trends and relationships.

However ratios are not predictive if they are based on historical information.

They ignore future action by management.

They can be manipulated by window dressing or creative accounting.

They may be distorted by differences in accounting polices.

⚫ Creative accounting/window dressing

Creative accounting refers to the accounting practices that are designed to mislead the view that the

users of financial statement has on an entitiy’s underlying economic performance. Typically creative

accounting is used to increase profit, inflate asset values or understate liabilities.

In the past companies could smooth profit to maintain a steady upward trend by making use of general

provision. An upward profit trend is reassuring to both existing and potential investors or of benefit to

bonus-seeking directors. As the restrictions on provisions have tightened, companies have found other

ways to manipulate profit, such as unsuitable revenue recognition or inappropriate accruals.

Creative accounting techniques can also be used to manipulate the gearing level of a company. A

company that is highly geared has high interest payments that reduce the amount of distributable profit

available to shareholders and increases the risk associated with the company, making it more difficult

to obtain future lending.

Other reasons for creative accounting could include the desire to influence share price, to keep the

company’s financial results within agreed limits set by creditors, personal incentives or to pay less tax.

Window dressing is a method of carrying out transactions in order to distort the position shown by the

financial statements and generally improve the position shown by them.

⚫ Change in accounting policies:

It is necessary to be able to assess the impact of accounting policies on the calculation of ratios.

Comparison between businesses that follow different polices becomes a major issue if IFRS Standards

give either choice a judgment to companies i.e. IAS 40 or IAS 16.

142

ACCA- FR 2021-2022 Part D-Chapter 16

⚫ Limitations of ration analysis

Companies with different reporting date would have different financial results in a highly seasonal

industry.

Company with major asset acquisition the end of the accounting period would invalidate the comparison

with another company even in the same industry.

Although there are general guidelines, there is no such thing as an ideal ratio. A quick ratio of less than

1:1 would be acceptable in some businesses, but dangerously low for many others.

Unless ratios are calculated on a uniform basis, from uniform data, comparisons can be very misleading.

The statement of financial position shown in the F/S may not be representative of the financial position

at other times in the year. Many businesses set the end of their accounting period to a date on which

there is a relatively low amount of trading activity.

Ratios based on historical cost accounts do not give a true picture of trends from year to year. An

apparent increase in profit may not be a true increase, because of the effects of inflation.

Financial statements only reflect those activities which can be expressed in money term. They do not

give a complete picture of the activities of a business.

The application of accounting policies in the preparation of F/S must be understood when attempting

to interpret financial ratios.

The earning power of a business may well be affected by factors which are not reflected in the F/S. Thus

these do not necessarily represent a complete picture of a business, but only a collection of those parts

which can be translated into money terms.

Ratio must not be used as the sole test of efficiency.

A few simple ratios do not provide an automatic means of running a company.

⚫ Inter-firm comparisons:

It can be useful to compare ratios for an individual company with those of other firms in the same industry.

However, comparing the financial statements of similar businesses can be misleading because:

The businesses may use different accounting polices.

Ratios may not be calculated according to the same formula.

Large organization can achieve economies of scale.

Companies with the same industry can serve completely different market and there may be differences

in sales mix and product range. 2.7 Additional information:

⚫ Budgeted figures

⚫ Other management information

⚫ Industry averages

⚫ Figures for a similar business

⚫ Figures for the business over a period of time.

143

ACCA- FR 2021-2022 Part D-Chapter 16

Non-financial information:

⚫ Market share

⚫ Key employee information

⚫ Sales mix information

⚫ Product range information

⚫ The size of the order book

⚫ The long-term plans of management.

2.8 The not-for-profit and public sector entities

⚫ Not-for-profit and public sector entities produce financial statements in the same way as profit-making

entities do but, while they are expected to remain solvent, their performance cannot be measured simply

by the bottom line.

⚫ A public sector entity is not expected to show a profit or to underspend its budget. In practice, central

government and local government departments know that if they underspend the budget, next year's

allocation will be correspondingly reduced. This leads to a rash of digging up the roads and other

expenditure just before the end of the financial year as councils strive to spend any remaining funds.

⚫ Private and public sector entities are judged principally on the basis of what they have achieved, not how

much or how little they have spent in achieving it. So how is performance measured?

⚫ Public sector entities

These will have performance measures laid down by government. The emphasis is on economy, efficiency

and effectiveness. Departments and local councils have to show how they have spent public money and

what level of service they have achieved. Performance measurement will be based on Key Performance

Indicators (KPIs). Examples of these for a local council could be:

Number of homeless people rehoused

% of rubbish collections made on time

Number of children in care adopted

Public sector entities use the services of outside contractors for a variety of functions. They then have to be

able to show that they have obtained the best possible value for what they have spent on outside services.

This principle is usually referred to as Value For Money (VFM). In the UK, local authorities are required to

report under a system known as Best Value. They have to show that they applied 'fair competition' in awarding

contracts.

⚫ Best Value is based on the principle of the 'four Cs':

Challenging why, how and by whom a service is provided.

Comparing performance against other local authorities.

Consulting service users, the local community etc.

Using fair Competition to secure efficient and effective services.

144

ACCA- FR 2021-2022 Part D-Chapter 16

2.9 Summary of ratios

Types of ratios Name of ratio Formula

Profitability

Return on capital employed (ROCE) = PBIT

Equity + long-term debt

Return on equity = PAT

OSC + Reserves

Asset turnover = Sales

Capital employed

Net profit margin = PBIT

Sales

Gearing

Debt-to-equity ratio = long-term liabilities

Total equity

Interest cover = PBIT

Interest payment

Liquidity

Current ratio = Current asset

Current liability

Quick ratio (Acid Test) = Current asset – stock

Current liabilities

Working capital

Inventory turnover period = Inventory ×365 days

Cost of sales

Receivables collection period = Trade receivable×365 days

Credit Sales

Payable payment period = Trade payable×365 days

Credit purchases

Operating cycle =

Inventory turnover period

+ Receivables collection period

– Payable payment period

Investment ratio

EPS = PAT

No. of ordinary shares

P/E ratio = Share price

EPS

Dividend cover = EPS

Dividend per share

Dividend yield Dividend per share

Current share price

2.10 Group aspects

When analysing financial performance it is important to consider the effect of parent/subsidiary relationships.

Ratios based on consolidated financial statements can obscure the performance of the parent or the subsidiary.

It is particularly important to be able to isolate the effects of acquisitions or disposals of subsidiaries on the

financial statements and on the ratios.

145

ACCA- FR 2021-2022 Part D-Chapter 16

2.10.1 Acquisitions

⚫ Ratio analysis is particularly relevant when a company is considering an acquisition. In reviewing the

financial statements of the acquiree, the acquirer needs to be alert to any evidence of window

dressing. Have transactions been undertaken to boost the results of the acquiree prior to the sale?

And are there favourable business relationships or supply chains that will no longer apply post-

acquisition?

⚫ It will also be important to review the effect of the acquisition on the group financial statements. This

will require calculating underlying ratios, to see what results would have looked like if the acquisition

had not taken place.

2.10.2 Disposals

⚫ When a disposal takes place, IFRS 5 requires the results of the discontinued operation to be separately

presented, so that users can assess properly the performance of the continuing operations. In

carrying out ratio analysis it is also necessary to eliminate the effects of group companies or trading

operations that have been disposed of.

⚫ This means removing the results of the disposal from profit or loss (including any profit or loss on

disposal, which is a one-off item) and removing the net assets of the operation disposed of from

capital employed. Then important ratios such as gross profit %, operating profit % and ROCE can be

calculated for the remaining business.

⚫ In this way it will be possible to judge whether the disposal had a favorable result – it may even turn

out that the operation disposed of was the most profitable part of the business! It may also be

possible to judge whether the sale was correctly priced – perhaps that profitable subsidiary or division

should have been priced a bit higher.

Example 1 Past exam Dec 2010 Q3

Hardy is a public listed manufacturing company. Its summarized financial statements for the year ended 30

September 20Y0 (and 20X9 comparatives) are:

Income statements for the year ended 30 September:

20Y0 20X9

$’000 $’000

Revenue 29,500 36,000

Cost of sales (25,500) (26,000)

Gross profit 4,000 10,000

Distribution costs (1,050) (800)

Administrative expenses (4,900) (3,900)

Investment income 50 200

Finance costs (600) (500)

Profit (loss) before taxation (2,500) 5,000

Income tax (expense) relief 400 (1,500)

Profit (loss) for the year (2,100) 3,500

146

ACCA- FR 2021-2022 Part D-Chapter 16

Statements of financial position as at 30 September:

20Y0 20X9

$’000 $’000 $’000 $’000

Assets

Non-current assets

Property, plant and equipment 17,600 24,500

Investments at fair value through profit or

loss 2,400 4,000

Current assets 20,000 28,500

Inventory and work-in-progress 2,200 1,900

Trade receivables 2,200 2,800

Tax asset 600 nil

Bank 1,200 6,200 100 4,800

–––––– ––––––– –––––– –––––––

Total assets 26,200 33,300

––––––– –––––––

Equity and liabilities

Equity

Equity shares of $1 each 13,000 12,000

Share premium 1,000 nil

Revaluation reserve nil 4,500

Retained earnings 3,600 6,500

––––––– –––––––

17,600 23,000

Non-current liabilities

Bank loan 4,000 5,000

Deferred tax 1,200 700

Current liabilities Trade payables

3,400 2,800

Current tax payable nil 3,400 1,800 4,600

–––––– ––––––– –––––– –––––––

Total equity and liabilities 26,200 33,300

––––––– –––––––

The following information has been obtained from the Chairman’s Statement and the notes to the financial

statements:

‘Market conditions during the year ended 30 September 20Y0 proved very challenging due largely to difficulties in

the global economy as a result of a sharp recession which has led to steep falls in share prices and property values.

Hardy has not been immune from these effects and our properties have suffered impairment losses of $6 million

in the year.’

The excess of these losses over previous surpluses has led to a charge to cost of sales of $1·5 million in addition to

the normal depreciation charge.

147

ACCA- FR 2021-2022 Part D-Chapter 16

‘Our portfolio of investments at fair value through profit or loss has been ‘marked to market’ (fair valued) resulting

in a loss of $1·6 million (included in administrative expenses).’

There were no additions to or disposals of non-current assets during the year.

‘In response to the downturn the company has unfortunately had to make a number of employees redundant

incurring severance costs of $1·3million (included in cost of sales) and undertaken cost savings in advertising and

other administrative expenses.’

‘The difficulty in the credit markets has meant that the finance cost of our variable rate bank loan has increased

from 4·5% to 8%. In order to help cash flows, the company made a rights issue during the year and reduced the

dividend per share by 50%.’ ‘Despite the above events and associated costs, the Board believes the company’s underlying performance has

been quite resilient in these difficult times.’

Required:

Analyse and discuss the financial performance and position of Hardy as portrayed by the above financial

statements and the additional information provided.

Your analysis should be supported by profitability, liquidity and gearing and other appropriate ratios (up to 10

marks available). (25 marks)

Example 2 Past exam June 2015 Q2

Yogi is a public company and extracts from its most recent financial statements are provided below:

Statements of profit or loss for the year ended 31 March

20X5 20X4

$’000 $’000

Revenue 36,000 50,000

Cost of sales -24,000 -30,000

––––––– –––––––

Gross profit 12,000 20,000

Profit from sale of division (see note (i)) 1,000 nil

Distribution costs -3,500 -5,300

Administrative expenses -4,800 -2,900

Finance costs -400 -800

––––––– –––––––

Profit before taxation 4,300 11,000

Income tax expense -1,300 -3,300

––––––– –––––––

Profit for the year 3,000 7,700

––––––– –––––––

148

ACCA- FR 2021-2022 Part D-Chapter 16

Statements of financial position as at 31 March

20X5 20X4

$’000 $’000 $’000 $’000

ASSETS

Non-current assets

Property, plant and equipment 16,300 19,000

Intangible – goodwill nil

2,000

16,300 21,000

Current assets

Inventory

21,000

3,400 5,800

Trade receivables 1,300 2,400

Bank 1,500

6,200 nil

8,200

Total assets 22,500 29,200

EQUITY AND LIABILITIES

Equity

Equity shares of $1 each 10,000 10,000 Retained earnings 3,000 4,000

Non-current liabilities

10% loan notes

13,000

14,000

4,000 8,000

Current liabilities

Bank overdraft

nil 1,400

Trade payables 4,300 3,100

Current tax payable 1,200 5,500 2,700 7,200

Total equity and liabilities 22,500 29,200

Notes

(i) On 1 April 20X4, Yogi sold the net assets (including goodwill) of separately operated division of its business

for $8m cash on which it made a profit of $1m. This transaction required shareholder approval and, in

order to secure this, the management of Yogi offered shareholders a dividend of 40 cents for each share

in issue out of the proceeds of the sale. The trading results of the division which are included in the

statement of profit or loss for the year ended 31 March 20X3 above are:

$000

Revenue 18,000

Cost of sales (10,000)

Gross profit 8,000

Distribution costs (1,000)

Administrative expenses (1,200)

Profit before interest and tax 5,800

149

ACCA- FR 2021-2022 Part D-Chapter 16

(ii) The following selected ratios for Yogi have been calculated for the year ended 31 March 20X4(as reported

above):

Gross profit margin 40.0%

Operating profit margin 23.6%

Return on capital employed

(profit before interest and tax / (total assets – current liabilities)

53.6%

Net asset turnover 2.27 times

Required

(a)Calculate the equivalent ratios for Yogi:

(i) For the year ended 31 March 20X4, after excluding the contribution made by the division that has been

sold; and

(ii) For the year ended 31 March 20X5, excluding the profit on the sale of the division. (5 marks)

(b) Comment on the comparative financial performance and position of Yogi for the year ended 31 March 20X5.

(15 marks)

150